0% found this document useful (0 votes)
596 views

Geometry - 02 - Triangles

Learn the basics of high school geometry. In this document, we assume some basic knowledge on angles and transversals, and build from there. We look at Isosceles and Equilateral triangles, area and perimeter of a triangle, Hero's formula (also called Heron's formula) for area, the famous Pythagorean Theorem. We also look at some advanced topics like angle bisectors, medians, altitudes and their related cirles (incircle, circumcircle, etc). We apply the concepts we have learnt in the context of competition problems from AMC, CEMC, NMTC and other math competitions.

Uploaded by

Azizmanva
Copyright
© © All Rights Reserved
Available Formats
Download as PDF, TXT or read online on Scribd
0% found this document useful (0 votes)
596 views

Geometry - 02 - Triangles

Learn the basics of high school geometry. In this document, we assume some basic knowledge on angles and transversals, and build from there. We look at Isosceles and Equilateral triangles, area and perimeter of a triangle, Hero's formula (also called Heron's formula) for area, the famous Pythagorean Theorem. We also look at some advanced topics like angle bisectors, medians, altitudes and their related cirles (incircle, circumcircle, etc). We apply the concepts we have learnt in the context of competition problems from AMC, CEMC, NMTC and other math competitions.

Uploaded by

Azizmanva
Copyright
© © All Rights Reserved
Available Formats
Download as PDF, TXT or read online on Scribd
You are on page 1/ 152

TRIANGLES

REVISION: 1578

01 SEPTEMBER 2021
AZIZ MANVA
[email protected]

ALL RIGHTS RESERVED


Get all the files at: https://bit.ly/azizhandouts
Aziz Manva ([email protected])

TABLE OF CONTENTS
6.5 Generalizations 62
TABLE OF CONTENTS ................................. 2
Part III: Right Triangles 63
Part I: Basics 4
7. PYTHAGOREAN THEOREM ................. 63
1. ANGLES IN A TRIANGLE ....................... 4 7.1 Basics 63
1.1 Terminology 4 7.2 Multiple and Nested Triangles 68
1.2 Sum of Angles 4 7.3 Ladders and Ramps 69
1.3 Sum of Angles: Applications 9 7.4 Flagpoles and Flying Animals 71
1.4 Exterior Angles 11 7.5 Directions 72
1.5 Five Pointed Stars 15 7.6 Review and Challenge 75

2. TYPES OF TRIANGLES......................... 17 8. USING PYTHAGORAS THEOREM........ 76


2.1 Equilateral Triangles 17 8.1 Algebraic Applications 76
2.2 Right Angled Triangles 19 8.2 Geometrical Applications: 2D 77
2.3 Acute, Obtuse and Scalene Triangles 21 8.3 Geometrical Applications: 3D 81

3. ISOSCELES TRIANGLES ....................... 22 9. SPECIAL RIGHT TRIANGLES ............... 84


3.1 Angles Basics 22 9.1 Isosceles Right Triangles 84
3.2 Angles: Applications 26 9.2 Applications 86
3.3 Sides 34 9.3 Equilateral Triangles 89
3.4 Area 35 9.4 More with Area 91
3.5 Review 37 9.5 𝟑𝟎° − 𝟔𝟎° − 𝟗𝟎° Triangles 92
3.6 Applications (Optional) 37 9.6 Review and Challenge 95

Part II: Area and Perimeter 39 Part IV: Lines and Circles 96
4. BASICS ................................................... 39 10. ANGLE BISECTORS ............................ 96
4.1 Summary and Formulas 39 10.1 Angle Chasing (Bisectors) 96
4.2 Perimeter 39 10.2 Angle Bisector Theorem 99
4.3 Right Triangles: Area 41 10.3 Ratios 103
4.4 General Triangles 43 10.4 Equilateral Triangles 106

5. APPLICATIONS ..................................... 46 11. INCENTRE AND INRADIUS ............ 109


5.1 Ratios 46 11.1 Incenter 109
5.2 Similarity 46 11.2 Inradius: Definition 111
5.3 Complementary Areas 47 11.3 Calculating Inradius 114
5.4 Other Topics 47 11.4 Tangents 115
11.5 Isosceles and Equilateral Triangles 120
6. HERON’S FORMULA ............................ 49 11.6 Right Triangles 121
6.1 Heronian Triangles 49 11.7 Review 124
6.2 Non-Heronian Triangles 55
6.3 Special Cases 57
12. PERPENDICULAR BISECTORS....... 126
6.4 Applications 61 12.1 Perpendicular Bisectors 126

P a g e 2 | 152
Get all the files at: https://bit.ly/azizhandouts
Aziz Manva ([email protected])

12.2 Circumcenter 129 Part V: Triangle Inequality 143


12.3 Circumcircle 131
12.4 Equilateral Triangles 132 14. TRIANGLE INEQUALITY ................ 143
12.5 Circumradius 134
12.6 Review 138 14.1 Basics 143
14.2 Maximum, Minimum and Range 148
13.MEDIANS AND ALTITUDES ............140 14.3 Geometrical Inequalities 149
14.4 Review 149
13.1 Medians 140
14.5 AMC Questions 150
13.2 Altitudes 141
13.3 Equilateral Triangles 142

P a g e 3 | 152
Get all the files at: https://bit.ly/azizhandouts
Aziz Manva ([email protected])

PART I: BASICS
1. ANGLES IN A TRIANGLE
1.1 Terminology
A. Definition
A triangle is a closed three-sided figure.
It has three sides, three angles and three vertices.
B. Terminology and Basics

Based on Angles Type of Triangle Properties


All the angles are acute Acute-angled
One angle is obtuse Obtuse-angled You cannot have a second obtuse angle
One angle is a right-angle Right Angled Remaining two angles are
complementary

Based on Sides
No two sides are equal Scalene
At least two sides are Isosceles Angles opposite equal sides are equal
equal
All three sides are equal Equilateral Angles are also equal - 60 degrees
An Equilateral triangle is also isosceles

1.2 Sum of Angles


1.1: Sum of Angles of a Triangle
The sum of angles of a triangle is 𝟏𝟖𝟎°

A. Numerical Applications
We can use the property of the sum of angles of a triangle to find missing values.

Example 1.1 𝐴
See the diagram alongside (which is not drawn to scale), and then answer each
question independently.
1. If ∠𝐴 = 65°, and ∠𝐵 = 70°, then find the measure of ∠𝐶.
2. If ∠𝐵 = 55°, and ∠𝐶 = 58°, then find the measure of ∠𝐴.
3. If ∠𝐴 = 87°, and ∠𝐶 = 48°, then find the measure of ∠𝐵. 𝐵 𝐶

Example 1.2
Find the values of the missing angle in a triangle with angles:
A. 35, 45, 𝑏
B. 25, 75, 𝑐
C. 90, 60, 𝑎

Example 1.3
The measures of the three interior angles of a triangle are 50, 55 and 𝑥. What is the degree measure of the
largest interior angle of this triangle? (MathCounts 2006 School Sprint)

P a g e 4 | 152
Get all the files at: https://bit.ly/azizhandouts
Aziz Manva ([email protected])

𝑥 = 180 − 50 − 55 = 75

Example 1.4
What is the measure of angle 4 if 𝑚∠1 = 76, 𝑚∠2 = 27 and 𝑚∠3=17? (MathCounts
2008 State Countdown)

Sum of Angles of a Triangle


Let’s assume that angles
∠1 + ∠2 + ∠3 = 𝑥, ∠5 + ∠6 = 𝑦
By angles in a triangle:
𝑥 + 𝑦 = 180 ⇒ 𝑦 = 180 − 𝑥 = 60

∠4 = 180 − 𝑦 = 180 − 60 = 120

Sum of Angles of a Quadrilateral


𝑅𝑒𝑓𝑙𝑒𝑥 𝐴𝑛𝑔𝑙𝑒 𝑜𝑓 ∠4 = 360 − 76 − 27 − 17 = 240
∠4 = 360 − 240 = 120

Example 1.5
Four points 𝐵, 𝐴, 𝐸, and 𝐿 are on a straight line, as shown. The point G is off the line
so that ∠𝐵𝐴𝐺 = 120 and ∠𝐺𝐸𝐿 = 80. If the reflex angle at 𝐺 is 𝑥, then what does 𝑥
equal? (CEMC 2005 Gauss 8)

∠𝐴 = 180 − 120 = 60
∠𝐸 = 180 − 80 = 100
∠𝐺 = 180 − 60 − 100 = 20
𝑅𝑒𝑓𝑙𝑒𝑥 ∠𝐺 = 360 − 20 = 340

B. Conceptual Applications
The sum of angles of a triangle limits values that the angles can take. These are important in ruling out cases
which are not possible.

Example 1.6
How many triangles can be formed such that exactly one of their angles is acute?

Let the measure of the angles be 𝑎, 𝑥 and 𝑦, where a is the acute angle. Then:
𝑎 + 𝑥 + 𝑦 = 180

Since neither 𝑥 nor 𝑦 are acute, the minimum value they can take is 90°. Substitute this in the above equation:
𝑎 + 90 + 90 = 180 ⇒ 𝑎 + 180 = 180 ⇒ 𝑎 = 0° ⇒ 𝑵𝒐𝒕 𝑽𝒂𝒍𝒊𝒅

Hence, even if we take the minimum value of 90° for the remaining two angles, we do not get a valid triangle.
Hence, such a triangle is not possible.
There are zero such triangles.

Example 1.7
In a triangle, what is the maximum number of:
A. Acute Angles

P a g e 5 | 152
Get all the files at: https://bit.ly/azizhandouts
Aziz Manva ([email protected])

B. Right Angles
C. Obtuse Angles

Part A
All three angles can be acute. For example, consider:
Δ𝐴𝐵𝐶: ∠𝐴 = ∠𝐵 = ∠𝐶 = 60°

Part B
A triangle with one right angle is possible. For example, consider:
Δ𝐴𝐵𝐶: ∠𝐴 = 90°, ∠𝐵 = ∠𝐶 = 45°

Consider a triangle with two right angles


Δ𝐴𝐵𝐶: ∠𝐴 = ∠𝐵 = 90° ⇒ ∠𝐴 + ∠𝐵 = 180° ⇒ ∠𝐶 = 0° ⇒ 𝑵𝒐𝒕 𝑽𝒂𝒍𝒊𝒅
Hence, two right angles are not possible.

Part C
A triangle with one obtuse angle is possible. For example, consider:
Δ𝐴𝐵𝐶: ∠𝐴 = 100°, ∠𝐵 = ∠𝐶 = 40°

Consider a triangle with two obtuse angles:


Δ𝐴𝐵𝐶: ∠𝐴 > 90°, ∠𝐵 > 90° ⇒ ∠𝐴 + ∠𝐵 > 180° ⇒ 𝑵𝒐𝒕 𝑽𝒂𝒍𝒊𝒅
Hence, two obtuse angles are not possible.

Example 1.8
If one angle in a triangle is obtuse, then the other two angles must be:

Consider a triangle with an obtuse angle:


Δ𝐴𝐵𝐶: ∠𝐴 > 90°, ∠𝐵 + ∠𝐶 < 90° ⇒ ∠𝐵, ∠𝐶 𝑎𝑟𝑒 𝑎𝑐𝑢𝑡𝑒
Hence, the remaining two angles must be acute.

Example 1.9
How many triangles exist such that one angle is 115°, and the second angle is 89°?

Consider a triangle with two of the angles given above:


Δ𝐴𝐵𝐶: ∠𝐴 = 115°, ∠𝐵 = 89° ⇒ ∠𝐴 + ∠𝐵 = 115 + 89 = 204° > 180° ⇒ 𝑵𝒐𝒕 𝑽𝒂𝒍𝒊𝒅

C. Ratios

Example 1.10
The angles in a triangle are in the ratio 1:2:3. Find the angles.

180 180
= = 30
1+2+3 6
Angles are
30 × 1 = 30, 30 × 2 = 60, 30 × 3 = 90

Example 1.11
The measures of the interior angles of a particular triangle are in a 5:6:7 ratio. What is the measure, in degrees,

P a g e 6 | 152
Get all the files at: https://bit.ly/azizhandouts
Aziz Manva ([email protected])

of the smallest interior angle? (MathCounts 2004 School Sprint)

180
5: 6: 7 → 5 + 6 + 7 = 18 → = 10 → 10 × 5 = 50
18

Example 1.12
If the degree measures of the angles of a triangle are in the ratio 3: 3: 4, what is the degree measure of the
largest angle of the triangle? (AMC 8 2017/6)

180
3: 3: 4 → 3 + 3 + 4 = 10 → = 18 → 18 × 4 = 72
10

Example 1.13
The angle measures of the three angles of a triangle are in the ratio 1:3:6. What is the number of degrees in the
measure of the largest angle? (MathCounts 2002 Chapter Countdown)

1+3+6=10
180/10*18 = 108

Example 1.14
The measures of the angles of a triangle are in a ratio of 3:5:7. What is the degree measure of the largest angle?
(MathCounts 2009 Chapter Countdown)
180
3: 5: 7 → 3 + 5 + 7 = 15 → = 12 → 12 × 7 = 84
15

Example 1.15
The measures of the angles of a triangle are in the ratio 5:6:7. What is the number of degrees in the largest of
these angles? (MathCounts 1992 Warm-Up 11)

5 + 6 + 7 = 18
180
𝐿𝑎𝑟𝑔𝑒𝑠𝑡 𝑎𝑛𝑔𝑙𝑒 = × 7 = 70
18

Example 1.16
6
The sum of two angles of a triangle is 5 of a right angle, and one of these two angles is 30∘ larger than the other.
What is the degree measure of the largest angle in the triangle? (AMC 10B 2011/7)
Let the angles be
𝑎, 𝑎 + 30
We know that
6
𝑎 + (𝑎 + 30) = × 90 ⇒ 2𝑎 + 30 = 108 ⇒ 𝑎 = 39 ⇒ 𝑎 + 30 = 69
5
Which means that the third angle in the triangle is:
180 − 108 = 72° ⇒ 𝐿𝑎𝑟𝑔𝑒𝑠𝑡 𝐴𝑛𝑔𝑙𝑒 = 72°
D. Percentage

Example 1.17
Find the measures of the angles in Δ𝐴𝐵𝐶 if, ∠𝐴 is 50% more than ∠𝐵, and ∠𝐶 is 50% less than ∠𝐵.
Let the measure of

P a g e 7 | 152
Get all the files at: https://bit.ly/azizhandouts
Aziz Manva ([email protected])

∠𝐵 = 𝑏 ⇒ ∠𝐴 = 1.5𝑏 ⇒ ∠𝐶 = 0.5𝑏
𝑏 + 1.5𝑏 + 0.5𝑏 = 180 ⇒ 3𝑏 = 180 ⇒ 𝑏 = 60

Example 1.18
Find the measure of ∠𝐵 in Δ𝐴𝐵𝐶 if, ∠𝐴 is 𝑑% more than ∠𝐵, and ∠𝐶 is 𝑑% less than ∠𝐵.

∠𝐵 = 𝑏
∠𝐴 is 𝑑% more than ∠𝐴
𝑑 𝑑𝑏
∠𝐴 = 𝑏 + ×𝑏 =𝑏+
100 100
∠𝐶 is 𝑑% less than ∠𝐶
𝑑 𝑑𝑏
∠𝐶 = 𝑏 − ×𝑏 =𝑏−
100 100

𝒅𝒃 𝒅𝒃

𝑏 +𝑏+ +𝑏− = 180 ⇒ 3𝑏 = 180 ⇒ ∠𝐵 = 𝑏 = 60
⏟ 𝟏𝟎𝟎 ⏟ 𝟏𝟎𝟎
∠𝐵
∠𝐴 ∠𝐶

Example 1.19
Find the measures of the angles in Δ𝐴𝐵𝐶 if, ∠𝐵 is 50% of ∠𝐴, and ∠𝐶 is 50% of ∠𝐵.

Let the measure of


∠𝐶 = 𝑐 ⇒ ∠𝐵 = 2𝑐 ⇒ ∠𝐴 = 4𝑐
180 360 720
𝑐 + 2𝑐 + 4𝑐 = 180 ⇒ 7𝑐 = 180 ⇒ ∠𝐶 = 𝑐 = ⇒ ∠𝐵 = 2𝑐 = ⇒ ∠𝐴 = 4𝑐 =
7 7 7
E. Linear Equations

Example 1.20
What, in degrees, is the measure of the largest angle in Δ𝑃𝑄𝑅? (CEMC 2010
Cayley)

3𝑥 + 6𝑥 + 𝑥 = 180 ⇒ 𝑥 = 18 ⇒ 6𝑥 = 108

Example 1.21
In triangle 𝐴𝐵𝐶, the measure of angle 𝐴 is 𝑥 degrees, the measure of angle 𝐵 is 2𝑥 degrees and the measure of
angle 𝐶 is 5𝑥 degrees. What is the value of 𝑥? Express your answer as a decimal to the nearest tenth.
(MathCounts 2008 School Countdown)

𝑥 + 2𝑥 + 5𝑥 = 180 ⇒ 𝑥 = 22.5

Example 1.22
In triangle ABC, the measure of ∠𝐴 is 86 degrees. The measure of ∠𝐵 is 22 degrees more than three times the
measure of ∠𝐶. What is the measure, in degrees, of ∠𝐶? (MathCounts 2010 State Team)

86 + (3𝑐 + 22) + 𝑐 = 180 ⇒ 𝑐 = 18

Example 1.23
The angles in a triangle are 2𝑥 + 5, 𝑥 − 10, and 3𝑥 + 65. Find their ratio.

P a g e 8 | 152
Get all the files at: https://bit.ly/azizhandouts
Aziz Manva ([email protected])

(2𝑥 + 5) + (𝑥 − 10) + (3𝑥 + 65) = 180 ⇒ 6𝑥 + 60 = 180 ⇒ 𝑥 = 20 ⇒ 45: 10: 125 = 9: 2: 25

F. Simultaneous Linear Equations

Example 1.24
Find the measures of the three angles in a triangle if the difference in measure of two of the angles is five
degrees, and the sum of these same two angles is three more than twice the third angle.

Let
1𝑠𝑡 𝐴𝑛𝑔𝑙𝑒 = 𝑥 ⇒ 2𝑛𝑑 𝐴𝑛𝑔𝑙𝑒 = 𝑥 + 5
Let
3𝑟𝑑 𝐴𝑛𝑔𝑙𝑒 = 𝑦

𝑥 + (𝑥 + 5) = 2𝑦 + 3 ⇒ 𝑦 = 𝑥 + 1

And we also know that the sum of angles in a triangle is 180°:


2𝑥 + 5 + 𝑥 + 1 = 180 ⇒ 𝑥 = 58

1.3 Sum of Angles: Applications


A. Parallel Lines

Example 1.25
In triangle 𝑃𝑄𝑅, point 𝑇 is on 𝑃𝑅 and point 𝑆 is on 𝑃𝑄 such that 𝑇𝑆 ∥ 𝑅𝑄. The measure of ∠𝑅𝑃𝑄 is 65, and the
measure of ∠𝑇𝑆𝑄 is 145. What is the measure of ∠𝑃𝑅𝑄? (MathCounts 2009 Warm-up 9)

Draw a diagram.
By co-interior angles:
∠𝑆𝑄𝑅 = 180 − 145 = 35
By sum of angles in a triangle:
∠𝑃𝑅𝑄 = 180 − 65 − 35 = 80

Example 1.26
In the diagram, 𝑃𝑊 is parallel to 𝑄𝑋, 𝑆 and 𝑇 lie on 𝑄𝑋, and 𝑈 and 𝑉 are the
points of intersection of 𝑃𝑊 with 𝑆𝑅 and 𝑇𝑅, respectively. If ∠𝑆𝑈𝑉 = 120 and
∠ 𝑉𝑇𝑋 = 112, what is the measure of ∠𝑈𝑅𝑉? (CEMC 2010 Pascal)

By co-interior angles:
∠𝑅𝑆𝑇 = 180 − 120 = 60
By angles in a linear pair:
∠𝑅𝑇𝑆 = 180 − 112 = 68
By angles in a triangle:
∠𝑈𝑅𝑉 = ∠𝑆𝑅𝑉 = 180 − 60 − 68 = 52

Example 1.27
In the diagram, 𝑙 ∥ 𝑘. What is the number of degrees in ∠𝑆𝑅𝑄? (MathCounts 1997
School Sprint)

P a g e 9 | 152
Get all the files at: https://bit.ly/azizhandouts
Aziz Manva ([email protected])

∠𝑅𝑆𝑄 = 180 − 130 = 50


∠𝑅𝑄𝑆 = 90
∠𝑆𝑅𝑄 = 180 − 90 − 50 = 40

B. Bisectors
The bisectors of an angle of a triangle divide the angle equally.

Example 1.28

C. Trisectors

Example 1.29
The trisectors of angles B and C of scalene triangle ABC meet at points P and Q, as
shown. Angle A measures 39 degrees and angle QBP measures 14 degrees. What is the
measure of angle BPC? (MathCounts 2008 State Sprint)

∠𝐴𝐵𝐶 = 3 × 14 = 42
∠𝐴𝐶𝐵 = 180 − 42 − 39 = 99
99
∠𝑃𝐶𝐵 = = 33
3
∠𝐵𝑃𝐶 = 180 − 14 − 33 = 133

Example 1.30
In triangle ABC,∠𝐴 = 120. A point D is inside the triangle such ∠𝐷𝐵𝐶 = 2∠𝐴𝐵𝐷 and ∠𝐷𝐶𝐵 = 2∠𝐴𝐶𝐷. Find
∠𝐵𝐷𝐶.

∠𝐴𝐵𝐶 + ∠𝐴𝐶𝐵 = 180 − 120 = 60°


∠𝐴𝐵𝐶 + ∠𝐴𝐶𝐵 = 3𝑥 + 3𝑦

3𝑥 + 3𝑦 = 60 ⇒ 𝑥 + 𝑦 = 20 ⇒ ⏟
2𝑥 + 2𝑦 = 40
∠𝐷𝐵𝐶+∠𝐷𝐶𝐵

∠𝐵𝐷𝐶 = 180 − (∠𝐷𝐵𝐶 + ∠𝐷𝐶𝐵) = 180 − 40 = 140


D. Nested Triangles

Example 1.31
Point 𝐷 is on side 𝐴𝐶 of triangle 𝐴𝐵𝐶, ∠𝐴𝐵𝐷 = 15 and ∠𝐷𝐵𝐶 = 50. What is
the measure of angle 𝐵𝐴𝐷, in degrees? (MathCounts 2006 Chapter
Countdown)

∠𝐵𝐴𝐷 = 180 − 90 − 50 − 15 = 25

P a g e 10 | 152
Get all the files at: https://bit.ly/azizhandouts
Aziz Manva ([email protected])

Example 1.32
Find the number of degrees in the measure of angle 𝑥. (MathCounts 1995 School Sprint)

By sum of angles in Δ𝐷𝐵𝐶:


∠𝐷𝐶𝐵 = 180 − 108 − 23 = 49
By sum of angles in Δ𝐷𝐴𝐶:
𝑥 = ∠𝐷𝐴𝐵 = 180 − 26 − 23 − 49 = 82
E. Quadrilaterals

Example 1.33
Rectangle 𝑊𝑋𝑌𝑍 is drawn on Δ𝐴𝐵𝐶, such that point 𝑊
lies on segment 𝐴𝐵, point 𝑋 lies on segment 𝐴𝐶, and
points 𝑌 and Z lies on segment 𝐵𝐶, as shown. If
𝑚∠𝐵𝑊𝑍 = 26 and 𝑚∠𝐶𝑋𝑌 = 64, what is 𝑚∠𝐵𝐴𝐶, in
degrees? (MathCounts 2010 Chapter Countdown)

∠𝑊𝐵𝑍 = 90 − 26 = 64
∠𝑋𝐶𝑌 = 90 − 64 = 26
∠𝐵𝐴𝐶 = 180 − (64 + 26) = 180 − 90 = 90

1.4 Exterior Angles


A. Definition
If the side of a triangle is produced, the angle so formed is called an exterior angle.
Each angle of a triangle has two exterior angles.
In all, there are six exterior angles of a triangle – giving three pairs of
vertically opposite angles. Each pair has equal measure.

1.2: Two Angles exterior to same angle are congruent


The two exterior angles formed at a vertex of a triangle are congruent.

In the diagram, because of vertically opposite angles, we must have:


𝑥=𝑦

1.3: Exterior Angle is supplementary to adjacent interior angle


The exterior angle of a triangle is supplementary to its adjacent interior angle.

The exterior angle forms a linear pair with the adjacent interior angle
Hence, the two angles are supplementary.

P a g e 11 | 152
Get all the files at: https://bit.ly/azizhandouts
Aziz Manva ([email protected])

Example 1.34
In the diagram, which is not drawn to scale:
A. if ∠𝑥 = 74, find ∠𝑎.
B. if ∠𝑎 = 137, find ∠𝑥.
C. if ∠𝑎 = 90, what kind of triangle is it?
D. If ∠𝑎 is acute, what kind of triangle is it?
E. If ∠𝑎 is obtuse, can we say anything about what kind of triangle it is?

∠𝑥 and ∠𝑎 form a linear pair


Part A
𝑥 + 𝑎 = 180 ⇒ 74 + 𝑎 = 180 ⇒ 𝑎 = 106
Part B
𝑥 + 𝑎 = 180 ⇒ 𝑥 + 137 = 180 ⇒ 𝑥 = 43
Part C
Right-Angled Triangle
Part D
Obtuse-Angled Triangle
Part E
We can’t conclude that it is an acute angled triangle because one of the other interior angles might be right-
angled, or obtuse.

Example 1.35
A triangle has an angle with measure 63. What is the measure of the exterior angle to that angle?

180– 63 = 117

Example 1.36
A triangle has exterior angle twice its interior angle for each of its angles. What kind of triangle is it?

Method I
𝐼𝑛𝑡𝑒𝑟𝑖𝑜𝑟 𝐴𝑛𝑔𝑙𝑒 = 𝑥 ⇒ 𝐸𝑥𝑡𝑒𝑟𝑖𝑜𝑟 𝐴𝑛𝑔𝑙𝑒 = 2𝑥
𝑥 + 2𝑥 = 180 ⇒ 3𝑥 = 180 ⇒ 𝑥 = 60
This is true for each angle in the triangle, and hence the triangle is equilateral.

Method II
𝐼𝑛𝑡𝑒𝑟𝑖𝑜𝑟 𝐴𝑛𝑔𝑙𝑒 = 𝑥 ⇒ 𝐸𝑥𝑡𝑒𝑟𝑖𝑜𝑟 𝐴𝑛𝑔𝑙𝑒 = 180 − 𝑥
180 − 𝑥 = 2𝑥 ⇒ 3𝑥 = 180 ⇒ 𝑥 = 60
This is true for each angle in the triangle, and hence the triangle is equilateral.

P a g e 12 | 152
Get all the files at: https://bit.ly/azizhandouts
Aziz Manva ([email protected])

1.4: Exterior Angle is Sum of Two Interior Angles


Exterior angle of a triangle is the sum of two non-adjacent
interior angles.

Let the two non-adjacent interior angles be


𝑥 𝑎𝑛𝑑 𝑦
Since the sum of angles of a triange is 180°, the third angle must
be:
180 − 𝑥 − 𝑦
Since the exterior angle forms a linear pair with the adjacent
interior angle, we must have:
𝐸𝑥𝑡𝑒𝑟𝑖𝑜𝑟 𝐴𝑛𝑔𝑙𝑒 = 180 − (180 − 𝑥 − 𝑦) = 𝑥 + 𝑦

Example 1.37
The measure of an exterior angle of a triangle is 75 degrees. If one of the non-adjacent interior angles measures
28 degrees, what is the number of degrees in the other non-adjacent interior angle? (MathCounts 2011 State
Countdown)

𝑥 + 28 = 75 ⇒ 𝑥 = 75 − 28 = 47

Example 1.38
Triangle 𝐴𝐵𝐶 is a right triangle. If the measure of angle 𝑃𝐴𝐵 is 𝑥 and
the measure of angle 𝐴𝐶𝐵 is expressed in the form 𝑀𝑥 + 𝑁 with 𝑀 =
1, what is the value of 𝑀 + 𝑁? (MathCounts 2005 State Countdown)

By exterior angle property:


∠𝑃𝐴𝐵 = ∠𝐴𝐵𝐶 + ∠𝐴𝐶𝐵
Substitute ∠𝑃𝐴𝐵 = 𝑥, ∠𝐴𝐵𝐶 = 90:
𝑥 = 90 + ∠𝐴𝐶𝐵 ⇒ ∠𝐴𝐶𝐵 = 𝑥 − 90 ⇒ 𝑀 = 1, 𝑁 = −90 ⇒ 𝑀 + 𝑁 = 1 − 90 = −89

Example 1.39
In the diagram, which is not drawn to scale, if
A. ∠𝑥 = 65, ∠𝑦 = 75, ∠𝑧 = 20, then find ∠𝑎.
B. ∠𝑥 = 55, ∠𝑦 = 80, ∠𝑧 = 10, then find ∠𝑎.
C. ∠𝑦 = 2.5∠𝑥, ∠𝑧 = 0.5∠𝑥 𝑎𝑛𝑑 ∠𝑎 = ∠𝑥 + 90, then find ∠𝑥.

Part A
∠𝑝 = ∠𝑥 + ∠𝑦 = 65 + 75 = 140
∠𝑎 = ∠𝑝 + ∠𝑧 = 140 + 20 = 160

Part B
∠𝑝 = ∠𝑥 + ∠𝑦 = 55 + 80 = 135
∠𝑎 = 𝑝 + ∠𝑧 = 135 + 10 = 145

P a g e 13 | 152
Get all the files at: https://bit.ly/azizhandouts
Aziz Manva ([email protected])

Part C
∠𝑝 = ∠𝑥 + ∠𝑦 = ∠𝑥 + 2.5∠𝑥 = 3.5∠𝑥
∠𝑎 = ∠𝑝 + ∠𝑧 = 3.5∠𝑥 + 0.5∠𝑥 = 4∠𝑥

4∠𝑥 = ∠𝑥 + 90 ⇒ ∠𝑥 = 30

Example 1.40
In the diagram, 𝑄𝑅𝑆 is a straight line. What is the measure of ∠𝑅𝑃𝑆, in
degrees? (CEMC 2009 Pascal)

By the exterior angle property:


∠𝑃𝑅𝑆 = 48 + 67 = 115
By the sum of angles in a triangle property:
∠𝑅𝑃𝑆 = 180 − 115 − 38 = 27

Example 1.41
If ∠𝐴 = 60∘ , ∠𝐸 = 40∘ and ∠𝐶 = 30∘ , then ∠𝐵𝐷𝐶 =(AMC 8 1994/7)

By the exterior angle property:


∠𝐵 = ∠𝐴 + ∠𝐸 = 60 + 40 = 100

By the sum of angles in a triangle:


∠𝐵𝐷𝐶 = 180 − ∠𝐶 − ∠𝐵 = 180 − 30 − 100 = 50

1.5: Sum of Exterior Angles of A Triangle


Sum of exterior angles of a triangle is 𝟑𝟔𝟎°

Consider the triangle drawn, with interior angles


𝑥, 𝑦, 𝑧
Therefore, by angles in a straight line, the exterior angles
are:
180 − 𝑥, 180 − 𝑦, 180 − 𝑧
And they have sum
180 − 𝑥 + 180 − 𝑦 + 180 − 𝑧 = 540 − (𝑥 + 𝑦 + 𝑧)
Substitute 𝑥 + 𝑦 + 𝑧 = 180 (since these are the angles of a
triangle:
540 − 180 = 360°

Example 1.42
In the diagram alongside (which is not drawn to scale), find
A. 𝑎 + 𝑏 + 𝑐 + 𝑑 + 𝑒 + 𝑓
B. 𝑥 + 𝑦 + 𝑧

Part A

P a g e 14 | 152
Get all the files at: https://bit.ly/azizhandouts
Aziz Manva ([email protected])

Since the sum of exterior angles of a triangle is 360, we must have


𝑎 + 𝑑 + 𝑒 = 360
𝑏 + 𝑐 + 𝑓 = 360
Adding the two equations above:
𝑎 + 𝑏 + 𝑐 + 𝑑 + 𝑒 + 𝑓 = 360 + 360 = 720
Part B
Name angles 𝑥 ′ , 𝑦 ′ , 𝑧′ as in the diagram. Note that, by vertically opposite angles:
𝑥 = 𝑥′, 𝑦 = 𝑦′, 𝑧 = 𝑧′
Hence:
𝑥 + 𝑦 + 𝑧 = 𝑥 ′ + 𝑦 + 𝑧 ′ = 180
Where we get last equality since 𝑥 ′ , 𝑦 ′ , 𝑧′ are the angles of a triangle.

1.5 Five Pointed Stars


A. Five-Pointed Star

Example 1.43
The degree measure of angle 𝐴 is (AMC 8 1999/21)

Consider the angle with 110° as an exterior angle:


2𝑛𝑑 𝐼𝑛𝑡𝑒𝑟𝑖𝑜𝑟 𝐴𝑛𝑔𝑙𝑒 = 70
Angle vertically opposite
70
Angle forming linear pair with 100
= 180 − 100 = 80
By sum of angles in a triangle
∠𝐴 = 180 − 80 − 70 = 30

Example 1.44
If ∠𝐴 = 20∘ and ∠𝐴𝐹𝐺 = ∠𝐴𝐺𝐹, then ∠𝐵 + ∠𝐷 = (AMC 8 2000/24)

In Isosceles ΔAFG:
180 − 20 160
∠𝐴𝐹𝐺 = = = 80
2 2
By exterior angle property:
∠𝐵 + ∠𝐷 = ∠𝐴𝐹𝐺 = 80

Example 1.45
Show that the sum of the measures of the angles of the vertices of a five-pointed star is equal to the sum of the
measures of the angles of a triangle. (NMTC Primary/Final 2004/16)

Method I: Using Triangles


𝐼𝑛 Δ𝐴𝐶𝐹: ∠𝐴 + ∠𝐶 + ∠𝐶𝐹𝐴 = ∠180 ⏟
⇒ ∠𝐴 + ∠𝐶 = ∠180 − ∠𝐶𝐹𝐴
𝑬𝒒𝒖𝒂𝒕𝒊𝒐𝒏 𝑰
𝐼𝑛 ΔDFE: ∠𝐶𝐸𝐷 + ∠𝐸𝐷𝐴 + ∠𝐷𝐹𝐸 = 180 ⇒ ⏟
∠𝐶𝐸𝐷 + ∠𝐸𝐷𝐴 = 180 − ∠𝐷𝐹𝐸 = ⏟
180 − ∠𝐶𝐹𝐴 = ⏟
∠A + ∠C
𝑻𝒂𝒌𝒆 𝒕𝒐 𝒕𝒉𝒆 𝒐𝒕𝒉𝒆𝒓 𝒔𝒊𝒅𝒆 𝑽𝒆𝒓𝒕𝒊𝒄𝒂𝒍𝒍𝒚 𝑺𝒖𝒃𝒔𝒕𝒊𝒕𝒖𝒕𝒆 𝑰
𝑶𝒑𝒑𝒐𝒔𝒊𝒕𝒆 𝑨𝒏𝒈𝒍𝒆𝒔
𝐼𝑛 ΔBED: 180 = ∠𝐸𝐵𝐷 + ∠𝐵𝐷𝐸 + ∠𝐵𝐸𝐷 = ∠𝐵 + ∠𝐷 + ∠𝐸 + ∠𝐶𝐸𝐷 + ∠𝐸𝐷𝐴 = ∠𝐵 + ∠𝐷 + ∠𝐸 + ∠𝐴 + ∠𝐶

P a g e 15 | 152
Get all the files at: https://bit.ly/azizhandouts
Aziz Manva ([email protected])

Method I: Using Exterior Angle Property of Triangles:

Method II: Using Triangles and Pentagons


Pentagon
The points of intersection of the lines of the five-pointed star form a pentagon. Note that the angles of the
pentagon form three triangles, and hence the sum of the these angles
must be
180 × 3
Hence, we must have:
∠𝐴𝐽𝐶 + ∠𝐵𝐹𝐷 + ∠𝐶𝐺𝐸 + ∠𝐴𝐻𝐷 + ∠𝐵𝐽𝐸 = 180 × 3

Consider the five triangles below, and note that the sum of the angles of
each triangle must add up to 180°.

In Δ𝐴𝐶𝐽: ∠𝐴 + ∠𝐶 + ∠𝐴𝐽𝐶 = 180


In ΔBDF: ∠𝐵 + ∠𝐷 + ∠𝐵𝐹𝐷 = 180
In ΔCEG: ∠𝐶 + 𝐸 + ∠𝐶𝐺𝐸 = 180
In ΔADH: ∠𝐴 + ∠𝐷 + ∠𝐴𝐻𝐷 = 180
In ΔBEJ: ∠𝐵 + ∠𝐸 + ∠𝐵𝐽𝐸 = 180

Add the above five equations:


2(∠𝐴 + ∠𝐵 + ∠𝐶 + ∠𝐷 + ∠𝐸) + ∠𝐴𝐽𝐶 + ∠𝐵𝐹𝐷 + ∠𝐶𝐺𝐸 + ∠𝐴𝐻𝐷 + ∠𝐵𝐽𝐸 = 180 ∗ 5

Substitute, as calculated above that sum of angles of a pentagon is 180 × 3:

2(∠𝐴 + ∠𝐵 + ∠𝐶 + ∠𝐷 + ∠𝐸) + 180 × 3 = 180 × 5


2(∠𝐴 + ∠𝐵 + ∠𝐶 + ∠𝐷 + ∠𝐸) = 180 × 2
∠𝐴 + ∠𝐵 + ∠𝐶 + ∠𝐷 + ∠𝐸 = 180

P a g e 16 | 152
Get all the files at: https://bit.ly/azizhandouts
Aziz Manva ([email protected])

2. TYPES OF TRIANGLES
2.1 Equilateral Triangles
A. Definition
Triangle with three sides equal
OR
Triangle with three angles equal
Properties
➢ Angles of equilateral triangle measure 60 degrees.
➢ Converse is true: If angles of a triangle measure 60 degrees, the triangle is equilateral

Example 2.1
An equilateral triangle has sides 3, 2𝑥 + 3 and 3 sin 𝑦. What is 𝑥𝑦?

Since the triangle is equilateral, all three sides must be equal, and
hence, we must have:
2𝑥 + 3 = 3 ⇒ 𝑥 = 0 ⇒ 𝑥𝑦 = 0
Key Idea: In a difficult question, try working from the parts
that you do understand.

B. Angles

Example 2.2
In the diagram, if Δ𝐴𝐵𝐶 and Δ𝑃𝑄𝑅 are equilateral, then what is the
measure of ∠𝐶𝑋𝑌 in degrees? (CEMC 2007 Cayley)

This question has appeared in multiple exams. Solving it requires combining different properties, and some
level of creativity.

By angles in a linear pair:


∠𝑌𝐵𝑃 = 180 − 65 − 60 = 55
∠𝑌𝑃𝐵 = 180 − 75 − 60 = 45
By angles in a triangle:
∠𝑋𝑌𝐶 = ∠𝐵𝑌𝑃 = 180 − 55 − 45 = 80
By vertically opposite angles:
∠𝑋𝑌𝐶 = ∠𝐵𝑌𝑃 = 80
By angles in a triangle
∠𝐶𝑋𝑌 = 180 − 60 − 80 = 40

C. Exterior Angles

Example 2.3
What is the measure of the exterior angle of an equilateral triangle?
180 − 60 = 120

P a g e 17 | 152
Get all the files at: https://bit.ly/azizhandouts
Aziz Manva ([email protected])

D. Ratios

Example 2.4
In an equilateral triangle, the angles have measures of 3p + 10, 4𝑞 − 10 and 5𝑟. An exterior angle of the triangle
has measure 8𝑠. What is 𝑝: 𝑞: 𝑟: 𝑠?

The interior angles are equal to 60.


3p + 10 = 60 ⇒ p = 50/3
4q – 10 = 60 ⇒ q = 70/4 = 35/2
5r = 60 ⇒ r = 12
Exterior angle is equal to 120
8s = 120 ⇒ s = 3/2
50 35 3
𝑝: 𝑞: 𝑟: 𝑠 = : : 12: = 100: 105: 72: 9
3 2 2

E. Perimeter, Sides and Area

Example 2.5
If an equilateral triangle has perimeter six units, what is its side length, height and area?

6
𝑆𝑖𝑑𝑒 𝐿𝑒𝑛𝑔𝑡ℎ = =2
3
√3 √3
𝐻𝑒𝑖𝑔ℎ𝑡 = 𝑠= × 2 = √3
2 2
√3 2 √3
𝐴𝑟𝑒𝑎 = 𝑠 = × 22 = √3
4 4

From the above, we see that the numerical value of the height of an equilateral triangle is equal to its area when
the side length is 2. Show this algebraically.

√3 √3 2
𝑠=𝐴⇒ 𝑠= 𝑠 ⇒ 2𝑠 = 𝑠 2 ⇒ 𝑠 = 2
2 4

Example 2.6
What is the perimeter of an equilateral triangle with height 12?

√3 2 2
ℎ= 𝑠⇒𝑠= ℎ= × 12 = 8√3 ⇒ 𝑝 = 3𝑠 = 24√3
2 √3 √3

F. Area of Regular Hexagon


A regular hexagon consists of six equilateral triangles.
Hence, the area of a regular hexagon is six times the area of an equilateral triangle
√3 2
𝐴𝑟𝑒𝑎 = 6 × 𝑠
4

Example 2.7
What is the area of a regular hexagon with side length 3 units?

P a g e 18 | 152
Get all the files at: https://bit.ly/azizhandouts
Aziz Manva ([email protected])

√3 2 √3
𝐴𝑟𝑒𝑎 = 6 × 𝑠 =6× × 32 = 9√3
4 4
2.2 Right Angled Triangles
A. Definition
A triangle with a right angle is called right-angled triangle.

Example 2.8
How many triangles exist with two right angles?

Zero

Example 2.9
Find the number of right triangles with angles of integral measure, no two of which are equal.

We can tabulate all the possibilities like this:


First Angle 1° 2° . . . 44°
Second Angle 89° 88° . . . 46°

And hence, the number of triangles is:


44
B. Ratios

Example 2.10
The acute angles in a right-angled triangle are in the ratio 1:3. Find the angles

90 90
= = 22.5
1+3 4
Angles are 22.5 * 1 = 22.5, 22.5 * 3 = 67.5

Example 2.11
The ratio of the measures of the acute angles of a right triangle is 8:1. In degrees, what is the measure of the
largest angle of the triangle? (MathCounts 2010 School Countdown)

90

C. Prime Numbers

Example 2.12
The acute angles of a right triangle are 𝑎∘ and 𝑏 ∘ , where 𝑎 > 𝑏 and both 𝑎 and 𝑏 are prime numbers. What is the
least possible value of 𝑏? (AMC 10B 2020/4)

In a right triangle, the acute angles must add up to:


180 − 90 = 90°

Hence, we want:
𝑎 + 𝑏 = 90, 𝑎, 𝑏 𝑏𝑜𝑡ℎ 𝑝𝑟𝑖𝑚𝑒

P a g e 19 | 152
Get all the files at: https://bit.ly/azizhandouts
Aziz Manva ([email protected])

Now, try values of 𝑎, 𝑏, starting with the smallest. We need only consider prime values for 𝑏:
𝑏 = 2 ⇒ 𝑎 = 88 ⇒ 𝑁𝑜𝑡 𝑝𝑟𝑖𝑚𝑒
𝑏 = 3 ⇒ 𝑎 = 87 ⇒ 𝑁𝑜𝑡 𝑝𝑟𝑖𝑚𝑒
𝑏 = 5 ⇒ 𝑎 = 85 ⇒ 𝑁𝑜𝑡 𝑝𝑟𝑖𝑚𝑒
𝑏 = 7 ⇒ 𝑎 = 83 ⇒ 𝑃𝑟𝑖𝑚𝑒 ⇒ 𝑆𝑚𝑎𝑙𝑙𝑒𝑠𝑡

Property 2.13: Angles of an Isosceles Right-Angled Triangle


The angles in an isosceles right-angled triangle are
𝟒𝟓, 𝟒𝟓, 𝒂𝒏𝒅 𝟗𝟎

Since the triangle is right-angled, it must have one angle


90°
If one angle of the triangle is 90°, then sum of the remaining angles must be:
180°
⏟ − ⏟
90° = 90°
𝑆𝑢𝑚 𝑜𝑓 𝐴𝑛𝑔𝑙𝑒𝑠 𝑅𝑖𝑔ℎ𝑡 𝐴𝑛𝑔𝑙𝑒
𝑜𝑓 𝑎 𝑇𝑟𝑖𝑎𝑛𝑔𝑙𝑒

But, since it is an isosceles triangle, its base angles must be equal. Hence, the value of each base angle must be:
𝑏 + 𝑏 = 90 ⇒ 2𝑏 = 90 ⇒ 𝑏 = 45°

Example 2.14
What kind of triangle is formed when, in an isosceles triangle, the sum of the angles opposite the equal sides is
equal to the third angle?

Draw a diagram, and use the sum of angles property:


𝑥 + 𝑥 + 2𝑥 = 180 ⇒ 𝑥 = 45
And, hence the triangle is:
45 − 45 − 90 (Isosceles Right − Angled Triangle)

D. Exterior Angles
We have seen the property of exterior angles earlier.
➢ The exterior angle of a triangle forms a linear pair with its adjacent
interior angle.
➢ The exterior angle of a triangle is the sum of the two non-adjacent interior angles.

Example 2.15
Consider a right angled triangle with measure 𝑎 for an angle not opposite its hypotenuse. Find, in terms of 𝑎,
the measure of the exterior angles of that triangle.

One angle is 90°. Hence, the angle exterior to it is


180 − 90 = 90
We know one angle which is not the right angle is 𝑎°.
Hence, the angle exterior to it is:
(180 − 𝑎)°
And the third exterior angle is the sum of the two non-
adjacent interior angles, which is
(90 + 𝑎)°

P a g e 20 | 152
Get all the files at: https://bit.ly/azizhandouts
Aziz Manva ([email protected])

Example 2.16
Δ𝐴𝐵𝐶 is equilateral with side length six units, and Δ𝑋𝑌𝑍 is right-angled with hypotenuse 5 units. What is the
positive difference between the sum of the six exterior angles of △ABC and the sum of the six exterior angles of
△XYZ?

If the angles of any triangle are 𝑎, 𝑏, and 𝑐, then the exterior angles are:
= 𝑎 + 𝑏, 𝑏 + 𝑐, 𝑎𝑛𝑑 𝑐 + 𝑎
Adding the three:
2(𝑎 + 𝑏 + 𝑐) = 2(180) = 360
This is true for any triangle. Hence, difference is zero.

2.3 Acute, Obtuse and Scalene Triangles


A. Acute
A triangle with all angles acute is called an acute-angled triangle.

Example 2.17
If all the angles of a triangle are acute and equal, then find the measure of each angle in the triangle.

3𝑥 = 180 ⇒ 𝑥 = 60
B. Obtuse
A triangle with one obtuse angle is called an obtuse triangle.

Example 2.18
How many triangles with two obtuse angles exist?

𝑎 + 𝑏 + 𝑐 = 180 ⇒ 𝑐 = 180 − (𝑎 + 𝑏)
Since a and b are both obtuse, we must have
𝑎 > 90, 𝑏 > 90 ⇒ 𝑎 + 𝑏 > 180
Hence,
𝑐 = 180 − (𝑎 + 𝑏) ⇒ 𝑵𝒆𝒈𝒂𝒕𝒊𝒗𝒆 ⇒ 𝑵𝒐𝒕 𝑷𝒐𝒔𝒔𝒊𝒃𝒆

Hence, the number of such triangles is zero.


C. Scalene

P a g e 21 | 152
Get all the files at: https://bit.ly/azizhandouts
Aziz Manva ([email protected])

3. ISOSCELES TRIANGLES
3.1 Angles Basics
A. Definition

3.1: Isosceles Triangles


A triangle with at least two angles equal is called an isosceles triangle.
The equal angles are called the base angles, and the third angle is called the vertex angle.
The side between the two base angles is called the base, and the point opposite the
base is called the vertex.

Example 3.2
In the diagram alongside ∠𝐵 = ∠𝐶, making Δ𝐴𝐵𝐶 an Isosceles Triangle. Name the:
A. Base Angles
B. Vertex Angle

𝐵𝑎𝑠𝑒 𝐴𝑛𝑔𝑙𝑒𝑠: ∠𝐵, ∠𝐶


𝑉𝑒𝑟𝑡𝑒𝑥 𝐴𝑛𝑔𝑙𝑒: ∠𝐴
B. Geometrical Problems
3.3: Base Angle Property
Base Angles of an Isosceles Triangle are equal.

Example 3.4
In the figure alongside, Δ𝐴𝐵𝐶 is isosceles with base angles ∠𝐵 and ∠𝐶 and vertex ∠𝐴. Find:
A. ∠𝐴 if ∠𝐵 and ∠𝐶 are each equal to 40°.
B. the base angles if angle ∠𝐴 = 40°.
C. the remaining angles if base ∠𝐵 = 50°.
D. ∠𝐴 if ∠𝐵 = 3𝑥 − 5 and ∠𝐶 = 2𝑥 + 5.

Part A
180 − 40 − 40 = 180 − 80 = 100°
Part B
180 − 40 140
= = 70°
2 2
Part C

180 − 50 − 50 = 180 − 100 = 80°


Part D
∠𝐵 = ∠𝐶 ⇒ 3𝑥 − 5 = 2𝑥 + 5 ⇒ 𝑥 = 10 ⇒ ∠𝐵 = ∠𝐶 = 25
∠𝐴 = 180 − 25 − 25 = 180 − 50 = 130

C. Word Problems

Example 3.5
In an isosceles triangle, the vertex angle is 35°. What are the other angles?

P a g e 22 | 152
Get all the files at: https://bit.ly/azizhandouts
Aziz Manva ([email protected])

180 – 35 145
Each angle = = = 72.5°
2 2

Example 3.6
In an isosceles triangle, the vertex angle is twice a base angle. Find the angles of the triangle.
𝐿𝑒𝑡 𝐵𝑎𝑠𝑒 𝐴𝑛𝑔𝑙𝑒 = 𝑏 ⇒ 𝑉𝑒𝑟𝑡𝑒𝑥 𝐴𝑛𝑔𝑙𝑒 = 2𝑏
And now since the sum of angles of a triangle is 180°, we must have:
𝑏 + 𝑏 + 2𝑏 = 180 ⇒ 4𝑏 = 180 ⇒ 𝑏 = 45°
And hence the angles must be:
45°, 45°, 90°

Example 3.7
An isosceles, obtuse triangle has one angle with a degree measure that is 50% larger than the measure of a right
angle. What is the measure, in degrees, of one of the two smallest angles in the triangle? Express your answer as
a decimal to the nearest tenth. (MathCounts 2008 Chapter Countdown)

𝐿𝑎𝑟𝑔𝑒𝑟 𝐴𝑛𝑔𝑙𝑒 = 90 × 1.5 = 135


180 − 135 45
𝑆𝑚𝑎𝑙𝑙𝑒𝑟 𝐴𝑛𝑔𝑙𝑒 = = = 22.5
2 2

Example 3.8
One of the base angles of an isosceles triangle is 35. What is the difference between the other two angles?

𝑉𝑒𝑟𝑡𝑒𝑥 𝐴𝑛𝑔𝑙𝑒 = 180 − 35 × 2 = 110


𝐷𝑖𝑓𝑓𝑒𝑟𝑒𝑛𝑐𝑒 = 110 − 35 = 75

D. Multiple Values

Example 3.9
One angle of an isosceles triangle is 45. What is the sum of the possible values of the vertex angle?

Case I: 45 is a base angle


𝑉𝑒𝑟𝑡𝑒𝑥 𝐴𝑛𝑔𝑙𝑒 = 180 − 45 × 2 = 180 − 90 = 90°
Case II: 45 is a vertex angle
𝑉𝑒𝑟𝑡𝑒𝑥 𝐴𝑛𝑔𝑙𝑒 = 45
Sum of the possible values
90 + 45 = 135

Example 3.10
Triangle 𝑃𝑄𝑅 is isosceles and the measure of angle 𝑅 is 40. The possible measures of angle 𝑃 are 𝑥, 𝑦, 𝑧. What is
the value of the sum 𝑥 + 𝑦 + 𝑧? (MathCounts 2003 National Countdown)

Case I: ∠𝑷 is a base angle, ∠𝑹 is also a base angle


∠𝑃 = ∠𝑅 = 40
Case II: ∠𝑷 is a base angle, ∠𝑹 is the vertex
180 − 40 140
∠𝑃 = = = 70
2 2

P a g e 23 | 152
Get all the files at: https://bit.ly/azizhandouts
Aziz Manva ([email protected])

Case III: ∠𝑷 is the vertex angle


∠𝑃 = 180 − 40 − 40 = 100

𝑥 + 𝑦 + 𝑧 = 40 + 70 + 100 = 210

Example 3.11
Two angles of an isosceles triangle measure 70∘ and 𝑥 ∘ . What is the sum of the three possible values of 𝑥? (AMC
8 2009/19)

We consider three cases.


Case I: Base Angles are 𝟕𝟎° and 𝟕𝟎°
Then, the vertex angle must be
𝑥 = 180 − 70 − 70 = 40°

Case II: Base Angles are 𝒙° and 𝒙°


Then, the vertex angle must be 70°
And hence we must have
180 − 70 110
𝑥= = = 55°
2 2
Case II: Base Angles are 𝟕𝟎° and 𝒙° = 𝟕𝟎°
In this case, we directly have
𝑥 = 70°
And hence, the final answer is the sum of all possible values, which is:
40 + 55 + 70 = 165°

Example 3.12
Two isosceles triangles each have at least one angle that measures 70. In the first triangle, the measure in
degrees of each of the remaining two angles is even. In the second triangle, the measure in degrees of each of
the remaining two angles is odd. In the first triangle, the sum of the equal angles is 𝑆. In the second triangle, the
sum of the equal angles is 𝑇. The value of 𝑆 + 𝑇 is (Gauss 7 2020/18)

Two cases:
𝑂𝑛𝑒 𝐵𝑎𝑠𝑒 𝐴𝑛𝑔𝑙𝑒 = 70° ⇒ 𝑂𝑡ℎ𝑒𝑟 𝐵𝑎𝑠𝑒 𝐴𝑛𝑔𝑙𝑒 = 70° ⇒ 𝑉𝑒𝑟𝑡𝑒𝑥 𝐴𝑛𝑔𝑙𝑒 = 40°
From the above, we know that we found the first triangle
𝑆 = 70 + 70 = 140

180 − 70 110
𝑉𝑒𝑟𝑡𝑒𝑥 𝐴𝑛𝑔𝑙𝑒 = 70° ⇒ 𝐸𝑎𝑐ℎ 𝑏𝑎𝑠𝑒 𝐴𝑛𝑔𝑙𝑒 = = = 55
2 2
From the above, we know that we have found the second triangle.
𝑇 = 55 + 55 = 110°

And we want to find:


𝑆 + 𝑇 = 140 + 110 = 250°

E. Percentages

Example 3.13
In an isosceles triangle, the vertex angle is 50% more than a base angle. Find the angles of the triangle.

P a g e 24 | 152
Get all the files at: https://bit.ly/azizhandouts
Aziz Manva ([email protected])

50 1
𝐿𝑒𝑡 𝐵𝑎𝑠𝑒 𝐴𝑛𝑔𝑙𝑒 = 𝑏 ⇒ 𝑉𝑒𝑟𝑡𝑒𝑥 𝐴𝑛𝑔𝑙𝑒 = 𝑏 + × 𝑏 = 𝑏 + 𝑏 = 1.5𝑏
100 2

And since the sum of angles of a triangle is 180°, we must have:

7 360 3 540
𝑏 + 𝑏 + 1.5𝑏 = 180 ⇒ 3.5𝑏 = 180 ⇒ 𝑏 = 180 ⇒ 𝑏 = ⇒ 𝑏=
2 7 2 7

Example 3.14
In an isosceles triangle, the vertex angle is 50% less than a base angle. Find the angles of the triangle.

50 1
𝐿𝑒𝑡 𝐵𝑎𝑠𝑒 𝐴𝑛𝑔𝑙𝑒 = 𝑏 ⇒ 𝑉𝑒𝑟𝑡𝑒𝑥 𝐴𝑛𝑔𝑙𝑒 = 𝑏 − × 𝑏 = 𝑏 − 𝑏 = 0.5𝑏
100 2

And since the sum of angles of a triangle is 180°, we must have:


5
𝑏 + 𝑏 + 0.5𝑏 = 180 ⇒ 2.5𝑏 = 180 ⇒ 𝑏 = 180 ⇒ 𝑏 = 72 ⇒ 0.5𝑏 = 36
2
F. Ratios

Example 3.15
The vertex angle and the base angle of a triangle are in the ratio 3: 2. Find the possible values of the vertex
angle.

Case I: Vertex Angle is Smaller


180
2: 3: 3 → = 22.5 → 22.5 × 2 = 45
8
Case I: Vertex Angle is Larger
180 180 540
3: 2: 2 → → ×3=
7 7 7
G. Algebra

Example 3.16
One angle in an isosceles triangle is double another angle. Find the sum of the possible values of the vertex
angle.

Example 3.17
Triangle ABC is isosceles with angle A congruent to angle B. The measure of angle C is 30 degrees more than the
measure of angle A. What is the number of degrees in the measure of angle C? (MathCounts 2004 Chapter
Countdown)

P a g e 25 | 152
Get all the files at: https://bit.ly/azizhandouts
Aziz Manva ([email protected])

Let
∠𝐴 = ∠𝐵 = 𝑥 ⇒ ∠𝐶 = 𝑥 + 30
𝑥 + 𝑥 + 𝑥 + 30 = 180 ⇒ 3𝑥 = 150 ⇒ 𝑥 = 50 ⇒ 𝑥 + 30 = 80

3.2 Angles: Applications


A. Vertically Opposite Angles

Example 3.18
In the diagram, Δ𝑃𝑄𝑅 is isosceles. What is the value of 𝑥? (CEMC 2008 Gauss 7)

180 − 140 140


𝑥= = = 70°
2 2
B. Exterior Angles

Example 3.19
In triangle 𝐴𝐵𝐶, 𝐴𝐶 = 𝐵𝐶, and 𝑚∠𝐵𝐴𝐶 = 40. What is the number of degrees in angle
𝑥? (MathCounts 1998 School Countdown)

𝑥 = 180 − ∠𝐶𝐵𝐴 = 180 − ∠𝐵𝐴𝐶 = 180 − 40 = 140

Example 3.20
If the exterior angle of the vertex angle of an isosceles triangle has measure 40, then what are the measures of
the remaining angles of the triangle?

180 − 40 140
𝐸𝑎𝑐ℎ 𝑎𝑛𝑔𝑙𝑒 = = = 70
2 2

Example 3.21
In △ 𝐴𝐵𝐶, 𝐷 is a point on side 𝐴𝐶 such that 𝐵𝐷 = 𝐷𝐶 and
∠𝐵𝐶𝐷 measures 70∘ . What is the degree measure of ∠𝐴𝐷𝐵?
(AMC 8 2014/9)

∠𝐴𝐷𝐵 = 70 + 70 = 140

Example 3.22
Triangles 𝐵𝐷𝐶 and 𝐴𝐶𝐷 are coplanar and isosceles. If we have 𝑚∠𝐴𝐵𝐶 = 70, what is
𝑚∠𝐵𝐴𝐶, in degrees? (MathCounts 2010 School Countdown)

∠𝐵𝐷𝐶 = 70
70
∠𝐵𝐴𝐶 = = 35
2

P a g e 26 | 152
Get all the files at: https://bit.ly/azizhandouts
Aziz Manva ([email protected])

C. Parallel Lines

Example 3.23
̅̅̅̅ is parallel to the segment through 𝐴, and 𝐴𝐵 = 𝐵𝐶. What is the
𝐵𝐶
number of degrees represented by 𝑥? (MathCounts 2000 State
Countdown)

By alternate interior angles


∠𝐴𝐵𝐶 = 124
In Isosceles Δ𝐴𝐵𝐶:
180 − 124 56
∠𝐵𝐴𝐶 = = = 28
2 2
By angles on a straight line:
𝑥 = 180 − 124 − 28 = 28

Example 3.24
In Δ𝐴𝐵𝐶, 𝐴𝐶 = 𝐵𝐶, 𝑚∠𝐷𝐶𝐵 = 40, and 𝐶𝐷 ∥ 𝐴𝐵. What is the number of degrees
in 𝑚∠𝐸𝐶𝐷? (MathCounts 1999 School Sprint)

∠𝐶𝐵𝐴 = ∠𝐷𝐶𝐵 = 40
∠𝐶𝐴𝐵 = ∠𝐶𝐵𝐴 = 40
∠𝐸𝐶𝐷 = ∠𝐶𝐴𝐵 = 40

D. Two Triangles

Example 3.25
∠1 + ∠2 = 180∘.
∠3 = ∠4
Find ∠4. (AMC 8 1997/12)

∠2 = 40 + 70 = 110
180 − 110 70
∠4 = = = 35
2 2

Example 3.26
In the diagram, 𝑃𝑅𝑇 and 𝑄𝑅𝑆 are straight lines. What is the value of 𝑥? (CEMC 2008
Cayley)

In Isosceles Δ𝑃𝑄𝑅:
180 − 40 140
∠𝑃𝑅𝑄 = = = 70
2 2
By Vertically Opposite Angles
∠𝑆𝑅𝑇 = ∠𝑃𝑅𝑄 = 70

P a g e 27 | 152
Get all the files at: https://bit.ly/azizhandouts
Aziz Manva ([email protected])

In Isosceles Δ𝑅𝑆𝑇:
180 − 70 110
𝑥 = ∠𝑅𝑇𝑆 = = = 55
2 2

Example 3.27
In the diagram, point 𝐸 lies on line segment 𝐴𝐵, and triangles 𝐴𝐸𝐷 and 𝐵𝐸𝐶 are
isosceles. Also, ∠𝐷𝐸𝐶 is twice ∠𝐴𝐷𝐸. What is the measure of ∠𝐸𝐵𝐶 in degrees?
(CEMC 2006 Pascal)

∠𝐷𝐸𝐴 = 70
∠𝐷𝐸𝐶 = 2 × ∠𝐴𝐷𝐸 = 2 × (180 − 70 × 2) = 2 × 40 = 80
∠𝐶𝐸𝐵 = 180 − 70 − 80 = 30
180 − 30 150
∠𝐸𝐵𝐶 = = = 75
2 2

E. Sequence of Triangles

Example 3.28
In isosceles triangle 𝐴𝐵𝐶, if 𝐵𝐶 is extended to a point 𝑋 such
that 𝐴𝐶 = 𝐶𝑋, what is the number of degrees in the measure of
angle 𝐴𝑋𝐶? (MathCounts 1993 Chapter Countdown)

By angles in a linear pair


𝐸𝑥𝑡𝑒𝑟𝑖𝑜𝑟 𝐴𝑛𝑔𝑙𝑒 ∠𝐴𝐶𝑋 = 180 − ∠𝐴𝐶𝐵 = 180 − ∠𝐴𝐵𝐶 = 180 − 30 = 150
In Isosceles Δ𝐴𝐶𝑋:
180 − 150 30
∠𝐴𝑋𝐶 = = = 15
2 2

Example 3.29
In the figure, 𝐵𝐴 = 𝐴𝐷 = 𝐷𝐶 and point 𝐷 is on segment 𝐵𝐶. The measure
of angle 𝐴𝐶𝐷 is 22.5 degrees. What is the measure of angle 𝐴𝐵𝐶?
(MathCounts 2008 Workout 2)

∠𝐴𝐵𝐶 = ∠𝐴𝐷𝐵 = 2 × 22.5 = 45


F. Many Triangles

Challenge 3.30
In triangle 𝐴𝐵𝐶, 𝐴𝑋 = 𝑋𝑌 = 𝑌𝐵 = 𝐵𝐶 and the measure of angle 𝐴𝐵𝐶 is 120
degrees. What is the number of degrees in the measure of angle 𝐵𝐴𝐶?
(MathCounts 1993 National Target)

In Isosceles Δ𝐴𝑋𝑌:
∠𝑋𝑌𝐴 = ∠𝐵𝐴𝐶 = 𝑎
In Isosceles ΔB𝑋𝑌, use the exterior angle property:
∠𝑋𝐵𝑌 = ∠𝐵𝑋𝑌 = 𝑎 + 𝑎 = 2𝑎
In ΔB𝑋𝑌 use the sum of angles property:

P a g e 28 | 152
Get all the files at: https://bit.ly/azizhandouts
Aziz Manva ([email protected])

∠𝑋𝑌𝐵 = 180 − 2𝑎 − 2𝑎 = 180 − 4𝑎


Use the angles in a straight-line property:
∠𝐵𝐶𝑌 = ∠𝐵𝑌𝐶 = 180 − (180 − 4𝑎) − 𝑎 = 3𝑎
Use the angles in a triangle property:
∠𝑌𝐵𝐶 = 180 − 3𝑎 − 3𝑎 = 180 − 6𝑎
Use the adjacent angles property:
∠𝐴𝐵𝐶 = ∠𝑋𝐵𝑌 + ∠𝑌𝐵𝐶 = 2𝑎 + 180 − 6𝑎 = 180 − 4𝑎
And now that we have an expression for ∠𝐴𝐵𝐶, equate it to its value:
180 − 4𝑎 = 120 ⇒ 𝑎 = 15

G. Right Angled Triangles

Example 3.31
In the diagram, 𝐴𝐷 = 𝐵𝐷 = 𝐶𝐷 and ∠𝐵𝐶𝐴 = 40. What is the measure of ∠𝐵𝐴𝐶?
(CEMC 2007 Gauss 8)

Let
∠𝐵𝐶𝐴 = 𝑥 ⇒ ∠𝐷𝐴𝐶 = 𝑥
By the exterior angle property
∠𝐵𝐷𝐴 = 2𝑥
In Isosceles Δ𝐴𝐵𝐷:
180 − 2𝑥
∠𝐵𝐴𝐷 = = 90 − 𝑥
2
And finally, using adjacent angles:
∠𝐵𝐴𝐶 = ∠𝐵𝐴𝐷 + ∠𝐷𝐴𝐶 = 𝑥 + 90 − 𝑥 = 90

Note that the value of ∠𝐵𝐴𝐶 was not needed.

If the median to a triangle is equal to the two segments formed at the base,
the triangle is right-angled.

Example 3.32
In Δ𝐴𝐵𝐶, what is the value of 𝑥 + 𝑦? (CEMC 2005 Pascal)

𝑥 + 𝑦 = 90

Example 3.33
In the diagram, 𝑅𝑆𝑃 is a straight line and ∠𝑄𝑆𝑃 = 80. What is the measure of
∠𝑃𝑄𝑅, in degrees? (CEMC 2008 Gauss 8)

Using the property above,


∠𝑃𝑅𝑄 = 90

Example 3.34
In the diagram alongside, if ∠𝐴𝐵𝐶 = 35, what is ∠𝐴𝐶𝐵?

∠𝐴𝐶𝐵 = 90 − 35 = 65

P a g e 29 | 152
Get all the files at: https://bit.ly/azizhandouts
Aziz Manva ([email protected])

H. Congruent Triangles

Example 3.35
Suppose that there are two congruent triangles Δ𝐴𝐵𝐶 and Δ𝐴𝐶𝐷 such that 𝐴𝐵 = 𝐴𝐶 =
𝐴𝐷, as shown in the following diagram. If ∠𝐵𝐴𝐶 = 20, then what is ∠𝐵𝐷𝐶? (AOPS
Alcumus, Geometry, Isosceles and Equilateral Triangles)

180 − 20 160
∠𝐴𝐶𝐵 = = = 80
2 2
Since Δ𝐴𝐵𝐶 and Δ𝐴𝐶𝐷 are congruent:
∠𝐴𝐶𝐷 = ∠𝐴𝐶𝐵 = 80

Then, in isosceles Δ𝐵𝐶𝐷:


𝑉𝑒𝑟𝑡𝑒𝑥 𝐴𝑛𝑔𝑙𝑒 = ∠𝐵𝐶𝐷 = ∠𝐴𝐶𝐵 + ∠𝐴𝐶𝐷 = 80 + 80 = 160
180 − 160 20
∠𝐵𝐷𝐶 = = = 10
2 2
I. Nested Triangles

Example 3.36
In the diagram, 𝑃𝑄 = 𝑃𝑅 and ∠𝑄𝑅𝑃 = 65. What is the value of 𝑥? (CEMC 2009 Cayley)

∠𝑆𝑄𝑅 = 180 − 90 − 65 = 25
𝑥 = 65 − 25 = 40

Example 3.37
In triangle 𝐶𝐴𝑇, we have ∠𝐴𝐶𝑇 = ∠𝐴𝑇𝐶 and ∠𝐶𝐴𝑇 = 36∘ . If 𝑇𝑅 bisects ∠𝐴𝑇𝐶, then
∠𝐶𝑅𝑇 =(AMC 8 2000/13)

180 − 36 144
∠𝐴𝐶𝑇 = ∠𝐴𝑇𝐶 = = = 72
2 2
72
∠𝑅𝑇𝐶 = = 36
2
∠𝐶𝑅𝑇 = 180 − 72 − 36 = 72

Example 3.38
Triangles 𝐴𝐵𝐶 and 𝐴𝐷𝐶 are isosceles with 𝐴𝐵 = 𝐵𝐶 and 𝐴𝐷 = 𝐷𝐶. Point 𝐷 is inside triangle 𝐴𝐵𝐶, angle 𝐴𝐵𝐶
measures 40 degrees, and angle 𝐴𝐷𝐶 measures 140 degrees. What is the degree measure of angle 𝐵𝐴𝐷? (AMC
10A 2007/8)

P a g e 30 | 152
Get all the files at: https://bit.ly/azizhandouts
Aziz Manva ([email protected])

Draw a diagram.
Apply the properties of isosceles triangles in Δ𝐴𝐵𝐶:
∠𝐴𝐵𝐶 = 40 ⇒ ∠𝐵𝐴𝐶 = ∠𝐵𝐶𝐴 = 70°
Apply the properties of isosceles triangles in Δ𝐴𝐷𝐶:
∠𝐴𝐶𝐶 = 140 ⇒ ∠𝐷𝐴𝐶 = ∠𝐷𝐶𝐴 = 20°
And now we can find the needed angle:
∠𝐵𝐴𝐷 = ∠𝐵𝐴𝐶 − ∠𝐷𝐴𝐶 = 50°

Example 3.39
In the diagram, 𝑃 is on 𝑅𝑆 so that 𝑄𝑃 bisects ∠𝑆𝑄𝑅. Also, 𝑃𝑄 = 𝑃𝑅, ∠𝑅𝑆𝑄 = 2𝑦, and
∠𝑅𝑃𝑄 = 3𝑦. What is the measure, in degrees, of ∠𝑅𝑃𝑄 (CEMC 2008 Cayley)

By exterior angle of a triangle:


∠𝑅𝑃𝑄 = ∠𝑅𝑆𝑄 + ∠𝑃𝑄𝑅 ⇒ 3𝑦 = 2𝑦 + 𝑥 ⇒ 𝑥 = 𝑦
In Isosceles Δ𝑅𝑃𝑄
∠𝑃𝑅𝑄 = ∠𝑃𝑅𝑄 = 𝑥
By sum of angles property in Δ𝑅𝑃𝑄:
𝑦 + 3𝑦 + 𝑦 = 180 ⇒ 3𝑦 = 108

J. Overlapping Triangles

Example 3.40
Triangles 𝐴𝐵𝐶 and 𝐴𝐵𝐷 are isosceles with 𝐴𝐵 = 𝐴𝐶 = 𝐵𝐷, and 𝐵𝐷 intersects 𝐴𝐶 at 𝐸.
If 𝐵𝐷 ⊥ 𝐴𝐶, then what is the value of ∠𝐶 + ∠𝐷? (1996 AHSME)

∠𝐶 = 𝑐 ⇒ ∠𝐴𝐵𝐶 = 𝑐
In Δ𝐸𝐵𝐶:
∠𝐵𝐸𝐶 = 90 ⇒ ∠𝐸𝐵𝐶 = 90 − 𝑐
∠𝐷 = 𝑑 ⇒ ∠𝐵𝐴𝐷 = 𝑑 ⇒ 𝐴𝐵𝐷 = 180 − 2𝑑

∠𝐴𝐵𝐶 = ∠𝐴𝐵𝐷 + ∠𝐸𝐵𝐶 ⇒ ⏟


𝑐 =⏟
180 − 2𝑑 + 90
⏟ − 𝑐 ⇒ 𝑐 + 𝑑 = 135
∠𝐴𝐵𝐶 ∠𝐴𝐵𝐷 ∠𝐸𝐵𝐶

K. Quadrilaterals

Example 3.41
In the figure, 𝐴𝐵𝐶𝐷 and 𝐵𝐸𝐹𝐺 are squares, and 𝐵𝐶𝐸 is an equilateral triangle.
What is the number of degrees in angle 𝐺𝐶𝐸? (MathCounts 1991 National
Countdown)

In Δ𝐶𝐵𝐺

P a g e 31 | 152
Get all the files at: https://bit.ly/azizhandouts
Aziz Manva ([email protected])

∠𝐶𝐵𝐺 = 60 + 90 = 150
Also Δ𝐶𝐵𝐺 is isosceles, so
180 − 150 30
∠𝐵𝐶𝐺 = = = 15
2 2
And finally
∠𝐺𝐶𝐸 = ∠𝐵𝐶𝐸 − ∠𝐵𝐶𝐺 = 60 − 15 = 45

Example 3.42
Square 𝐴𝐵𝐶𝐷 and equilateral triangle 𝐴𝐸𝐷 are coplanar and share ̅̅̅̅
𝐴𝐷, as shown. What
is the measure, in degrees, of angle 𝐵𝐴𝐸? (MathCounts 2004 School Sprint)

∠𝐵𝐴𝐸 = ∠𝐵𝐴𝐷 − ∠𝐸𝐴𝐷 = 90 − 60 = 30°

Example 3.43
Jane makes a square 𝑊𝑋𝑌𝑍. John identifies point 𝑃 outside the square such that Δ𝑃𝑊𝑋 is equilateral. Find
A. ∠𝑃𝑊𝑍
B. ∠𝑊𝑃𝑍
C. ∠𝑍𝑃𝑌
D. ∠𝑃𝑍𝑌

Draw a diagram.
Part A
∠𝑃𝑊𝑍 = ∠𝑃𝑊𝑋 + ∠𝑋𝑊𝑍 = 60 + 90 = 150
Part B
180 − 150 30
∠𝑊𝑃𝑍 = = = 15
2 2
Part C
∠𝑍𝑃𝑌 = 60 − 15 − 15 = 30
Part D
∠𝑃𝑍𝑌 = 90 − 15 = 75

(Continuation) Example 3.44


Equilateral Δ𝑃𝑊𝑋 is constructed on side 𝑊𝑋. Similarly, construct equilateral Δ𝑄𝑊𝑍 outside the square. Find
∠𝑃𝑄𝑊.
Draw a diagram.
By angles around a point:
∠𝑄𝑊𝑃 = 360 − 60 − 90 − 60 = 150
In Isosceles Δ𝑃𝑄𝑊:
180 − 150 30
∠𝑃𝑄𝑊 = = = 15
2 2

(Continuation) Example 3.45


Note that equilateral Δ𝑃𝑊𝑋 is constructed on side 𝑊𝑋. Similarly, construct
equilateral Δ𝑄′𝑊𝑍 inside the square. Find ∠𝑃𝑄′𝑊.

P a g e 32 | 152
Get all the files at: https://bit.ly/azizhandouts
Aziz Manva ([email protected])

Draw a diagram.
∠𝑃𝑊𝑄 ′ = 60 + 30 = 90
In Isosceles Δ𝑃𝑊𝑄′
180 − 90 90
∠𝑃𝑄 ′ 𝑊 = = = 45
2 2

Challenge 3.46
In quadrilateral 𝑃𝑄𝑅𝑆, diagonals 𝑃𝑅 and 𝑆𝑄 intersect at 𝑂 inside 𝑃𝑄𝑅𝑆, 𝑆𝑃 = 𝑆𝑄 = 𝑆𝑅 =
1, and ∠𝑄𝑆𝑅 = 2∠𝑄𝑆𝑃. Marc determines the measure of the twelve angles that are the
interior angles of Δ𝑃𝑂𝑆, Δ𝑃𝑂𝑄, Δ𝑅𝑂𝑆, and Δ𝑅𝑂𝑄. The measure of each of these angles, in
degrees, is a positive integer, and exactly six of these integers are prime numbers. How
many different quadrilaterals have these properties and are not rotations or translations
of each other? (Gauss 8 2019/25)

Let
∠𝑄𝑆𝑃 = 2𝜃 ⇒ ∠𝑄𝑆𝑅 = 2 × 2𝜃 = 4𝜃

∴ In Isosceles Δ𝑃𝑆𝑅:
180 − ∠𝑃𝑆𝑅 180 − 6𝜃
∠𝑆𝑃𝑅 = ∠ 𝑆𝑅𝑃 = = = 90 − 3𝜃
2 2
∴ In Isosceles Δ𝑆𝑃𝑄
180 − ∠𝑃𝑆𝑄 180 − 2𝜃
∠𝑆𝑃𝑄 = ∠𝑆𝑄𝑃 = = = 90 − 𝜃
2 2
∠𝑅𝑃𝑄 = ∠𝑆𝑃𝑄 − ∠𝑆𝑃𝑅 = 90 − 𝜃 − (90 − 3𝜃) = 90 − 𝜃 − 90 + 3𝜃 = 2𝜃

In ΔPOQ:
∠𝑃𝑂𝑄 = 180 − 2𝜃 (90 − 𝜃) = 180 − 2𝜃 − 90 + 𝜃 = 90 − 𝜃
⏟ −⏟
∠𝑂𝑃𝑄 ∠𝑃𝑄𝑆

By Vertically Opposite Angles:


∠𝑅𝑂𝑆 = ∠𝑃𝑂𝑄 = 90 − 𝜃
By angles in a linear pair:
∠𝑄𝑂𝑅 = 180 − ⏟ (90 − 𝜃) = 180 − 90 + 𝜃 = 90 + 𝜃
∠𝑃𝑂𝑄
By Vertically Opposite Angles:
∠𝑃𝑂𝑆 = ∠𝑄𝑂𝑅 = 90 + 𝜃
𝑆𝑄 = 𝑆𝑅
∴ In Isosceles Triangle 𝑆𝑄𝑅:
180 − ∠𝑄𝑆𝑅 180 − 4𝜃
∠𝑆𝑄𝑅 = ∠𝑆𝑅𝑄 = = = 90 − 2𝜃
2 2
In Triangle 𝑅𝑂𝑄:
∠𝑂𝑅𝑄 = 180 − (90
⏟ + 𝜃) − (90
⏟ − 2𝜃) = 180 − 90 − 𝜃 − 90 + 2𝜃 = 𝜃
∠𝑄𝑂𝑅 ∠𝑂𝑄𝑅
∠𝑆𝑅𝑃 = ⏟
90 − 2𝜃 − ⏟
𝜃 = 90 − 3𝜃
∠𝑆𝑅𝑄 ∠𝑃𝑅𝑄

P a g e 33 | 152
Get all the files at: https://bit.ly/azizhandouts
Aziz Manva ([email protected])

L. Polygons

Example 3.47
In regular pentagon 𝐴𝐵𝐶𝐷𝐸, diagonal 𝐴𝐶 is drawn, as shown. Given that each interior
angle of a regular pentagon measures 108 degrees, what is the measure of angle
𝐶𝐴𝐵? (MathCounts 2010 School Sprint)

180 − 108 72
∠𝐶𝐴𝐵 = = = 36
2 2

M. Counting

Challenge 3.48
In triangle 𝐴𝐵𝐶, 𝐴𝐵 is congruent to 𝐴𝐶, the measure of angle 𝐴𝐵𝐶 is 72 and segment 𝐵𝐷
bisects angle 𝐴𝐵𝐶 with point 𝐷 on side 𝐴𝐶. If point 𝐸 is on side 𝐵𝐶 such that segment
𝐷𝐸 is parallel to side 𝐴𝐵, and point 𝐹 is on side 𝐴𝐶 such that segment 𝐸𝐹 is parallel to
segment 𝐵𝐷, how many isosceles triangles are in the figure shown? (MathCounts 2008
State Sprint)

Let’s calculate the angles first.


𝐴𝐵 ≅ 𝐴𝐶 ⇒ ∠𝐴𝐵𝐶 = ∠𝐴𝐶𝐵 = 72

Ans=7

3.3 Sides
A. Perimeter
In an isosceles triangle, not only are the angles congruent, the sides opposite them are also congruent.
In fact, this also works the other way around.
If two sides of a triangle are congruent, it is an isosceles triangle, and the angles
opposite the sides are congruent.

Example 3.49
The perimeter of Δ𝐴𝐵𝐶 is 32. If ∠𝐴𝐵𝐶 = ∠𝐴𝐶𝐵 and 𝐵𝐶 = 12, what is the length of
𝐴𝐵? (CEMC 2009 Gauss 8)

32 − 12 20
𝐴𝐵 = = = 10
2 2

Example 3.50
If the base of an isosceles triangle is 12 units long, and the perimeter is 35 units, what are the lengths of the
other two sides?

35 – 12 23
Each side = = = 11.5°
2 2

Example 3.51
Two sides of an isosceles triangle are 15 cm and 10 cm. What is the greatest possible perimeter of this triangle,
in centimetres? (MathCounts 2005 School Countdown)

P a g e 34 | 152
Get all the files at: https://bit.ly/azizhandouts
Aziz Manva ([email protected])

15+15+10=40

Example 3.52
The congruent sides of an isosceles triangle are each 5 cm long, and the perimeter is 17 cm. In centimeters,
what is the length of the base? (MathCounts 2004 Chapter Countdown)

17 − 5 − 5 = 7

B. Identifying Triangles

Example 3.53
An isosceles triangle is a triangle with two sides of equal length. How many
of the five triangles on the square grid below are isosceles? (AMC 8
1988/9)

C. Ratios

D. Algebra

Example 3.54
In the triangle, ∠𝐴 = ∠𝐵. What is 𝑥? (MathCounts 1996 State Countdown)

2𝑥 + 2 = 3𝑥 − 1 ⇒ 𝑥 = 3

E. Multiple Triangles

3.4 Area
A. Finding Area

Example 3.55
An isosceles triangle has sides 13, 13 and 10. What is the area of the triangle?

1
𝐴𝑟𝑒𝑎 𝑜𝑓 Δ = ℎ𝑏 = 2 × 12 × 10 = 60
2

Example 3.56
What is the number of square units in the area of a triangle whose sides measure 5, 5 and 6 units? (MathCounts
2004 Chapter Countdown)

1 1
𝐴𝑟𝑒𝑎 𝑜𝑓 Δ = ℎ𝑏 = × 4 × 6 = 12
2 2

Example 3.57
An isosceles triangle has sides 𝑥, 𝑥 and 𝑦. In terms of 𝑥 and 𝑦, what is the area of the triangle?

P a g e 35 | 152
Get all the files at: https://bit.ly/azizhandouts
Aziz Manva ([email protected])

By the Pythagorean Theorem:


𝑦 2 𝑦 2 4𝑥 2 − 𝑦 2
𝑥 2 = ℎ2 + ( ) ⇒ ℎ2 = 𝑥 2 − =
2 4 4
Taking square roots both sides:
4𝑥 2 − 𝑦 2 √4𝑥 2 − 𝑦 2
ℎ=√ =
4 2
And, now we can find the area of the triangle as:
1 1 √4𝑥 2 − 𝑦 2 𝑦√4𝑥 2 − 𝑦 2
𝐴𝑟𝑒𝑎 𝑜𝑓 Δ = ℎ𝑏 = × ×𝑦 =
2 2 2 4

Example 3.58
In △ 𝐴𝐵𝐶, 𝐴𝐵 = 𝐵𝐶 = 29, and 𝐴𝐶 = 42. What is the area of △ 𝐴𝐵𝐶? (AMC 8 2015/6)

B. Finding Sides

Example 3.59
The base of isosceles △ 𝐴𝐵𝐶 is 24 and its area is 60. What is the length of one of the congruent sides? (AMC 8
2007/14)

Example 3.60
Let 𝐴 be the area of the triangle with sides of length 25, 25, and 30. Let 𝐵 be the area
of the triangle with sides of length 25, 25, and 40. What is the relationship between 𝐴 and 𝐵? (AMC 8 2011/16)
9 3 4 16
(𝐴)𝐴 = 𝐵 (𝐵)𝐴 = 𝐵 (𝐶)𝐴 = 𝐵 (𝐷)𝐴 = 𝐵 (𝐸)𝐴 = 𝐵
16 4 3 9

1 1
𝐴𝑟𝑒𝑎 𝑜𝑓 𝑇𝑟𝑖𝑎𝑛𝑔𝑙𝑒 = ℎ𝑏 = ( 𝑏) ℎ
2 2
From the diagram, we see that the values are the same in both triangles (but interchanged).

P a g e 36 | 152
Get all the files at: https://bit.ly/azizhandouts
Aziz Manva ([email protected])

Hence, the areas are equal.


Option C is correct.

C. Multiple Figures

Example 3.61
A triangle has side lengths 10, 10, and 12. A rectangle has width 4 and area equal to the area of the triangle.
What is the perimeter of this rectangle? (AMC 10B 2010/7)

3.5 Review
A. Cumulative Review
Example 3.62
State true or false:
A. A triangle has six exterior angles.
B. A triangle with one obtuse angle is called obtuse-angled, with one right angle is called right-angled, and
with one acute angle is called acute-angled.
C. The sum of the six exterior angles of a triangle is always equal to 360 degrees.
D. The hypotenuse is the shortest side of a right-angled triangle.

True
False. All three angles must be acute for it to be called an acute-angled triangle.
True
False. It is the longest side.

Example 3.63
Point O lies between parallel lines AB and XY, to the left of A and X. What is ∠BAO + ∠ABO + ∠YXO?
Connect points A and X.
Sum of angles of △AXO is 180.
∠BAX + YXA = 180 (Co-Interior Angles)

3.6 Applications (Optional)


A. Circle

3.64: Angle subtended by diameter of circle is right angle


Consider two points 𝐴𝐵 on the circumference of a circle. If 𝐴𝐵 is a diameter of the circle, and 𝐶 is any point
other than 𝐴 and 𝐵 on the circumference on the circle, then show Δ𝐴𝐵𝐶 is a right triangle.

Note that O is center of circle the circle and hence


𝑂𝐴 = 𝑂𝐶 = 𝑂𝐵 = 𝑟

In Isosceles Δ𝐴𝑂𝐶, let


∠𝐶𝐴𝑂 = ∠𝐴𝐶𝑂 = 𝛼
In Isosceles Δ𝐶𝑂𝐵, let
∠𝑂𝐶𝐵 = ∠𝑂𝐵𝐶 = 𝛽

In Δ𝐴𝐶𝐵, by sum of angles of a triangle:

P a g e 37 | 152
Get all the files at: https://bit.ly/azizhandouts
Aziz Manva ([email protected])

∠𝐶𝐴𝐵 + ∠𝐴𝐶𝐵 + ∠𝐶𝐵𝐴 = 180°


𝛼 + (𝛼 + 𝛽) + 𝛽 = 180°
2𝛼 + 2𝛽 = 180°
𝛼 + 𝛽 = 90° ⇒ ∠𝐴𝐶𝐵 = 90° ⇒ Δ𝐴𝐵𝐶 𝑖𝑠 𝑎 𝑟𝑖𝑔ℎ𝑡 𝑡𝑟𝑖𝑎𝑛𝑔𝑙𝑒

P a g e 38 | 152
Get all the files at: https://bit.ly/azizhandouts
Aziz Manva ([email protected])

PART II: AREA AND PERIMETER


4. BASICS
4.1 Summary and Formulas
A. Area and Perimeter of Triangles
2-Dimensional Figures
Area Perimeter Height
Triangle
General 1 Sum of lengths of sides
ℎ𝑏
Formula 2
Heron’s √𝑠 (𝑠 − 𝑎)(𝑠 − 𝑏)(𝑠 − 𝑐)
Formula s = semi perimeter = 0.5(a + b +
c)
a, b, c are the sides of the triangle
Right Angled ½ * Lengths of the perpendicular
Triangle (Non-hypotenuse) sides
Equilateral √3 Side * 3 √3
Triangle Side2 × Side
4 2
Two triangles with:
➢ same base and height have equal areas
➢ same height have area in proportion to base
➢ same base have area in proportion to height

Similar triangles have areas in proportion to the square of their sides.

4.2 Perimeter
A.

Example 4.1
Triangles 𝐴𝐵𝐶, 𝐴𝐷𝐸, and 𝐸𝐹𝐺 are all equilateral. Points 𝐷 and 𝐺 are midpoints of
𝐴𝐶 and 𝐴𝐸, respectively. If 𝐴𝐵 = 4, what is the perimeter of figure 𝐴/𝐵𝐶𝐷𝐸𝐹𝐺?
(AMC 8 2000/15)


4 +⏟
4 + ⏟
2 + ⏟
2 +⏟
1 +⏟
1 +⏟
1 = 15
𝐴𝐵 𝐵𝐶 𝐶𝐷 𝐷𝐸 𝐸𝐹 𝐹𝐺 𝐺𝐴

Example 4.2
The lengths of the sides of a triangle in inches are three consecutive integers. The length of the shortest side is
30% of the perimeter. What is the length of the longest side? (AMC 8 2010/13)

Let the sides be


𝑝 − 1, 𝑝, 𝑝 + 1
The perimeter is then:
𝑝 − 1 + 𝑝 + 𝑝 + 1 = 3𝑝
And the shortest side is 30% of the perimeter:
𝑝 − 1 = 0.3(3𝑝) ⇒ 𝑝 − 1 = 0.9𝑝 ⇒ 0.1𝑝 = 1 ⇒ 𝑝 = 10 ⇒ 𝑝 + 1 = 11

P a g e 39 | 152
Get all the files at: https://bit.ly/azizhandouts
Aziz Manva ([email protected])

Example 4.3
In the figure below, choose point 𝐷 on 𝐵𝐶 so that △ 𝐴𝐶𝐷 and △ 𝐴𝐵𝐷 have equal
perimeters. What is the area of △ 𝐴𝐵𝐷? (AMC 8 2017 /16)

Perimeter of Δ𝐴𝐵𝐷
= 𝐴𝐷 + 4 + (5 − 𝑥)
Perimeter of Δ𝐴𝐶𝐷
= 𝐴𝐷 + 3 + 𝑥
Since the two perimeters are equal, we must have:
𝐴𝐷 + 4 + (5 − 𝑥) = 𝐴𝐷 + 3 + 𝑥 ⇒ 𝑥 = 3 ⇒ 5 − 𝑥 = 2

Area of Δ𝐴𝐵𝐶
1 1
= [𝐴𝐵𝐶] = ℎ𝑏 = × 3 × 4 = 6
2 2
Note that Δ𝐴𝐵𝐷 and Δ𝐴𝐶𝐷 have the same height.
And hence, their areas will be in the ratio of their bases.

Hence, we need to divide [𝐴𝐵𝐶] in the ratio 2: 3:


2 12
×6=
5 5

Example 4.41
Sarah and Bob are trying to find the area of triangle ABC. Sarah mistakenly uses AB and the height from A
(instead of the height from C) and Barry mistakenly uses BC and the height from C(instead of the height from
A). Sarah finds an area of 12 and Bob finds an area of 27.
A. What is the area of triangle ABC?
B. What is the ratio AD:CE?

Method I
See Diagram

1/2 * c * h_2 = 12
1/2 * a * h_1 = 27

Multiply the two:


(1/2 * c * h_1) * (1/2 * a * h_1) = 12 * 27

Note that:
Area of Triangle = A = 1/2 * c * h_1 = 1/2 * a * h_1

1
https://artofproblemsolving.com/community/c3t48f3h2559714_mathematics

P a g e 40 | 152
Get all the files at: https://bit.ly/azizhandouts
Aziz Manva ([email protected])

Hence:
A^2 = 12 * 27 = 36 * 9
A = 18

Method II
Similarity

4.3 Right Triangles: Area


A. Area of a Right Triangle
Is given by half the product of the legs

Example 4.5
What is the area of a right triangle with legs 5 units and 6 units?

1 1
𝐴𝑟𝑒𝑎 = ℎ𝑏 = × 5 × 6 = 15
2 2

Example 4.6
Two sides of a right triangle have length 3 units and 4 units. What is the sum of the possible areas of the
triangle?
Case I: Given Sides are the Legs
1 1
𝐴𝑟𝑒𝑎 = ℎ𝑏 = × 3 × 4 = 3 × 2 = 6
2 2
Case II: 4 is the hypotenuse
Then, we must have
32 + 𝑏 2 = 42 ⇒ 𝑏 2 = 16 − 9 ⇒ 𝑏 = √7
1 1 3√7
𝐴𝑟𝑒𝑎 = ℎ𝑏 = × 3 × √7 =
2 2 2
Sum of the areas
3√7 12 + 3√7
=6+ =
2 2
Case III: 3 is the hypotenuse
The hypotenuse is the longest side. Hence, since
4>3
3 cannot be the hypotenuse.
B. Basics
We have already seen the basics of area of right triangles and isosceles triangles. Now, we look at more difficult
questions.

Example 4.7
Rectangle 𝐴𝐵𝐶𝐷 and right triangle 𝐷𝐶𝐸 have the same area. They are
joined to form a trapezoid, as shown. What is 𝐷𝐸? (AMC 8 2014/14)

P a g e 41 | 152
Get all the files at: https://bit.ly/azizhandouts
Aziz Manva ([email protected])

C. Right Triangles

Example 4.8 (Move to: Area Properties)


An isosceles right triangle with legs of length 8 is partitioned into 16 congruent
triangles as shown. The shaded area is (AMC 8 1992/10)

Method I
The area of the larger triangle is
1 1
ℎ𝑏 = × 8 × 8 = 32
2 2
Since 10 out of the 16 triangles are shaded, the shaded area is:
10 5
= × 32 = × 32 = 20
16 8

Method II
The area of each shaded triangle
1 1
ℎ𝑏 = × 2 × 2 = 2
2 2
And the area of the shaded region
= 2 × 10 = 20

Example 4.9
Points 𝐵, 𝐷, and 𝐽 are midpoints of the sides of right triangle 𝐴𝐶𝐺. Points 𝐾, 𝐸, 𝐼
are midpoints of the sides of triangle 𝐽𝐷𝐺, etc. If the dividing and shading process
is done 100 times (the first three are shown) and 𝐴𝐶 = 𝐶𝐺 = 6, then the total area
of the shaded triangles is nearest (AMC 8 1999/25)

1 1 9
[𝐵𝐶𝐷] = [𝐽𝐷𝐺] = ℎ𝑏 = × 3 × 3 = 𝑢𝑛𝑖𝑡𝑠 2
2 2 2
36 9 27
[𝐴𝐽𝐷𝐶] = [𝐺𝐶𝐴] − [𝐺𝐷𝐽] = − =
2 2 2
9
[𝐵𝐶𝐷] 9 1
= 2 = =
[𝐴𝐽𝐷𝐶] 27 27 3
2
Hence, the required shading is:
1 1 1
[𝐺𝐶𝐴] = × × 6 × 6 = 6
3 3 2

Example 4.10
Triangle 𝐴𝐵𝐶 has a right angle at 𝐵. Point 𝐷 is the foot of the altitude from 𝐵, 𝐴𝐷 = 3, and 𝐷𝐶 = 4. What is the
area of △ 𝐴𝐵𝐶? (AMC 10A 2009/10)

Example 4.11
The isosceles right triangle 𝐴𝐵𝐶 has right angle at 𝐶 and area 12.5. The rays trisecting ∠𝐴𝐶𝐵 intersect 𝐴𝐵 at 𝐷
and 𝐸. What is the area of △ 𝐶𝐷𝐸? (AMC 10A 2014/19)

P a g e 42 | 152
Get all the files at: https://bit.ly/azizhandouts
Aziz Manva ([email protected])

D. Finding Third Height

Example 4.12
Given a triangle with side lengths 15, 20, and 25, find the triangle's smallest height. (AMC 10A 2002/13)

(15,20,25) = 5 × (3,4,5) ⇒ 𝑅𝑖𝑔ℎ𝑡 − 𝑎𝑛𝑔𝑙𝑒𝑑 𝑇𝑟𝑖𝑎𝑛𝑔𝑙𝑒

1
𝐴 = 15 × 20 × = 150
2

Smallest height will be drawn to the longest base (hypotenuse):


1 150 × 2
× ℎ × 25 = 150 ⇒ ℎ = = 12
2 25

Example 4.13
The two legs of a right triangle, which are altitudes, have lengths 2√3 and 6. How long is
the third altitude of the triangle? (AMC 10A 2014/9)

E. Proving that a Triangle is Right

Example 4.14
Given the areas of the three squares in the figure, what is the area of the interior triangle?
(AMC 8 2003/6)

Example 4.15
2Δ 2Δ
The sides 𝑥 and 𝑦 of a scalene triangle satisfy 𝑥 + =𝑦+ , where Δ is the area of the triangle. If 𝑥 = 60, 𝑦 =
𝑥 𝑦
63, what is the length of the largest side of the triangle? (IOQM 2021/16)

2Δ 2Δ 2𝑦Δ − 2𝑥Δ 2Δ(𝑦 − 𝑥) 𝑥𝑦


− =𝑦−𝑥 ⇒ = 𝑦−𝑥 ⇒ = 𝑥𝑦 ⇒ Δ =
𝑥 𝑦 𝑥𝑦 𝑦−𝑥 2

(𝑥, 𝑦, 𝑧) = (60,63,89)

4.4 General Triangles


A. Inner and Outer

Example 4.16
In the figure, the outer equilateral triangle has area 16, the inner equilateral triangle has area 1, and the three
trapezoids are congruent. What is the area of one of the trapezoids? (AMC 8 2008/4)

P a g e 43 | 152
Get all the files at: https://bit.ly/azizhandouts
Aziz Manva ([email protected])

16 − 1 15
= =5
3 3

B. Finding Third Side

Example 4.17
Two sides of a triangle have lengths 10 and 15. The length of the altitude to the third side is the average of the
lengths of the altitudes to the two given sides. How long is the third side? (AMC 10A 2013/15)

2Δ = 10𝑥 = 15𝑦 = ℎ𝑏 = 𝑘
𝑘 𝑘
𝑥= ,𝑦 =
10 15
𝑘 𝑘 2𝑘 2𝑘
𝑏= = 𝑥+𝑦 = = = 12
ℎ 𝑘 𝑘 5𝑘
2 +
10 15 30

C. Inscribed Triangle

Example 4.18
Circle C has radius 6 cm. How many square centimeters are in the area of the largest possible inscribed triangle
having one side as a diameter of circle C? (MathCounts 2000 State Countdown)

1
× ⏟
6 × 12
⏟ = 36
2 𝐻𝑒𝑖𝑔ℎ𝑡 𝐵𝑎𝑠𝑒
=𝑅𝑎𝑑𝑖𝑢𝑠 =𝐷𝑖𝑎𝑚𝑒𝑡𝑒𝑟

D. Common Angle

4.19: Ratio of Area of Two Triangles with Angle of Same Measure


If in Δ𝐴𝐵𝐶 and ΔXYZ, ∠𝐵 = ∠𝑌, then
[𝐴𝐵𝐶] 𝐴𝐵 ∙ 𝐵𝐶
=
[𝑋𝑌𝑍] 𝑋𝑌 ∙ 𝑌𝑍

Example 4.20
In the right triangle △ 𝐴𝐶𝐸, we have 𝐴𝐶 = 12, 𝐶𝐸 = 16, and 𝐸𝐴 = 20. Points 𝐵, 𝐷, and 𝐹 are located on 𝐴𝐶, 𝐶𝐸,
and 𝐸𝐴, respectively, so that 𝐴𝐵 = 3, 𝐶𝐷 = 4, and 𝐸𝐹 = 5. What is the ratio of the area of △ 𝐷𝐵𝐹 to that of △
𝐴𝐶𝐸? (AMC 10B 2004/18)

P a g e 44 | 152
Get all the files at: https://bit.ly/azizhandouts
Aziz Manva ([email protected])

[𝐴𝐵𝐹] 𝐴𝐵 × 𝐴𝐹 3 × 15 3
= = =
[𝐴𝐶𝐸] 𝐴𝐶 × 𝐴𝐸 12 × 20 16
[𝐵𝐶𝐷] 𝐵𝐶 × 𝐶𝐷 9×4 3
= = =
[𝐴𝐶𝐸] 𝐴𝐶 × 𝐶𝐸 12 × 16 16
[𝐹𝐸𝐷] 𝐸𝐹 × 𝐸𝐷 5 × 12 3
= = =
[𝐴𝐶𝐸] 𝐴𝐸 × 𝐸𝐶 20 × 16 16
Using Complementary Areas:
[𝐷𝐵𝐹] [𝐴𝐵𝐹] [𝐵𝐶𝐷] [𝐹𝐸𝐷] 3 3 3 9 7
=1−( + + )=1−( + + )=1− =
[𝐴𝐶𝐸] [𝐴𝐶𝐸] [𝐴𝐶𝐸] [𝐴𝐶𝐸] 16 16 16 16 16

E. Mixing Concepts

Example 4.21
What is the area of a right-angled triangle (with hypotenuse 3 units) that has one angle equivalent to that of an
equilateral triangle?

Double the triangle to get an equilateral triangle.


Area of Right-angled triangle
= 0.5 * Area of Equilateral Triangle
= 0.5 * Root (3)/4 * 32
= 9 * Root (3)/8

Example 4.22
What is the maximum possible difference in areas of two triangles with perimeter 21 and integer side lengths
(if the one with less area is isosceles)?
A. [21Root (3) – 10]/3
B. [49Root (3) – 20]/4
C. [441Root (3)]/12
D. [64Root (5) – 10]/7

Dimensions of smaller triangle are 1, 10, 10


ℎ(Smaller Δ) = √102 − 0.52 = √99.75 ≈ 10
𝑏ℎ 1 × 10
𝐴(Smaller Δ) = = =5
2 2
√3 49√3
A[Greater △ (equilateral)] = × 72 =
4 4
49√3 49√3 − 20
Difference = −5=
4 4

Example 4.23
An isosceles right-angled triangle has 2/3 the area of an equilateral triangle with length 3. What is the
difference between the lengths of the longest sides of the two triangles?

P a g e 45 | 152
Get all the files at: https://bit.ly/azizhandouts
Aziz Manva ([email protected])

5. APPLICATIONS
5.1 Ratios
A. Equal Bases and Equal Heights

5.1: Triangles with Equal Bases


Triangles with equal bases have areas in the ratio of their heights.

5.2: Triangles with Equal Heights


Triangles with equal heights have areas in the ratio of their heights.

B. Median

5.3: Median divides triangle into two triangles with equal areas
Show that the two triangles formed by drawing the median of a triangle have equal areas

5.4: Area of Quadrilateral formed by Midpoints


Show that the area of the quadrilateral formed by joining the midpoints of the sides of a quadrilateral in order
has area half that of the original quadrilateral.

5.2 Similarity
A. Ratio of Areas

5.5: Ratio of Areas


If two similar shapes have side lengths in the ratio 𝑎: 𝑏, then their areas are in the ratio
𝑎2 : 𝑏 2

Example 5.6
Two similar triangles have sides in the ratio 1:3. What is the ratio of their areas?

𝑅𝑎𝑡𝑖𝑜 𝑜𝑓 𝑎𝑟𝑒𝑎𝑠 = 12: 32: = 1: 9

Example 5.7
What is the ratio of the area of the smaller triangle formed by joining the midpoints of a triangle to the original
triangle?

Example 5.8
An equilateral triangle of side length 10 is completely filled in by non-overlapping equilateral triangles of side
length 1. How many small triangles are required? (AMC 10B 2008/7)

Example 5.9
Let 𝐴𝐵𝐶 be an equilateral triangle. Extend side 𝐴𝐵 beyond 𝐵 to a point 𝐵′ so that 𝐵𝐵′ = 3 ⋅ 𝐴𝐵. Similarly,
extend side 𝐵𝐶 beyond 𝐶 to a point 𝐶 ′ so that 𝐶𝐶 ′ = 3 ⋅ 𝐵𝐶, and extend side 𝐶𝐴 beyond 𝐴 to a point 𝐴′ so that

P a g e 46 | 152
Get all the files at: https://bit.ly/azizhandouts
Aziz Manva ([email protected])

𝐴𝐴′ = 3 ⋅ 𝐶𝐴. What is the ratio of the area of △ 𝐴′ 𝐵′ 𝐶 ′ to the area of △ 𝐴𝐵𝐶? (AMC 10B 2017/19)

5.3 Complementary Areas

Example 5.10
Squares 𝐴𝐵𝐶𝐷, 𝐸𝐹𝐺𝐻, and 𝐺𝐻𝐼𝐽 are equal in area. Points 𝐶 and 𝐷 are the
midpoints of sides 𝐼𝐻 and 𝐻𝐸, respectively. What is the ratio of the area of the
shaded pentagon 𝐴𝐽𝐼𝐶𝐵 to the sum of the areas of the three squares? (AMC 8
2013/24)

Add Point D to make Rectangle 𝐵𝐷𝐼𝐶.


1 1
[𝐴𝐽𝐼𝐶𝐵] = [𝐴𝐽𝐷] − [𝐵𝐷𝐼𝐶] = × 2 × 1.5 − 1 × = 1.5 − 0.5 = 1
2 2
Ratio
= 1: 3

5.4 Other Topics


A. Other Concepts
Area of a triangle (or any polygon) will be maximum when the side lengths are equal.
Area of a triangle (or any polygon) will be minimum when the difference in side lengths is maximum.

Consider a rectangle with perimeter 18, and integer side lengths.


Hence, the sum of the length and the breadth is 9.

Possible values of the sides are:


Area (1, 8) = 8 8 7
Area (2, 7) = 14 14 5
Area (3, 6) = 18 18 3
Area (4, 5) = 20 20 1

Example 5.11
Equilateral △ 𝐴𝐵𝐶 has side length 2, 𝑀 is the midpoint of 𝐴𝐶,
and 𝐶 is the midpoint of 𝐵𝐷. What is the area of △ 𝐶𝐷𝑀? (AMC

P a g e 47 | 152
Get all the files at: https://bit.ly/azizhandouts
Aziz Manva ([email protected])

10B 2005/14)

Example 5.12
Triangle 𝐴𝑀𝐶 is isosceles with 𝐴𝑀 = 𝐴𝐶. Medians 𝑀𝑉 and 𝐶𝑈 are perpendicular to
each other, and 𝑀𝑉 = 𝐶𝑈 = 12. What is the area of △ 𝐴𝑀𝐶? (AMC 10A 2020/12)

P a g e 48 | 152
Get all the files at: https://bit.ly/azizhandouts
Aziz Manva ([email protected])

6. HERON’S FORMULA
6.1 Heronian Triangles
A. Area in Terms of Height and Base
We already know one formula for the area of a triangle.

6.1: Area of a Triangle


1
𝐴𝑟𝑒𝑎 = 𝐴 = ℎ𝑏
2

➢ If we do not know the height, we can still calculate the area using Heron’s Formula.
➢ Heron’s Formula is computationally involved, so we usually prefer other formulas before it.
B. Heron’s Formula

6.2: Heron’s Formula


𝐴𝑟𝑒𝑎 𝑜𝑓 𝑎 𝑇𝑟𝑖𝑎𝑛𝑔𝑙𝑒 = √𝑠 (𝑠 − 𝑎)(𝑠 − 𝑏)(𝑠 − 𝑐)
Where:
𝑎, 𝑏, 𝑐 𝑎𝑟𝑒 𝑡ℎ𝑒 𝑠𝑖𝑑𝑒𝑠 𝑜𝑓 the 𝑡𝑟𝑖𝑎𝑛𝑔𝑙𝑒
𝑎+𝑏+𝑐
𝑠 = 𝑠𝑒𝑚𝑖𝑝𝑒𝑟𝑖𝑚𝑒𝑡𝑒𝑟 =
2

Example 6.3
What is the area of a triangle with side lengths 20 units, 15 units and 7 units?

20 + 15 + 7 42
𝑠= = = 21
2 2
𝐴 = √21 (1)(6)(14) = √7 × 3 × 3 × 2 × 2 × 7 = √72 × 32 × 22 = 7 × 3 × 2 = 42

6.4: Triangle with the same perimeter and area


The triangle with sides 20, 15 and 7 has the same perimeter and area
𝑃 = 20 + 15 + 7 = 42
𝐴 = 42

It is a triangle with the same perimeter as area.

C. Heronian Triangles
In the example above, we got a “nice” answer. Because of the square root in the formula, we are not guaranteed
that answers will be “nice”. In fact, integer answers for areas are the exception, not the rule. Hence, triangles
with integers have a name.
Just as three integers that are the sides of a right triangle are called a Pythagorean Triplet, so three integers that
form the sides of a triangle with integer area form a Heronian Triangle.

6.5: Heronian Triangle


A triangle with integer side lengths and integer area is called a Heronian Triangle

P a g e 49 | 152
Get all the files at: https://bit.ly/azizhandouts
Aziz Manva ([email protected])

Example 6.6
A garden is in the shape of a triangle. The side lengths of the garden are 17 meters, 10 meters and 9 meters.
What is the area of the garden?

17 + 10 + 9 36
𝑠= = = 18
2 2
𝐴 = √(18)(1)(8)(9) = √(9)(1)(16)(9) = 9 × 4 = 36
D. Nested Triangles

Example 6.7
A triangle with side lengths of 26 units, 25 units and 3 units is drawn inside a triangle with side lengths of 30
units, 29 units and 5 units. Find the area of the region between the two triangles.

Inner Triangle
26 + 25 + 3 54
𝑠= = = 27
2 2
𝐴 = √(27)(1)(2)(24) = √(9)(1)(12)(12) = 3 × 12 = 36
Outer Triangle
30 + 29 + 5 64
𝑠= = = 32
2 2
𝐴 = √(32)(2)(3)(27) = √(64)(81) = 8 × 9 = 72
Shaded Area
Area between the two triangles
= 72 − 36 = 36

E. Costs

Challenge 6.8
A triangular patch of land with side lengths 29 meters, 25 meters and 6 meters is to be fenced at a cost of $4 per
meter of fence. The poles of the fence costs $5, and poles are at a distance of 1 meter. The vertices of the
triangle need two poles each. It is also to be planted with grass at a cost of $7 per square meter. What is the
total cost involved?

Number of Poles Needed - Concept


We need two poles each at the vertices, which are represented using blue points on
the diagram.
If the side length is 2, then we need a single pole between the vertices, colored green.
If the side length is 3, then we need two poles between the vertices, colored green.

Cost of Poles
𝑁𝑜. 𝑜𝑓 𝑃𝑜𝑙𝑒𝑠 𝑎𝑡 𝑉𝑒𝑟𝑡𝑖𝑐𝑒𝑠 = 3 × 2 = 6
𝑁𝑜. 𝑜𝑓 𝑃𝑜𝑙𝑒𝑠 𝑜𝑛 𝑡ℎ𝑒 𝑆𝑖𝑑𝑒𝑠 = 28 + 24 + 5 = 57
𝐶𝑜𝑠𝑡 = 5 × (6 + 57) = 5 × 63 = 315
Cost of Fence
𝐶𝑜𝑠𝑡 𝑜𝑓 𝐹𝑒𝑛𝑐𝑒 = 4 × 60 = 240
Cost of Grass
29 + 25 + 6 60
𝑠= = = 30
2 2

P a g e 50 | 152
Get all the files at: https://bit.ly/azizhandouts
Aziz Manva ([email protected])

𝐴 = √(30)(1)(5)(24) = √(12)(1)(25)(12) = 12 × 5 = 60
𝐶𝑜𝑠𝑡 = 7 × 60 = 420

Total Cost
= 240 + 315 + 420 = 975

F. Back Calculations / Number Theory


Since Heronian Triangles have integer side lengths, we use this while solving for the values of the sides of the
triangle. This takes us in the direction of Number Theory, and Diophantine Equations.

6.9: Semiperimeter is greater than any side


𝑠>𝑎
𝑠>𝑏
𝑠>𝑐

Example 6.10
A triangle with area 24 units and perimeter 32 units has a side length of 15. If all sides of the triangle are
integers, find the length of the other two sides.

32
𝑆𝑒𝑚𝑖𝑝𝑒𝑟𝑖𝑚𝑒𝑡𝑒𝑟 = 𝑠 = = 16
2
Let the sides of the triangle be
𝑎, 𝑏 𝑎𝑛𝑑 𝑐
We know that 𝑎 = 15. We need to find 𝑏 and 𝑐.

Substitute 𝑎 = 15, 𝑠 = 16, 𝐴 = 24 in 𝐴 = √𝑠(𝑠 − 𝑎)(𝑠 − 𝑏)(𝑠 − 𝑐):


24 = √(16)(1)(16 − 𝑏)(16 − 𝑐)
Prime Factorize 24 and 16:
23 × 3 = √(24 )(1)(16 − 𝑏)(16 − 𝑐)
Square both sides. Note that on the RHS, the square and the square root cancel:
26 × 32 = 24 (16 − 𝑏)(16 − 𝑐)
4
Divide both sides by 2 :
36 = (16 − 𝑏)(16 − 𝑐)

Because the sides of the triangle are integers, each factor on the right must be an integer.
Hence, we find the factor pairs of 36:
(1,36)(2,18)(3,12)(4,9)

Any of the individual sides cannot be greater than the semiperimeter.


16 − 𝑏 > 0 ⇒ 16 > 𝑏 ⇒ 𝑏 < 16
16 − 𝑐 > 0 ⇒ 16 > 𝑐 ⇒ 𝑐 < 16

𝑉𝑎𝑙𝑖𝑑 𝑂𝑝𝑡𝑖𝑜𝑛𝑠 𝑎𝑟𝑒 (3,12)(4,9)

Try the factor pair (3,12):


16 − 𝑏 = 3 ⇒ 𝑏 = 13
16 − 𝑐 = 12 ⇒ 𝑐 = 4

P a g e 51 | 152
Get all the files at: https://bit.ly/azizhandouts
Aziz Manva ([email protected])

𝑎 + 𝑏 + 𝑐 = 15 + 13 + 4 = 32 ⇒ 𝑊𝑜𝑟𝑘𝑠

(Alternate Solution) Example 6.11


A triangle with area 24 units and perimeter 32 units has a side length of 15. Find the length of the other two
sides.

This time we do not know that the sides are integers, so we change our approach:
15 + 𝑏 + 𝑐 = 32 ⇒ 𝑐 = 17 − 𝑏

24 = √(16)(1)(16 − 𝑏)(16 − (17 − 𝑏))


36 = (16 − 𝑏)(−1 + 𝑏)
36 = −16 + 16𝑏 + 𝑏 − 𝑏 2
𝑏 2 − 17𝑏 + 52 = 0
(𝑏 − 13)(𝑏 − 4) = 0
𝑏 ∈ {4,13}

Example 6.12
A triangle with integer side lengths has area 84, perimeter 48 units and a side length of 21. Find the sides of the
triangle.

48
𝑠= = 24
2
84 = √(24)(3)(24 − 𝑏)(24 − 𝑐)
2 × 32 × 72 = 23 × 32 (24 − 𝑏)(24 − 𝑐)
4

2 × 72 = (24 − 𝑏)(24 − 𝑐)
98 = (24 − 𝑏)(24 − 𝑐)

𝐹𝑎𝑐𝑡𝑜𝑟 𝑃𝑎𝑖𝑟𝑠 𝑜𝑓 98 = (1,98)(2,49)(7,14)

24 − 𝑏 = 7 ⇒ 𝑏 = 24 − 7 = 17
24 − 𝑐 = 14 ⇒ 𝑐 = 24 − 14 = 10

21 + 17 + 10 = 48 ⇒ 𝑊𝑜𝑟𝑘𝑠

Example 6.13
A triangle with integer side lengths has area 84, perimeter 72 units and a side length of 35. Find the sides of the
triangle.

72
𝑠= = 36
2
84 = √(36)(1)(36 − 𝑏)(36 − 𝑐)
2 × 32 × 72 = 22 × 32 (36 − 𝑏)(36 − 𝑐)
4

22 × 72 = (36 − 𝑏)(36 − 𝑐)
196 = (36 − 𝑏)(36 − 𝑐)

𝐹𝑎𝑐𝑡𝑜𝑟 𝑃𝑎𝑖𝑟𝑠 𝑜𝑓 196 = (1,196)(2,98)(4,49)(7,28)

36 − 𝑏 = 7 ⇒ 𝑏 = 36 − 7 = 29

P a g e 52 | 152
Get all the files at: https://bit.ly/azizhandouts
Aziz Manva ([email protected])

36 − 𝑐 = 28 ⇒ 𝑐 = 36 − 28 = 8

35 + 29 + 8 = 72 ⇒ 𝑊𝑜𝑟𝑘𝑠
G. Primitive Heronian Triangles
When we learn Pythagorean Triplets, which are the side lengths of a right triangle, which are integers, we are
most interested in Primitive Pythagorean Triplets, since all other triplets are multiples of these.
Similarly, for Heronian Triangles, we are interested in primitive Heronian Triangles.

6.14: Primitive Heronian Triangles


A Heronian triangle with sides 𝑎, 𝑏, and 𝑐 is primitive if
𝐻𝐶𝐹(𝑎, 𝑏, 𝑐) = 1

Example 6.15
The following triangles are Heronian Triangles. Are they primitive Heronian Triangles?
A. 3,4,5
B. 6,5,5
C. 26,25,3
D. 20,15,7
E. 30,26,8
F. 65,55,12

𝐻𝐶𝐹(3,4,5) = 1 ⇒ 𝑃𝑟𝑖𝑚𝑖𝑡𝑖𝑣𝑒
𝐻𝐶𝐹(6,5,5) = 1 ⇒ 𝑃𝑟𝑖𝑚𝑖𝑡𝑖𝑣𝑒
𝐻𝐶𝐹(26,25,3) = 1 ⇒ 𝑃𝑟𝑖𝑚𝑖𝑡𝑖𝑣𝑒
𝐻𝐶𝐹(20,15,7) = 1 ⇒ 𝑃𝑟𝑖𝑚𝑖𝑡𝑖𝑣𝑒
𝐻𝐶𝐹(30,26,8) = 2 ⇒ 𝑁𝑜𝑛 − 𝑃𝑟𝑖𝑚𝑖𝑡𝑖𝑣𝑒
𝐻𝐶𝐹(65,55,12) = 1 ⇒ 𝑃𝑟𝑖𝑚𝑖𝑡𝑖𝑣𝑒

6.16: List of Primitive Heronian Triangles


The table below gives a list of Heronian triangles, arranged in order of increasing area. These are all primitive.

Side Lengths Perimeter Semi- Area


Perimeter
15, 13, 4 24
17, 10, 9 36
26, 25, 3 36
20, 15, 7 42
29,25,6 60
20,13,11 66
30,29,5 72
15,14,13 84
21,17,10 84
35,29,8 84

6.17: Pythagorean Triplet Triangles


All triangles with side lengths which are Pythagorean Triplets are Heronian Triangles.

Primitive Pythagorean Triplets

P a g e 53 | 152
Get all the files at: https://bit.ly/azizhandouts
Aziz Manva ([email protected])

All Pythagorean Triplets have integer side lengths. The 3, 4, 5 20, 21, 29 11, 60, 61 36, 77, 85
area of the triangle is given by: 5, 12, 13 12, 35, 37 33, 56, 65 13, 84, 85
1 8, 15, 17 9, 40, 41 16, 63, 65 39, 80, 89
ℎ𝑏
2 7, 24, 25 28, 45, 53 48, 55, 73 65, 72, 97
In the adjoining table, note that exactly one of the legs is
even in every Pythagorean Triplet.
Hence:
1
ℎ𝑏 𝑖𝑠 𝑎𝑙𝑤𝑎𝑦𝑠 𝑎𝑛 𝑖𝑛𝑡𝑒𝑔𝑒𝑟 ⇒ 𝐴𝑙𝑙 𝑃𝑦𝑡ℎ𝑎𝑔𝑜𝑟𝑒𝑎𝑛 𝑇𝑟𝑖𝑝𝑙𝑒𝑡𝑠 𝑎𝑟𝑒 𝐻𝑒𝑟𝑜𝑛𝑖𝑎𝑛 𝑇𝑟𝑖𝑎𝑛𝑔𝑙𝑒𝑠
2
(Note: This is not a proof of the property).

6.18: Pythagorean Triple


For a right-angled triangle with sides 𝑝, 𝑞, 𝑟, where (𝑝, 𝑞, 𝑟) is a primitive Pythagorean Triplet, and 𝑟 is the
hypotenuse,
1 1
𝐴𝑟𝑒𝑎 = ℎ𝑏 = 𝑝𝑞 ∈ ℕ
2 2

We want to show that exactly one of 𝑝 and 𝑞 is even. That is, in


the table alongside, either Case III is true, or Case IV is true. I II III IV
𝑝 Even Odd Even Odd
We will do this using proof by contradiction. 𝑞 Even Odd Odd Even

Suppose Case I is true.


𝑝 𝑖𝑠 𝑒𝑣𝑒𝑛 ⇒ 𝑝2 𝑖𝑠 𝑒𝑣𝑒𝑛
𝑞 𝑖𝑠 𝑒𝑣𝑒𝑛 ⇒ 𝑞 2 𝑖𝑠 𝑒𝑣𝑒𝑛
⏟2 + 𝑞
𝑝 ⏟2 = 𝑟⏟2 ⇒ 𝐻𝐶𝐹(𝑝, 𝑞, 𝑟) ≥ 2 ⇒ (𝑝, 𝑞, 𝑟) 𝑖𝑠 𝑛𝑜𝑡 𝑝𝑟𝑖𝑚𝑖𝑡𝑖𝑣𝑒 ⇒ 𝐶𝑜𝑛𝑡𝑟𝑎𝑑𝑖𝑐𝑡𝑖𝑜𝑛
𝐸𝑣𝑒𝑛 𝐸𝑣𝑒𝑛 𝐸𝑣𝑒𝑛
Hence, Case I cannot be true.

Suppose Case II is true.


𝑝 𝑖𝑠 𝑜𝑑𝑑, 𝑞 𝑖𝑠 𝑜𝑑𝑑
Any odd number must be of the form 1=0+1 3=4−1
4𝑛 ± 1 5=4+1 7=8−1
(4𝑛 ± 1)2 = 16𝑛2 ± 8𝑛 + 1
One More than a Three More than a
Multiple of 4, then it Multiple of 4, then it can
𝑥 ≡ 1(𝑚𝑜𝑑 4) 𝑂𝑟 𝑥 ≡ 3(𝑚𝑜𝑑 4) ⇒ 𝑥 ≡ ±1(𝑚𝑜𝑑 4)
can be written as: be written as:
4𝑛 + 1 4𝑚 + 3 = 4𝑛 − 1
𝑥 2 ≡ 1(𝑚𝑜𝑑 4)

𝑝2 + 𝑞 2 ≡ 1 + 1 ≡ 2(𝑚𝑜𝑑 4)
The above statement means that 𝑝2 + 𝑞 2 is divisible by 2, but not by 4.

However, 𝑝2 + 𝑞 2 = 𝑟 2 and 𝑟 2 is a perfect square.


A perfect square that is divisible by 2, must be divisible by 4.
𝐶𝑜𝑛𝑡𝑟𝑎𝑑𝑖𝑐𝑡𝑖𝑜𝑛
𝐶𝑎𝑠𝑒 𝐼𝐼 𝑖𝑠 𝑛𝑜𝑡 𝑝𝑜𝑠𝑠𝑖𝑏𝑙𝑒.

6.19: Joining Pythagorean Triplet Triangles


A Pythagorean Triplet results in a Heronian Triangle. Putting two triangles which have side lengths from a
Pythagorean Triplet also results in a Heronian Triangle.

P a g e 54 | 152
Get all the files at: https://bit.ly/azizhandouts
Aziz Manva ([email protected])

Example 6.20
Two triangles with sides 3, 4 and 5 are joined together at a side of the same length. Find the possible values of
the perimeter of the resulting shape.

Case I: Joined at the Hypotenuse


This results in a square.
𝑃𝑒𝑟𝑖𝑚𝑒𝑡𝑒𝑟 = 2(4 + 3) = 2 × 7 = 14

Case II: Joined at a Leg


This has two possible outcomes, both resulting
in a Heronian Triangle with
𝑃𝑒𝑟𝑖𝑚𝑒𝑡𝑒𝑟 = 5 + 5 + 6 = 16
𝑃𝑒𝑟𝑖𝑚𝑒𝑡𝑒𝑟 = 5 + 5 + 8 = 18

H. Integer Multiples
A triangle with sides which are an integer
multiple of a Heronian Triangle will also be
Heronian.

6.21: Multiple of a Heronian Triangle


If (𝑎, 𝑏, 𝑐) is a Heronian Triangle with area 𝐴2 , then the triangle (𝑥𝑎, 𝑥𝑏, 𝑥𝑐) is also a Heronian Triangle with
area 𝑥 2 𝐴.
𝐴𝑟𝑒𝑎(𝑎, 𝑏, 𝑐) = 𝐴 ⇒ 𝐴𝑟𝑒𝑎(𝑥𝑎, 𝑥𝑏, 𝑥𝑐) = 𝑥 2 𝐴

Each side of the first triangle is multiplied by the (same) constant multiple to get the second triangle.
Hence, the second

6.2 Non-Heronian Triangles


A. Non-Heronian Triangles: Integer Semi-Perimeter

Example 6.22
What is the area of a triangle with sides 5, 6, and 7?

𝑎 + 𝑏 + 𝑐 5 + 6 + 7 18
𝑆𝑒𝑚𝑖𝑝𝑒𝑟𝑖𝑚𝑒𝑡𝑒𝑟 = 𝑠 = = = =9
2 2 2
𝐴 = √9 (2)(3)(4) = 6√6

B. Non-Heronian Triangles: Non-Integer Semi-Perimeter


If the semiperimeter of a triangle is odd, then each term in the formula is divided by 2. This generally increases
the calculations.

Example 6.23
What is the area of a triangle with sides 3, 4, and 6?

𝑎 + 𝑏 + 𝑐 3 + 4 + 6 13
𝑆𝑒𝑚𝑖𝑝𝑒𝑟𝑖𝑚𝑒𝑡𝑒𝑟 = 𝑠 = = = = 6.5
2 2 2

P a g e 55 | 152
Get all the files at: https://bit.ly/azizhandouts
Aziz Manva ([email protected])

Substitute 𝑎 = 3, 𝑏 = 4, 𝑐 = 6 into Heron’s formula (𝐴 = √𝑠 (𝑠 − 𝑎)(𝑠 − 𝑏)(𝑠 − 𝑐)):

13 7 5 1 1 √455
𝐴 = √6.5 (6.5 − 3)(6.5 − 4)(6.5 − 5) = √( ) ( ) ( ) ( ) = √13 × 7 × 5 =
2 2 2 2 4 4

Example 6.24
A triangle with sides 4, 5, 6 and another triangle with height 4 have the same area. What is the length of the
base of the second triangle?

4 + 5 + 6 15
𝑠= = = 7.5
2 2
15 7 5 3 15√7
𝐴 = √7.5(7.5 − 4)(7.5 − 5)(7.5 − 6) = √( )( )( )( ) =
2 2 2 2 4

1 1 15√7 15√7
ℎ𝑏 = 𝐴 ⇒ × 4 × 𝑏 = ⇒ 𝑏=
2 2 4 8

C. Triangle Inequality

6.25: Triangle Inequality


In a triangle with sides 𝑎, 𝑏 and 𝑐:
𝑎 <𝑏+𝑐
𝑏 <𝑎+𝑐
𝑐 <𝑎+𝑏
In a triangle, any side is less than the sum of the other two sides.

We take it as an axiom in geometry that


𝐴 𝑠𝑡𝑟𝑎𝑖𝑔ℎ𝑡 𝑙𝑖𝑛𝑒 𝑖𝑠 𝑡ℎ𝑒 𝑠ℎ𝑜𝑟𝑡𝑒𝑠𝑡 𝑑𝑖𝑠𝑡𝑎𝑛𝑐𝑒 𝑏𝑒𝑡𝑤𝑒𝑒𝑛 𝑡𝑤𝑜 𝑝𝑜𝑖𝑛𝑡𝑠.
And the property above directly follows from the axiom.

Example 6.26
In Δ𝐴𝐵𝐶, 𝐴𝐵 = 4, 𝐵𝐶 = 6 and 𝐴𝐶 = 10.
A. Does this triangle satisfy the triangle inequality?
B. What is the area of Δ𝐴𝐵𝐶?
C. Draw a diagram of Δ𝐴𝐵𝐶? What is the interpretation of area that you get?

Part A
𝐴𝐵 + 𝐵𝐶 = 4 + 6 = 10 = 𝐴𝐶
This violates the triangle inequality.
Part B

𝑎 + 𝑏 + 𝑐 4 + 6 + 10 20
𝑆𝑒𝑚𝑖𝑝𝑒𝑟𝑖𝑚𝑒𝑡𝑒𝑟 = 𝑠 = = = = 10
2 2 2
𝐴 = √10(0)(6)(4) = 0
Part C

P a g e 56 | 152
Get all the files at: https://bit.ly/azizhandouts
Aziz Manva ([email protected])

There is only one interpretation of the data above. The points 𝐴, 𝐵 and 𝐶 are collinear.

6.27: Degenerate Triangles


Triangles with zero area are degenerate triangles.

Example 6.28
A. What is the area of a triangle with 𝐴𝐵 = 4, 𝐵𝐶 = 5 and 𝐴𝐶 = 10?
B. Is it possible to draw a diagram of the “triangle” above? Does that let you interpret the area you got?

6.3 Special Cases


A. Equilateral Triangles

6.29: Heron’s Formula: Equilateral Triangle


If an equilateral triangle has side length 𝑎, then show that:
√3
Δ= × 𝑎2
4

Heron’s Formula for the area of a triangle is:


√𝑠 (𝑠 − 𝑎)(𝑠 − 𝑏)(𝑠 − 𝑐)
In Equilateral Δ𝐴𝐵𝐶:
𝑎=𝑏=𝑐
Hence,
𝑎 + 𝑏 + 𝑐 𝑎 + 𝑎 + 𝑎 3𝑎
𝑠= = =
2 2 2
3𝑎
Substitute, 𝑠 = 2 , 𝑐 = 𝑎, 𝑏 = 𝑎 in Heron’s Formula:

3𝑎 3𝑎 3𝑎 3𝑎
Δ = √( ) ( − 𝑎) ( − 𝑎) ( − 𝑎)
2 2 2 2

Simplify:
3𝑎 𝑎 𝑎 𝑎 𝑎2 √3
= √( ) ( ) ( ) ( ) = √3 = × 𝑎2
2 2 2 2 4 4
Which is what we wanted to prove.
𝑄. 𝐸. 𝐷
Example 6.30
Find the area of an equilateral triangle with side length √3 units.

Example 6.31
If an equilateral triangle has area 36, then find the perimeter of the triangle.

P a g e 57 | 152
Get all the files at: https://bit.ly/azizhandouts
Aziz Manva ([email protected])

6.32: Heron’s Formula: Equilateral Triangle


If an equilateral triangle has side length 𝑎, then show that the height of the
triangle is
√3
ℎ= 𝑎
2
We showed earlier that:
√3
Δ= × 𝑎2
4
Also, we know that:
1 1
Δ = × ℎ𝑏 = × ℎ𝑎
2 2
And since both the formulas have to give the same answer:
1 √3 √3
× ℎ𝑎 = × 𝑎2 ⇒ ℎ = 𝑎
2 4 2
Example 6.33
Find the height of an equilateral triangle with side length 4 units.

Example 6.34
Find the height of an equilateral triangle with area √4 𝑢𝑛𝑖𝑡𝑠.

B. Isosceles Triangle

6.35: Heron’s Formula: Isosceles Triangle


If an isosceles triangle has two equal sides, each with length 𝑎 and base 𝑏, then show that:
𝑏
𝐴𝑟𝑒𝑎 𝑜𝑓 𝑡ℎ𝑒 𝑇𝑟𝑖𝑎𝑛𝑔𝑙𝑒 = Δ = √4𝑎2 − 𝑏 2
4
Heron’s Formula for the area of a triangle is:
√𝑠 (𝑠 − 𝑎)(𝑠 − 𝑏)(𝑠 − 𝑐)
In Isosceles Δ𝐴𝐵𝐶:
𝑐 = 𝑡ℎ𝑖𝑟𝑑 𝑠𝑖𝑑𝑒 = 𝑎
Hence,
𝑎 + 𝑏 + 𝑐 𝑎 + 𝑎 + 𝑏 2𝑎 + 𝑏
𝑠= = =
2 2 2
2𝑎+𝑏
Substitute, 𝑠 = 2 , 𝑐 = 𝑎 in Heron’s Formula:

2𝑎 + 𝑏 2𝑎 + 𝑏 2𝑎 + 𝑏 2𝑎 + 𝑏
Δ = √( )( − 𝑎) ( − 𝑏) ( − 𝑎)
2 2 2 2

P a g e 58 | 152
Get all the files at: https://bit.ly/azizhandouts
Aziz Manva ([email protected])

2𝑎 + 𝑏 𝑏 2𝑎 − 𝑏 𝑏 2𝑎 + 𝑏 2𝑎 − 𝑏 𝑏 2
= √( ) ( )( ) ( ) = √( )( )( )
2 2 2 2 2 2 2
Apply (𝑥 + 𝑦)(𝑥 − 𝑦) = 𝑥 2 − 𝑦 2 inside the square root, and note that we can
move some perfect squares outside the square root:
𝑏
= √4𝑎2 − 𝑏 2
4
Which is what we wanted to prove.
𝑄. 𝐸. 𝐷

Example 6.36
Show that a triangle with sides 5, 5 and 6 is Heronian.

Example 6.37
If an isosceles triangle with area 24 has base 6, then find the perimeter of the triangle.

C. Right-Angled Triangle
We can exploit the symmetry in right-angled triangles

6.38: Heron’s Formula: Right-Angled Triangle


If a right-angled triangle has legs 𝑎 and 𝑏, and hypotenuse 𝑐, then show that:
𝑎+𝑏+𝑐
𝐴𝑟𝑒𝑎 𝑜𝑓 𝑡ℎ𝑒 𝑇𝑟𝑖𝑎𝑛𝑔𝑙𝑒 = 𝑠(𝑠 − 𝑐), 𝑠=
2
𝑎+𝑏+𝑐
Substitute 𝑠 = in 𝑠(𝑠 − 𝑐):
2
𝑎+𝑏+𝑐 𝑎+𝑏+𝑐 (𝑎 + 𝑏) + 𝑐 (𝑎 + 𝑏) − 𝑐
( )( − 𝑐) = ( )( )
2 2 2 2
Carry out the multiplication using (𝑥 + 𝑦)(𝑥 − 𝑦) = 𝑥 2 − 𝑦 2 :
(𝑎 + 𝑏)2 − 𝑐 2 𝑎2 + 2𝑎𝑏 + 𝑏 2 − 𝑐 2
= →𝑰
4 4
Since the triangle is right-angled, so, by the Pythagorean Theorem:
𝑎2 + 𝑏 2 = 𝑐 2 ⇒ 𝑎2 + 𝑏 2 − 𝑐 2 = 0 → 𝑰𝑰
Substitute II in I to get:
2𝑎𝑏 𝑎𝑏
=
4 2
Note that the last expression is the area of a right-angled triangle.
Hence,
𝑎𝑏
𝑠(𝑠 − 𝑐) = = 𝐴𝑟𝑒𝑎 𝑜𝑓 𝑡ℎ𝑒 𝑇𝑟𝑖𝑎𝑛𝑔𝑙𝑒
2
𝑎𝑏 1
Where 2 is the area of a right triangle by substituting ℎ = 𝑎, 𝑏 = 𝑏 in Δ = 2 × ℎ𝑏

Example 6.39
The perimeter of a right-angled triangle is 11. The hypotenuse of the triangle is 5. Find the area of the triangle.

P a g e 59 | 152
Get all the files at: https://bit.ly/azizhandouts
Aziz Manva ([email protected])

𝑝 11
𝑠= = = 5.5
2 2
𝐴 = 𝑠(𝑠 − 𝑐) = 5.5(5.5 − 5) = 5.5(0.5) = 2.75
D. Joining Midpoints

6.40: Area of Triangle formed by joining midpoints of a Triangle


1
Show that the triangle formed by joining the midpoints of the sides of a triangle has area 4 𝑡ℎ the original
triangle.
Note: This property can be proved using similar triangles, but we will do it here using Heron’s Formula. It is
useful to be able to prove a property in multiple ways, showing the connection between different areas of
mathematics.

Area of 𝚫𝑨𝑩𝑪
Consider Δ𝐴𝐵𝐶 with sides lengths 𝑎, 𝑏 𝑎𝑛𝑑 𝑐.

𝑎+𝑏+𝑐
Substituting 𝑠 = 2
in √𝑠(𝑠 − 𝑎)(𝑠 − 𝑏)(𝑠 − 𝑐):

𝑎+𝑏+𝑐 𝑎+𝑏+𝑐 𝑎+𝑏+𝑐 𝑎+𝑏+𝑐


= √( )( − 𝑎) ( − 𝑏) ( − 𝑐)
2 2 2 2

𝑎+𝑏+𝑐 𝑏+𝑐−𝑎 𝑎+𝑐−𝑏 𝑎+𝑏−𝑐


= √( )( )( )( )
2 2 2 2
1
= √(𝑎 + 𝑏 + 𝑐)(𝑏 + 𝑐 − 𝑎)(𝑎 + 𝑐 − 𝑏)(𝑎 + 𝑏 − 𝑐)
4

Area of 𝚫𝐗𝐘𝐙
Let 𝑋, 𝑌 𝑎𝑛𝑑 𝑍 be the midpoints of 𝐴𝐵, 𝐵𝐶 and 𝐶𝐴 respectively.
Then, by the Midpoint Theorem:
𝑎 𝑏 𝑐
𝑋𝑌 = , 𝑋𝑍 = , 𝑌𝑍 =
2 2 2

For Δ𝑋𝑌𝑍:
𝑎 𝑏 𝑐
+ + 𝑎+𝑏+𝑐
𝑠=2 2 2=
2 4
𝑎 𝑏 𝑐
+ + 𝑎+𝑏+𝑐 𝑎 𝑏 𝑐
Substituting 𝑠 = 2 2 2
2
= 4
and the sides as2 , 2 , 2 in √𝑠(𝑠 − 𝑎)(𝑠 − 𝑏)(𝑠 − 𝑐), we get:

𝑎+𝑏+𝑐 𝑎+𝑏+𝑐 𝑎 𝑎+𝑏+𝑐 𝑏 𝑎+𝑏+𝑐 𝑐


= √( )( − )( − )( − )
4 4 2 4 2 4 2

𝑎+𝑏+𝑐 𝑏+𝑐−𝑎 𝑎+𝑐−𝑏 𝑎+𝑏−𝑐


=√ ( )( )( )
4 4 4 4
1
= √(𝑎 + 𝑏 + 𝑐)(𝑏 + 𝑐 − 𝑎)(𝑎 + 𝑐 − 𝑏)(𝑎 + 𝑏 − 𝑐)
16
1 1
= ( √(𝑎 + 𝑏 + 𝑐)(𝑏 + 𝑐 − 𝑎)(𝑎 + 𝑐 − 𝑏)(𝑎 + 𝑏 − 𝑐))
4 4
1
= ([𝐴𝐵𝐶])
4

P a g e 60 | 152
Get all the files at: https://bit.ly/azizhandouts
Aziz Manva ([email protected])

6.4 Applications
A. Area of Quadrilaterals

6.41: Area of General Quadrilateral

Example 6.42
In a quadrilateral-shaped field, the corner connecting the sides with length 20 and 11 has a fence put up going
to the other corner, which has sides of 15 and 14. The length of the fence is 13. Find the difference in areas of
the two parts of the quadrilateral.

B. Quadrilateral with a right angle

Example 6.43
Quadrilateral with a right angle

C. Rhombus

Example 6.44
Find second diagonal given perimeter and first diagonal of rhombus

D. Arithmetic Sequence

Challenge 6.45
A carpenter makes a triangular table, the lengths of whose sides are in Arithmetic Progression. If the area of the
table is 6 sq. ft., then the perimeter of one such table is: (JMET 2011/90)
A. 6 ft
B. 24 ft
C. 12 ft
D. 36 ft
Note: As given in the exam, the question had multiple correct answers. Hence, add the further condition that the
sides of the triangle are integers. This guarantees a single correct answer.

Let the sides of the triangle be:


𝑥 − 𝑑, 𝑥, 𝑥 + 𝑑
Then
3𝑥
𝑝 = (𝑥 − 𝑑) + (𝑥) + (𝑥 + 𝑑) = 3𝑥 ⇒ 𝑠 =
2
3𝑥
Substitute 𝑠 = ,𝑎 = 𝑥 − 𝑑, 𝑏 = 𝑥, 𝑐 = 𝑥 + 𝑑 as the sides in Heron’s Formula 𝐴 = √𝑠 (𝑠 − 𝑎)(𝑠 − 𝑏)(𝑠 − 𝑐):
2

3𝑥 3𝑥 3𝑥 3𝑥
𝐴 = √ ( − 𝑥 − 𝑑) ( − 𝑥) ( − 𝑥 + 𝑑)
2 2 2 2
Simplify:

P a g e 61 | 152
Get all the files at: https://bit.ly/azizhandouts
Aziz Manva ([email protected])

3𝑥 𝑥 𝑥 𝑥 3𝑥 2 𝑥 2 2
3𝑥 2 𝑥 2
𝐴= √ ( − 𝑑) ( ) ( + 𝑑) = √ 2
( −𝑑 )⇒𝐴 = ( − 𝑑2 )
2 2 2 2 4 4 4 4

We also know that


𝐴 = 6 ⇒ 𝐴2 = 36
Hence:
3𝑥 2 𝑥 2 𝑥2
( − 𝑑2 ) = 36 ⇒ 𝑥 2 ( − 𝑑2 ) = 48
4 4 4
Substitute 𝑝 = 12 ⇒ 𝑥 = 4
𝑥2 16
𝑥 2 ( − 𝑑2 ) = 16 ( − 𝑑2 ) = 16(4 − 𝑑2 ) = 48 ⇒ 4 − 𝑑2 = 3 ⇒ 𝑑2 = 1 ⇒ 𝑑 = 1
4 4
𝑥−𝑑 =3
𝑥=4
𝑥+𝑑 =5
Hence, the sides of the triangle are:
3,4,5 ⇒ 𝑝 = 3 + 4 + 5 = 12
If you try the other options, you will not get valid answers, or you will not get integer answers.

Challenge 6.46 (Question Repeated, Shortcut Method)


A carpenter makes a triangular table, the lengths of whose sides are in Arithmetic Progression. If the area of the
table is 6 sq. ft., then the perimeter of one such table is: (JMET 2011/90)
A. 6 ft
B. 24 ft
C. 12 ft
D. 36 ft
Note: As given in the exam, the question had multiple correct answers. Hence, add the further condition that the
sides of the triangle are integers. This guarantees a single correct answer.

1
𝐴 = 6 ⇒ ℎ𝑏 = 6 ⇒ ℎ𝑏 = 12
2
If the base is an integer, height also has to be an integer.
Hence, the only possible values for the base are:
12 = 1 × 12 = 2 × 6 = 3 × 4
If in the above, you recognize that
3,4,5 𝑖𝑠 𝑎 𝑃𝑦𝑡ℎ𝑎𝑔𝑜𝑟𝑒𝑎𝑛 𝑇𝑟𝑖𝑝𝑙𝑒𝑡 ⇒ 𝑝 = 3 + 4 + 5 = 12 ⇒ 𝑶𝒑𝒕𝒊𝒐𝒏 𝑪

6.5 Generalizations
A. Area of Cyclic Quadrilaterals (Brahmagupta’s Formula)

6.47: Brahmagupta’s Formula: Area of a Cyclic Quadrilateral


If a cyclic quadrilateral has sides 𝑎, 𝑏, 𝑐 and 𝑑, then the area of the quadrilateral is given by
𝑎+𝑏+𝑐+𝑑
√(𝑠 − 𝑎)(𝑠 − 𝑏)(𝑠 − 𝑐)(𝑠 − 𝑑), 𝑠=
2

P a g e 62 | 152
Get all the files at: https://bit.ly/azizhandouts
Aziz Manva ([email protected])

PART III: RIGHT TRIANGLES


7. PYTHAGOREAN THEOREM
7.1 Basics
A. Statement

7.1: Right Angled Triangle: Terminology


➢ The side opposite the right angle in a right-angled triangle is called the hypotenuse. Hypotenuse is the
longest side.
➢ The two sides other than the hypotenuse in a right-angled triangle
are called the legs.

7.2: Pythagorean Theorem: Algebraic


𝒂𝟐 + 𝒃𝟐 = 𝒄𝟐
In a right-angled triangle, the sum of the squares of the lengths of the legs
is equal to the square of the length of the hypotenuse.

Example 7.3
3, 4 and 5 are the sides of a right-angled triangle. Verify that the Pythagorean Theorem holds.

32 + 42 = 9 + 16 = 25
52 = 25
B. Visual Interpretation of the Pythagorean Theorem

7.4: Pythagoras Theorem: Visual


The Pythagorean Theorem can be illustrated with a geometrical
interpretation.

If you draw a square on side


➢ AB (which has length 3), it will have area 3 × 3 = 9
➢ BC (which has length 4), it will have area 4 × 4 = 16
➢ AC (which has length 5), it will have area 5 × 5 = 25

Hence,
𝐴𝑟𝑒𝑎 𝑜𝑓 𝑇𝑤𝑜 𝐺𝑟𝑒𝑒𝑛 𝑆𝑞𝑢𝑎𝑟𝑒𝑠 = 𝐴𝑟𝑒𝑎 𝑜𝑓 𝐵𝑙𝑢𝑒 𝑆𝑞𝑢𝑎𝑟𝑒

Example 7.5
Right isosceles triangles are constructed on the sides of a 3-4-5 right
triangle, as shown. A capital letter represents the area of each triangle.
Which one of the following is true? (AMC 8 2002/16)
(A) 𝑋 + 𝑍 = 𝑊 + 𝑌
(B) 𝑊 + 𝑋 = 𝑍
(C) 3𝑋 + 4𝑌 = 5𝑍
1
(D) 𝑋 + 𝑊 = (𝑌 + 𝑍)
2
(E) 𝑋 + 𝑌 = 𝑍

P a g e 63 | 152
Get all the files at: https://bit.ly/azizhandouts
Aziz Manva ([email protected])

Method I:
From the visual representation in the previous property, we know that:
9 + 16 = 25

If we divide each square into two, we will get two right isosceles triangles,
each with area half of the original square, and hence:
9 16 25
+ =
2 2 2

Method II:
1 1 25
𝑍 = ℎ𝑏 = × 5 × 5 =
2 2 2
1 1 9
𝑋 = ℎ𝑏 = × 3 × 3 =
2 2 2
1 1 16
𝑌 = ℎ𝑏 = × 4 × 4 =
2 2 2
Hence,
𝑋 + 𝑌 = 𝑍 ⇒ 𝑂𝑝𝑡𝑖𝑜𝑛 𝐸
C. Finding the Hypotenuse

Example 7.6
The legs of a right-angled triangle have length three and four, respectively. Find the
A. Length of the hypotenuse.
B. Perimeter of the triangle

Substitute 𝑎 = 3, 𝑏 = 4:
𝑐 2 = 𝑎2 + 𝑏 2 = 32 + 42 = 9 + 16 = 25 ⇒ 𝑐 = 5
𝑃 = 𝑎 + 𝑏 + 𝑐 = 3 + 4 + 5 = 12

Example 7.7
If a right-angled triangle has legs of length four and five, what is the length of the
hypotenuse?

Substitute 𝑎 = 4, 𝑏 = 5:
𝑐 2 = 𝑎2 + 𝑏 2 = 42 + 52 = 16 + 25 = 41
𝑐 2 = 41
Take square roots:
𝑐 = √41

Example 7.8
A garden is in the shape of a triangle with a first fence perpendicular to its second fence. If the first fence is 5
meters long, and the second fence is 6 meters long, then find the cost, in dollars, of fencing the garden at 40
cents per meter.

𝑇ℎ𝑖𝑟𝑑 𝑆𝑖𝑑𝑒 = √52 + 62 = √25 + 36 = √61


𝑃𝑒𝑟𝑖𝑚𝑒𝑡𝑒𝑟 = 5 + 6 + √61 = 11 + √61
𝐶𝑜𝑠𝑡 𝑜𝑓 𝑓𝑒𝑛𝑐𝑒 = 0.4(11 + √61) = 4.4 + 0.4√61 𝑑𝑜𝑙𝑙𝑎𝑟𝑠

P a g e 64 | 152
Get all the files at: https://bit.ly/azizhandouts
Aziz Manva ([email protected])

7.9: Hypotenuse
The hypotenuse is the longest side of a right-angled triangle.

Example 7.10
𝐹𝑜𝑟 𝑡ℎ𝑖𝑠 𝑞𝑢𝑒𝑠𝑡𝑖𝑜𝑛, 𝑦𝑜𝑢 𝑐𝑎𝑛 𝑢𝑠𝑒 𝑖𝑛𝑓𝑜𝑟𝑚𝑎𝑡𝑖𝑜𝑛 𝑓𝑟𝑜𝑚 𝑒𝑎𝑐ℎ 𝑝𝑎𝑟𝑡 𝑖𝑛 𝑠𝑢𝑏𝑠𝑒𝑞𝑢𝑒𝑛𝑡 𝑝𝑎𝑟𝑡𝑠.
A park is in the shape of a right-angled triangle. The shortest side of the park is 3 meters. The second shortest
side is 5 meters. Abhay is driving his favorite tricycle from point 𝐴 to point 𝐵 (both are in the park, or on the
sides) on a straight path.
A. Find the maximum possible distance between A and B.
𝑚
B. If Abhay drives his tricycle at a speed of 0.5 𝑠 , find the time required.
C. If Abhay wants to take twenty seconds to travel the distance, what is the speed that he should travel at?

See diagram with a right-angled triangle.


𝑀𝑎𝑥 𝐷𝑖𝑠𝑡𝑎𝑛𝑐𝑒 = 𝐻𝑦𝑝𝑜𝑡𝑒𝑛𝑢𝑠𝑒 = √32 + 52 = √9 + 25 = √34

𝐷 √34 √34 1
𝑇= = = = √34 ÷ = √34 × 2 = 2√34
𝑆 0.5 1 2
2
D. Finding a Leg

Example 7.11
The longest side of a right-angled triangle has length 17. The shortest side has length 8. Find the length of the
third side.

Substitute 𝑎 = 8, 𝑐 = 17 in:
𝑎2 + 𝑏 2 = 𝑐 2 ⇒ 82 + 𝑏 2 = 172 ⇒ 64 + 𝑏 2 = 289 ⇒ 𝑏 2 = 225 ⇒ 𝑏 = 15

Example 7.12
Find the length of the other leg of a right-angled triangle, if the length of the hypotenuse is
9, and one leg is 5.

𝑎2 + 𝑏 2 = 𝑐 2 ⇒ 52 + 𝑏 2 = 92 ⇒ 𝑏 2 = 81 − 25 = 56 ⇒ 𝑏 = √56 = √14 × √4 = 2√14

E. Isosceles Triangles

Example 7.13
A. The leg of an isosceles right-angled triangle is 3 units. Find the length of the hypotenuse?
B. In an isosceles right-angled triangle, the shortest side is 10 units. What is the length of the longest side?
C. In an isosceles right-angled triangle, the shortest side is 4√2 units. What is the perimeter of the
triangle?
D. If the hypotenuse of an isosceles right-angled triangle is 10, what are the lengths of its sides?

Part A
𝑐 2 = 𝑎2 + 𝑎2 = 32 + 32 = 9 + 9 = 18 ⇒ 𝑐 = √18 = 3√2
Part B
𝑐 2 = 𝑎2 + 𝑎2 = 102 + 102 = 100 + 100 = 200 ⇒ 𝑐 = √200 = 10√2

P a g e 65 | 152
Get all the files at: https://bit.ly/azizhandouts
Aziz Manva ([email protected])

Part C
Use the Pythagorean Theorem:
2 2
𝑐 2 = 𝑎2 + 𝑎2 = (4√2) + (4√2) = 32 + 32 = 64 ⇒ 𝑐 = √64 = 8
And hence the perimeter is:
𝑃 = 4√2 + 4√2 + 8 = 8√2 + 8

Part D
𝑎2 + 𝑎2 = 102 ⇒ 2𝑎2 = 100 ⇒ 𝑎2 = 50 ⇒ 𝑎 = √50 = √25 × √2 = 5√2
F. Converse of Pythagorean Theorem

7.14: Converse of Pythagorean Theorem


If in a triangle with side lengths 𝑎, 𝑏, 𝑐:
𝑎2 + 𝑏 2 = 𝑐 2
then the triangle is a right-angled triangle.

The converse of Pythagoras Theorem lets us check whether a triangle is right-angled based on its sides.

Example 7.15
Check whether the lengths below are the sides of a right-angled triangle.
A. 5,6, and 7
B. 5, 11, 12
C. 8, 15, 17

Part A
52 + 62 = 25 + 36 = 61
2
7 = 49 ≠ 61 ⇒ 𝑁𝑜𝑡 𝑟𝑖𝑔ℎ𝑡 − 𝑎𝑛𝑔𝑙𝑒𝑑

Part B
No

Part C
Yes

Example 7.16
A triangle with sides 6, 8 𝑎𝑛𝑑 10 has one angle 30°. Find the difference between the other two angles.

Note that:
(6,8,10) = 2(3,4,5) ⇒ 𝑃𝑦𝑡ℎ𝑎𝑔𝑜𝑟𝑒𝑎𝑛 𝑇𝑟𝑖𝑝𝑙𝑒𝑡 ⇒ 𝑇𝑟𝑖𝑎𝑛𝑔𝑙𝑒 𝑖𝑠 𝑟𝑖𝑔ℎ𝑡 − 𝑎𝑛𝑔𝑙𝑒𝑑

Hence,
𝑂𝑛𝑒 𝑎𝑛𝑔𝑙𝑒 𝑖𝑛 𝑡ℎ𝑒 𝑡𝑟𝑖𝑎𝑛𝑔𝑙𝑒 𝑖𝑠 90°
𝑆𝑒𝑐𝑜𝑛𝑑 𝑎𝑛𝑔𝑙𝑒 𝑖𝑠 𝑔𝑖𝑣𝑒𝑛 𝑡𝑜 𝑏𝑒 30°
∴ 3𝑟𝑑 𝐴𝑛𝑔𝑙𝑒 = 180 − 90 − 30 = 60°
𝐷𝑖𝑓𝑓𝑒𝑟𝑒𝑛𝑐𝑒 𝑜𝑓 𝑡𝑤𝑜 𝑎𝑛𝑔𝑙𝑒𝑠 = 60 − 30 = 30°

Example 7.17
Triangle 𝐴𝐵𝐶 has sides of length 5, 12 and 13 units, and triangle 𝐷𝐸𝐹 has sides of length 8, 15 and 17 units.

P a g e 66 | 152
Get all the files at: https://bit.ly/azizhandouts
Aziz Manva ([email protected])

What is the ratio of the area of triangle 𝐴𝐵𝐶 to the area of triangle 𝐷𝐸𝐹? Express your answer as a common
fraction. (MathCounts 2005 State Countdown)

By the converse of Pythagorean Theorem:


𝐵𝑜𝑡ℎ 𝑡𝑟𝑖𝑎𝑛𝑔𝑙𝑒𝑠 𝑎𝑟𝑒 𝑟𝑖𝑔ℎ𝑡 𝑎𝑛𝑔𝑙𝑒𝑠.
Hence,
1
× 5 × 12 1
𝑅𝑎𝑡𝑖𝑜 𝑜𝑓 𝐴𝑟𝑒𝑎𝑠 = 2 =
1
× 8 × 15 2
2

Example 7.18
In the non-convex quadrilateral 𝐴𝐵𝐶𝐷 shown below, ∠𝐵𝐶𝐷 is a right angle, 𝐴𝐵 = 12,
𝐵𝐶 = 4, 𝐶𝐷 = 3, and 𝐴𝐷 = 13. What is the area of quadrilateral 𝐴𝐵𝐶𝐷? (AMC 8 2017
/18)

G. Pythagorean Triplets

7.19: Pythagorean Triplet


Sets of three integers that form the sides of a right-angled triangle are called Pythagorean Triplets.

Pythagorean Triplets are very useful to know and remember because they will often reduce your calculation
time.

7.20: Primitive Pythagorean Triplet


If the numbers in a Pythagorean Triplet have no common factor, then the Triplet is called primitive.

Example 7.21 Primitive Pythagorean Triplets


The table shows the 16 primitive Pythagorean Triplets 3, 4, 5 20, 21, 29 11, 60, 61 36, 77, 85
with integers less than 100. 5, 12, 13 12, 35, 37 33, 56, 65 13, 84, 85
A. Memorize the shaded triplets 8, 15, 17 9, 40, 41 16, 63, 65 39, 80, 89
B. Verify the shaded triplets. 7, 24, 25 28, 45, 53 48, 55, 73 65, 72, 97

32 + 42 = 9 + 16 = 25 = 52
52 + 122 = 25 + 144 = 169 = 132
82 + 152 = 64 + 225 = 289 = 172
72 + 242 = 49 + 576 = 625 = 252

7.22: Multiple of a Pythagorean Triplet2


Multiplying a Pythagorean triplet by a positive natural number also results in a Pythagorean Triplet:

(3, 4, 5) × 2 = (6, 8, 10)

Example 7.23
From the table above, multiply each of the four shaded Pythagorean Triplets by 2, 3 and 4 to get the
corresponding Pythagorean Triplets.

2
This video shows this property in action.

P a g e 67 | 152
Get all the files at: https://bit.ly/azizhandouts
Aziz Manva ([email protected])

×2 ×3 ×4
3, 4, 5 6,8,10 9,12,15 12,16,20
5, 12, 13 10,24,25 15,36,39 20,48,52
8, 15, 17 16,30,34 24,45,51
7, 24, 25

Example 7.24
“Simplifying” a Pythagorean Triplet is possible by dividing each number in the Triplet by their HCF to get a
smaller number. Simplify the following triplets
A. 30,40,50
B. 60,63,87
Part A
Divide each number by 𝐻𝐶𝐹(30,40,50) = 10 to get:
(3,4,5)
Part B
Divide each number by 𝐻𝐶𝐹(60,63,87) = 3 to get:
(20,21,29)

7.2 Multiple and Nested Triangles


Example 7.25
In the diagram, what is the perimeter of Δ𝑃𝑄𝑅? (CEMC Pascal 2008)

𝑥 𝑥
(25,20, 𝑥) = 5 (5,4, ) ⇒ = 3 ⇒ 𝑥 = 15
5 5
(8,15, 𝑦) ⇒ 𝑦 = 17

Example 7.26
Triangles BAD and BDC are right triangles with AB=12 units, BD=15 units, and
BC=17 units. What is the area, in square units, of quadrilateral ABCD? (MathCounts
2005 State Countdown)

Triangle BAD:
𝑥 𝑥 1 1
(𝑥, 12,15) = 3 ( , 4,5) ⇒ = 3 ⇒ 𝑥 = 9 ⇒ Δ = ℎ𝑏 = × 9 × 12 = 54
3 3 2 2

Triangle BDC:
Note that we have a Pythagorean Triple again:
1 1
(𝑦, 15,17) = 𝑦 = 8 ⇒ Δ = ℎ𝑏 = × 8 × 15 = 60
2 2
[𝐴𝐵𝐶𝐷] = [𝐵𝐴𝐷] + [𝐵𝐷𝐶] = 54 + 60 = 114

Example 7.27
As shown, a square is constructed on the hypotenuse of a right triangle whose
legs have lengths 3 units and 4 units. What is the area of the pentagon ABCDE, in
square units? (MathCounts 2005 Chapter Countdown)

P a g e 68 | 152
Get all the files at: https://bit.ly/azizhandouts
Aziz Manva ([email protected])

(3,4, 𝐸𝐶) ⇒ 𝐸𝐶 = 5 ⇒ 𝐸𝐶 2 = 25
1 1
[𝐸𝐷𝐶] = ℎ𝑏 = × 3 × 4 = 6
2 2
𝑇𝑜𝑡𝑎𝑙 = 25 + 6 = 31

Example 7.28
In any isosceles triangle 𝐴𝐵𝐶 with 𝐴𝐵 = 𝐴𝐶, the altitude AD bisects the base BC so that
𝐵𝐷 = 𝐷𝐶. As shown in Δ𝐴𝐵𝐶, 𝐴𝐵 = 𝐴𝐶 = 25 and 𝐵𝐶 = 14. Determine the length of the
altitude AD. (CEMC 2011 Fryer)

𝐵𝐷 = 𝐷𝐶 = 7
(7, 𝑥, 25) ⇒ 𝑥 = 24

7.3 Ladders and Ramps


A. Ladders
Applied questions on Pythagorean Theorem
often introduce real life scenarios that require
drawing a diagram. One such popular scenario
is a ladder next to a wall.
In the diagram alongside
➢ the wall is 90° with the ground, creating
a right-angled triangle
➢ The window is a vertex of the triangle
and gives extra flavor to the question.
➢ the length of the ladder gives the length
of the hypotenuse
➢ the height of the wall is one length of a leg of the triangle
➢ the distance between the foot of the ladder, and the foot of the wall is another leg of the triangle.

Example 7.29
A ladder of length 5 meters is inclined to climb up to a church window that is 4 meters above the ground. Find
the distance between the foot of the ladder and the church wall.

Refer diagram.
We see a Pythagorean Triplet
(3,4,5) ⇒ 𝑅𝑒𝑞𝑢𝑖𝑟𝑒𝑑 𝐷𝑖𝑠𝑡𝑎𝑛𝑐𝑒 = 3

Repeat the above question if the church window is 2 meters above the ground.

Refer Diagram
22 + 𝑑2 = 52 ⇒ 𝑑2 = 25 − 4 = 21 ⇒ 𝑑 = √21

𝐶𝑟𝑖𝑡𝑖𝑐𝑎𝑙 𝑇ℎ𝑖𝑛𝑘𝑖𝑛𝑔: In the first question, if the ladder is 4 meters, and the church
window is 5 meters high, what can you say?

P a g e 69 | 152
Get all the files at: https://bit.ly/azizhandouts
Aziz Manva ([email protected])

The hypotenuse should be the longest side in a right-angled triangle. However, that
is not the case in the diagram alongside.

Hence, the data given is inconsistent, or logically impossible.

Example 7.30
An 8.5-meter ladder is leaning against a vertical wall. How many meters is its base from the wall if the ladder
reaches 7.5 meters up the wall? (MathCounts 2007 State Countdown)

1
(𝑑, 7.5,8.5) = (2𝑑, 15,17) ⇒ 2𝑑 = 8 ⇒ 𝑑 = 4
2

Example 7.31
A ladder in the street is inclined to climb up to a house window that is 8 feet above the ground. The distance
from the foot of the ladder to the foot of the wall is 15 feet. Find the length of the ladder.

Repeat the above question if the window is 7 feet above the ground.

Example 7.32
A ladder is placed in the street to access a red house 15 meters away via a blue window that is 8 meters high.
The ladder is then turned on the same point to access a blue house that is on the other side of the street via a
red window. Find the height of the red window, if the width of the street is 22 meters.

√240 = 4√15

Repeat the above question if the red house is 13 meters away.

√152 = 2√38

B. Ramps
A ramp is an incline. People in wheelchairs cannot easily
navigate stairs, but find ramps easier to use. Ramps can
also be used to transport goods up an incline into a
warehouse, etc.
When drawing a diagram with a ramp, remember that we
can create a right-angle at the foot of where the ramp connects with the building:
➢ The length of the ramp will be the hypotenuse.

Example 7.33
A ramp of length 13 meters travels a horizontal distance of 12 meters.
A. Find the vertical distance it travels.

P a g e 70 | 152
Get all the files at: https://bit.ly/azizhandouts
Aziz Manva ([email protected])

B. Find the vertical distance travelled per unit of horizontal distance travelled.

5
5
12

Repeat Parts A. and B. if the ramp travels a horizontal distance of 11 meters.

√48 = 4√3
4√3
11

7.4 Flagpoles and Flying Animals


A. Flagpoles
Flagpoles are more interesting (and complex) compared to
ladders, and ramps. This is because:
➢ Flagpoles cast shadows, which are often shorter or
longer than the actual flagpole (creating similar
triangles).
➢ If flagpoles snap or break, then we are essentially
given the length of the flagpole, but which also
represents the sum of two sides of a triangle.

Example 7.34
The bottoms of two vertical poles are 12 feet apart and are on a region of flat ground. One pole is 6 feet tall and
the other is 15 feet tall. How long, in feet, is a wire stretched from the top of one pole to the top of the other
pole? (MathCounts 2010 School Countdown)

The sides are


ℎ ℎ
(9,12, ℎ) = 3 (3,4, ) ⇒ = 5 ⇒ ℎ = 15
3 3

Example 7.35
In heavy winds, a flagpole snaps 8 feet above the ground to touches the ground 15 feet away from the base of
the flagpole. What is the length of the flagpole?

8 + 17 = 25

Example 7.36
A flagpole is originally 5 meters tall. A hurricane snaps the flagpole at a point 𝑥 meters above the ground so that
the upper part, still attached to the stump, touches the ground 1 meter away from the base. What is 𝑥? (AMC

P a g e 71 | 152
Get all the files at: https://bit.ly/azizhandouts
Aziz Manva ([email protected])

10B 2009/10)

(5 − 𝑥)2 = 𝑥 2 + 1
25 − 10𝑥 + 𝑥 2 = 𝑥 2 + 1
24 = 10𝑥
𝑥 = 2.4

B. Flying Animals

Example 7.37
A hummingbird on the ground spots a flower and flies 2 yards in a straight line at an angle of 43° to reach the
nectar. After drinking the nectar, the hummingbird notes that the height of the flower is 5 feet above the
ground. Find the distance, in feet, between the point at which the hummingbird originally was, and the point on
the ground exactly above the flower. (1 𝑦𝑎𝑟𝑑 = 3 𝑓𝑒𝑒𝑡)

Consider Δ𝐹𝐺𝐻 with


𝐻 = 𝑂𝑟𝑖𝑔𝑖𝑛𝑎𝑙 𝐿𝑜𝑐𝑎𝑡𝑖𝑜𝑛 𝑜𝑓 𝐻𝑢𝑚𝑚𝑖𝑛𝑔𝑏𝑖𝑟𝑑
𝐹 = 𝐹𝑙𝑜𝑤𝑒𝑟
𝐺 = 𝑃𝑜𝑖𝑛𝑡 𝑜𝑛 𝐺𝑟𝑜𝑢𝑛𝑑 𝑑𝑖𝑟𝑒𝑐𝑡𝑙𝑦 𝑎𝑏𝑜𝑣𝑒 𝑓𝑙𝑜𝑤𝑒𝑟

Substitute 𝐹𝐻 = 6 𝑓𝑒𝑒𝑡, 𝐹𝐺 = 5 𝑓𝑒𝑒𝑡 in 𝐹𝐺 2 + 𝐺𝐻 2 = 𝐹𝐻 2 :


52 + 𝐺𝐻 2 = 62 ⇒ 𝐺𝐻 2 = 11 ⇒ 𝐺𝐻 = √11

7.5 Directions
A. Compass
Questions that involve directions require use of the Pythagorean Theorem,
particularly when the people travelling make turns that are 90°.
For this, it is important to know and be able to draw the compass directions,
which are given alongside.
In particular:
➢ The four main directions are North, East, South and West.
There are also four ancillary directions:
➢ Exactly in between North and West lies North-West (𝑁𝑊)
➢ Exactly in between North and East lies North-East (𝑁𝐸)
➢ Exactly in between South and West lies South-West (𝑆𝑊)
➢ Exactly in between South and East lies South-East (𝑆𝐸)

Example 7.38
A deer hunter travelled due north for 3 km, and then due west for 5 km.
A. What was his distance from his starting point?
B. What was his (approximate) compass direction from starting point.

P a g e 72 | 152
Get all the files at: https://bit.ly/azizhandouts
Aziz Manva ([email protected])

𝐷𝑖𝑠𝑡𝑎𝑛𝑐𝑒 = √32 + 52 = √9 + 25 = √34

𝐷𝑖𝑟𝑒𝑐𝑡𝑖𝑜𝑛 = 𝑁𝑜𝑟𝑡ℎ − 𝑊𝑒𝑠𝑡

Example 7.39
A deer hunter travelled due north for 12 km, then due east for 15 km, and then due south for 3 km. What was
his distance from his starting point?

𝐷 = √152 + 92 = √225 + 81 = √306 = 3√34

Example 7.40
A travelled 5 km North-North-West, and then 12 km East-North-East. What is his distance from his starting
point?

Apply Pythagoras (North-North-West is 90 degrees to the left of East-North-East):


Root (52 + 122) = Root (25 + 144) = Root (169) = 13 km.

Example 7.41
Aaron’s school is north-west from his house, 5 km away. Aaron’s library is south-west from his house, 12 km
away. Aaron’s math club is 3 km north-east from his house.
A. Draw a neatly labelled diagram showing the positions of Aaron’s house, school, library and math club.
B. What is the straight-line distance between Aaron’s school and his library?
C. Aaron drives in a straight line from his library to his math club. Then he remembers he forgot to issue
the book that he wanted. So, he drives back. What is the distance that he drives?
D. A bird flies from Aaron’s school to his math club. What is the distance that it flies?
E. Aaron drives from school to home, has an evening snack, and then goes to the math club. How much
more (or less) does he travel compared to the bird.

13
30
√34
8 − √34

Example 7.42
1 3 1
Bill walks 2 mile south, then 4 mile east, and finally 2 mile south. How many miles is he, in a direct line, from his
starting point? (AMC 8 2005/7)

P a g e 73 | 152
Get all the files at: https://bit.ly/azizhandouts
Aziz Manva ([email protected])

Bill makes the set of movements on the left diagram.


This is equivalent to the set of movements on the right
diagram, giving us a right-angled triangle.
Hence, by the Pythagorean Theorem:

3 2 9 25 5
𝑑 = √12 + ( ) = √1 + =√ =
4 16 16 4

Example 7.43
While walking on a plane surface, a traveler first headed 18 miles north, then 11 miles west, then 6 miles south
and finally 6 miles east. How many miles from the starting point was the traveler after these four legs of the
journey? (MathCounts 1996 Warm-Up 8)

The diagram on the left shows the movements of the traveler.


(5,12, 𝑥) ⇒ 𝑥 = 13

Example 7.44
Minneapolis-St. Paul International Airport is 8 miles southwest of downtown St. Paul and 10 miles southeast of
downtown Minneapolis. Which of the following is closest to the number of miles between downtown St. Paul
and downtown Minneapolis? (AMC 10B 2004/8)
(A) 13 (B) 14 (C) 15 (D) 16 (E) 17

Example 7.45
Point B is due east of point A. Point C is due north of point B. The distance between points A and C is 10√2, and
∠𝐵𝐴𝐶 = 45∘ . Point D is 20 meters due north of point C. The distance AD is between which two integers? (AMC
10B 2012/12)

P a g e 74 | 152
Get all the files at: https://bit.ly/azizhandouts
Aziz Manva ([email protected])

B. Boats

Example 7.46

7.6 Review and Challenge

P a g e 75 | 152
Get all the files at: https://bit.ly/azizhandouts
Aziz Manva ([email protected])

8. USING PYTHAGORAS THEOREM


A. Background
We have already seen how Pythagoras Theorem is useful in many situations involving triangles. However,
many geometrical figures consist of triangles, or can be broken into triangles.
We now look at some applications of Pythagorean Theorem in figures other than triangles.

8.1 Algebraic Applications


A. Triangle Inequality

Example 8.1
For what value of 𝑎 is there a right triangle with sides a+1, 6a, and 6a+1? (MathCounts 2003 National Sprint)

6𝑎 > 0 ⇒ 𝑎 > 0
𝑎 > 0 ⇒ 6𝑎 + 1 > 𝑎 + 1
Hence, the hypotenuse must be
6𝑎 + 1

(𝑎 + 1)2 + (6𝑎)2 = (6𝑎 + 1)2


𝑎2 + 2𝑎 + 1 + 36𝑎2 = 36𝑎2 + 12𝑎 + 1
𝑎2 − 10𝑎 = 0 ⇒ 𝑎 ∈ {0,10} ⇒ 𝑎 = 10
B. Linear Equations

Example 8.2
Twenty-nine is the shortest leg of a right triangle whose other leg and hypotenuse are consecutive whole
numbers. What is the sum of the lengths of the other two sides? (MathCounts 2009 Workout 2)

Let
𝐿𝑒𝑛𝑔𝑡ℎ 𝑜𝑓 𝑙𝑜𝑛𝑔𝑒𝑟 𝑙𝑒𝑔 = 𝑛 ⇒ 𝐿𝑒𝑛𝑔𝑡ℎ 𝑜𝑓 ℎ𝑦𝑝𝑜𝑡𝑒𝑛𝑢𝑠𝑒 = 𝑛 + 1 ⇒ 𝑆𝑢𝑚 𝑜𝑓 𝑡ℎ𝑒 𝑡𝑤𝑜 = 2𝑛 + 1
By the Pythagorean Theorem:
292 + 𝑛2 = (𝑛 + 1)2
292 + 𝑛2 = 𝑛2 + 2𝑛 + 1
841 = 2𝑛 + 1
C. Simultaneous Equations

Example 8.3
Let △ 𝑋𝑂𝑌 be a right-angled triangle with m∠XOY = 900 Let 𝑀 and 𝑁 be the midpoints of the legs 𝑂𝑋 and 𝑂𝑌,
respectively. Given 𝑋𝑁 = 19 and 𝑌𝑀 = 22, find 𝑋𝑌. (AMC 10B 2002/22)

𝑋𝑌 = √(2𝑎)2 + (2𝑏)2 = √4𝑎2 + 4𝑏 2 = √4(𝑎2 + 𝑏 2 ) = 2√𝑎2 + 𝑏 2

In right Δ𝑋𝑂𝑁, by Pythagorean Theorem:


(2𝑎)2 + 𝑏 2 = 192 ⇒ 4𝑎2 + 𝑏 2 = 361
𝑎2 + (2𝑏)2 = 222 ⇒ 𝑎2 + 4𝑏 2 = 484
Add Equations I and II:

P a g e 76 | 152
Get all the files at: https://bit.ly/azizhandouts
Aziz Manva ([email protected])

5𝑎2 + 5𝑏 2 = 845 ⇒ 𝑎2 + 𝑏 2 = 169 ⇒ 2√𝑎2 + 𝑏 2 = 26

8.2 Geometrical Applications: 2D


A. Isosceles Triangles: Area
Isosceles triangles have the property that the median, angle bisector and the altitude are all the same. We will
use this to find the area of an Isosceles Triangle.

Example 8.4
A tent with a triangular cross-section (when taken perpendicular to the ground) has sloping sides of length 6
units, and a width of 8 units. Find the height of the central pole of the tent.

Substitute 𝑎 = 4, 𝑐 = 6 in 𝑎2 + 𝑏 2 = 𝑐 2 :
42 + 𝑏 2 = 62
2
Solve for 𝑏 :
𝑏 2 = 36 − 16 = 20
Simplify the radical:
𝑏 = √20 = 2√5

Example 8.5
A house has a triangular roof. The sloping side of the roof has a length of side 5. The base of the house has a
length of 20, and a width of 6. The height from the base of the house to the base of the roof is 11 feet. Take a
cross-section of the house perpendicular to the base of the house. Find the area of:
A. Cross section of the house (not including the roof).
B. Cross section of the roof

Drawing a diagram
Here drawing a diagram is very important. The base of
the roof cannot be of length 20 since
5 + 5 = 10 < 20
Which violates the triangle inequality.

Hence, draw the diagram so that the base of the roof


has a length of
6
(Invalid diagram of cross-section shown on the right,
valid on the left)

Now the roof gives an isosceles triangle with sides:


5−5−6

Part A
𝐴𝑟𝑒𝑎 = 11 × 6 = 66

Part B
Drop a perpendicular from the vertex of the triangle.
The base of the triangle gets divided into two equal parts, each with length 3.

P a g e 77 | 152
Get all the files at: https://bit.ly/azizhandouts
Aziz Manva ([email protected])

Recognize a Pythagorean Triplet:


3−4−5
Hence, the area of the triangular cross-section is
1 1
ℎ𝑏 = × 4 × 6 = 12
2 2

8.6: Area of Isosceles Triangle


If an isosceles triangle has two equal sides 𝑎 and base 𝑏, then show that:
𝑏
𝐴𝑟𝑒𝑎 𝑜𝑓 𝑡ℎ𝑒 𝑇𝑟𝑖𝑎𝑛𝑔𝑙𝑒 = Δ = √4𝑎2 − 𝑏 2
4
Note: Do 𝑛𝑜𝑡 memorize this. Instead, apply the Pythagorean Theorem every time.

Draw a diagram.
Drop a perpendicular from Vertex 𝐵, intersecting base 𝐴𝐶 at 𝐷.

Since Δ𝐴𝐵𝐶 is isosceles, the altitude to the base is also the median, and hence:
𝑏
𝐴𝐷 = 𝐷𝐶 =
2
Also, let
𝑙𝑒𝑛𝑔𝑡ℎ 𝑜𝑓 𝐵𝐷 = ℎ

Then, in right Δ𝐵𝐴𝐷, by the Pythagorean Theorem:


𝑏 2
𝑎2 = ℎ2 + ( )
2
Solve for ℎ2 :
𝑏 2 4𝑎2 − 𝑏 2
ℎ2 = 𝑎2 − =
4 4
Take square roots both sides:
4𝑎2 − 𝑏 2 1
ℎ=√ = √4𝑎2 − 𝑏 2
4 2

1
Substitute ℎ = 2 √4𝑎2 − 𝑏 2 and 𝑏 = 𝑏 in the formula for area of a triangle to get:
1 1 1 𝑏
[𝐴𝐵𝐶] = × ℎ𝑏 = × ( √4𝑎2 − 𝑏 2 ) × 𝑏 = √4𝑎2 − 𝑏 2
2 2 2 4

Which was the equality to be proved.


𝑄. 𝐸. 𝐷

B. Isosceles Triangles: Perimeter

Example 8.7

C. Square

P a g e 78 | 152
Get all the files at: https://bit.ly/azizhandouts
Aziz Manva ([email protected])

Proof 8.8: Diagonal of a square


Show that a square with side length 𝑠 has diagonals with length √2𝑠

Note that, in square 𝐴𝐵𝐶𝐷, with side length 𝑠,


Δ𝐴𝐷𝐶 𝑖𝑠 𝑎 𝑟𝑖𝑔ℎ𝑡 𝑡𝑟𝑖𝑎𝑛𝑔𝑙𝑒

In right Δ𝐴𝐷𝐶, by the Pythagorean Theorem:


𝐷𝑖𝑎𝑔𝑜𝑛𝑎𝑙 = 𝐴𝐶 2 = 𝑠 2 + 𝑠 2 = 2𝑠 2 ⇒ 𝐴𝐶 = √2𝑠 2 = √2𝑠

Example 8.9
Find the diagonal of a square with the following side lengths:
A. 4
B. √7
4
C. √7

Part A
𝐹𝑟𝑜𝑚 𝐹𝑖𝑟𝑠𝑡 𝑃𝑟𝑖𝑛𝑐𝑖𝑝𝑙𝑒𝑠: 𝑑2 = 42 + 42 = 32 ⇒ 𝑑 = √32 = 4√2
𝑈𝑠𝑖𝑛𝑔 𝑡ℎ𝑒 𝑓𝑜𝑟𝑚𝑢𝑙𝑎: 𝑑 = 4√2

Part B

𝑑 = √2 × √7 = √14
Part C
4 4 4 4 4
𝑑 = √2 × √7 = √√4 × √7 = √4 × √7 = √28

Property 8.10: Diagonal of a square


The longest line connecting any two points on a square is the diagonal of the square.

Example 8.11
What is the length of the longest straight path through a square field with side length
√2 𝑢𝑛𝑖𝑡𝑠.

𝑠 = √2 ⇒ 𝑑 = √2𝑠 = √2 × √2 = 2

Example 8.12
A farmer has a square field with area 400 𝑢𝑛𝑖𝑡𝑠 2 where he plants alfalfa. He has his
house and a well in the field. What is the longest possible distance between the house
and the well?

𝐴 = 400 ⇒ 𝑠 = 20 ⇒ 𝑑 = 20√2

Example 8.13
Shaun is playing at a square park with length 100 𝑢𝑛𝑖𝑡𝑠. He tricycles down the shortest path from one corner to
𝑢𝑛𝑖𝑡𝑠
the opposite corner at a speed of 5 . At the other corner, his tricycle jams in the mud, and he walks back,
𝑚𝑖𝑛𝑢𝑡𝑒

P a g e 79 | 152
Get all the files at: https://bit.ly/azizhandouts
Aziz Manva ([email protected])

𝑢𝑛𝑖𝑡𝑠
crying, at a speed of 2 𝑚𝑖𝑛𝑢𝑡𝑒.
A. What is the time taken when tricycling?
B. What is the time taken when walking?
C. What is the difference in the time taken when tricycling, and walking?
𝑢𝑛𝑖𝑡𝑠 𝑢𝑛𝑖𝑡𝑠 𝑢𝑛𝑖𝑡𝑠
D. Note that the difference in speed when going and coming is 5 𝑚𝑖𝑛𝑢𝑡𝑒 − 2 𝑚𝑖𝑛𝑢𝑡𝑒
= 3 𝑚𝑖𝑛𝑢𝑡𝑒. Find the time
taken by someone to go from one corner to opposite corner via shortest path when travelling at a speed
𝑢𝑛𝑖𝑡𝑠
of 3 𝑚𝑖𝑛𝑢𝑡𝑒. Does this match your answer in Part C?

𝐿𝑒𝑛𝑔𝑡ℎ 𝑜𝑓 𝑃𝑎𝑡ℎ = 𝐷𝑖𝑎𝑔𝑜𝑛𝑎𝑙 = 100√2


Part A
𝐷 100√2
𝑇= = = 20√2 𝑚𝑖𝑛𝑢𝑡𝑒𝑠
𝑆 5
Part B
𝐷 100√2
𝑇= = = 50√2 𝑚𝑖𝑛𝑢𝑡𝑒𝑠
𝑆 2
Part C
𝐷𝑖𝑓𝑓𝑒𝑟𝑒𝑛𝑐𝑒 = 50√2 − 20√2 = 30√2
Part D
𝐷 100√2 1
𝑇= = = (33 ) √2 𝑚𝑖𝑛𝑢𝑡𝑒𝑠 ≠ 30√2
𝑆 3 3

Example 8.14
Points 𝐸 and 𝐹 are located on square 𝐴𝐵𝐶𝐷 so that △ 𝐵𝐸𝐹 is equilateral. What is
the ratio of the area of △ 𝐷𝐸𝐹 to that of △ 𝐴𝐵𝐸? (AMC 10A 2004/20)

D. Rectangle

Proof 8.15: Diagonal of a Rectangle


Show that a rectangle with length 𝑙 and width 𝑤 has diagonals with side length √𝑙 2 + 𝑤 2
Note that, in rectangle 𝐴𝐵𝐶𝐷, with length 𝑙 and width 𝑤,
Δ𝐴𝐷𝐶 𝑖𝑠 𝑎 𝑟𝑖𝑔ℎ𝑡 𝑡𝑟𝑖𝑎𝑛𝑔𝑙𝑒

In right Δ𝐴𝐷𝐶, by the Pythagorean Theorem:


𝐷𝑖𝑎𝑔𝑜𝑛𝑎𝑙 = 𝐴𝐶 2 = 𝑙 2 + 𝑤 2 ⇒ 𝐴𝐶 = √𝑙 2 + 𝑤 2
Q.E.D.

Example 8.16
4 4
The length of diagonal of a rectangle with sides √18 and √72 can be written in the form 𝑎√𝑏 where 𝑏 has no
perfect square terms. Find 𝑎 + 𝑏.

4 2 4 2
𝐷𝑖𝑎𝑔𝑜𝑛𝑎𝑙 2 = (√18) + (√72) = √18 + √72 = 3√2 + 6√2 = 9√2

𝐷𝑖𝑎𝑔𝑜𝑛𝑎𝑙 = √9√2 = 3√√2 ⇒ 𝑎 = 3, 𝑏 = √2 ⇒ 𝑎 + 𝑏 = 3 + √2

Example 8.17
P a g e 80 | 152
Get all the files at: https://bit.ly/azizhandouts
Aziz Manva ([email protected])

The perimeter of a rectangle is 56 meters. The ratio of its length to its width is 4:3. What is the length in meters
of a diagonal of the rectangle? (MathCounts 2005 State Team)

2(3𝑘 + 4𝑘) = 56 ⇒ 𝑘 = 4
𝐷𝑖𝑎𝑔𝑜𝑛𝑎𝑙 = √(3𝑘)2 + (4𝑘)2 = √9𝑘 2 + 16𝑘 2 = √25𝑘 2 = 5𝑘 = 20

E. Perimeter of a Trapezoid

Example 8.18
Draw a right triangle at the vertex

8.3 Geometrical Applications: 3D


A. Cube

8.19: Face and Space Diagonals


➢ A face diagonal of a cube is a diagonal that lies entirely on a face.
➢ A space diagonal of a cube is a diagonal that does not lie entirely on a face.

8.20: Face Diagonal of a Cube


A cube with side length 𝑠 has face diagonals with side length
√2𝑠

In right Δ𝐶𝐷𝐸, by the Pythagorean Theorem:


𝐶𝐸 2 = 𝑠 2 + 𝑠 2 ⇒ 𝐶𝐸 = √2𝑠

8.21: Space Diagonal of a Cube


A cube with side length 𝑠 has space diagonals with side length
√3𝑠

In right Δ𝐶𝐸𝐹, by the Pythagorean Theorem:


𝐶𝐹 2 = 2𝑠 2 + 𝑠 2 ⇒ 𝐶𝐹 = √3𝑠 2 = √3𝑠

Example 8.22
Find the length of the space diagonal of a cube with side length:
A. 3
B. √2
4
C. √27

𝑑 = 3√3
𝑑 = √2 × √3 = √6
4 4 4 4 4 4 4
𝑑 = √3 × √27 = √9 × √27 = √243 = √81 × √3 = 3 √3
4
1 3 5
𝑑 = √3 × √27 = 32 × 34 = 34

Property 8.23: Longest Diagonal of a Cube

P a g e 81 | 152
Get all the files at: https://bit.ly/azizhandouts
Aziz Manva ([email protected])

The longest diagonal of a cube is its space diagonal. All four space diagonals have the same length.

Example 8.24
A cubical container has a volume of 216 units. Find the length of the longest straight rod that can be kept in the
cubical container.

3
𝑉 = 216 ⇒ 𝑠 = √216 = 6 ⇒ 𝑑 = 6√3

Property 8.25: Finding Side given Diagonal


𝑑
𝑑 = √3𝑠 ⇒ 𝑠 =
√3

Example 8.26
The 𝑆𝑡𝑎𝑟𝑠ℎ𝑖𝑝 𝐸𝑛𝑡𝑒𝑟𝑝𝑟𝑖𝑠𝑒 spots an enemy spaceship. The ship engineers explains that if they position the
Enterprise at the bottom right, front, corner of an imaginary cube, and the enemy spaceship at the top, left, back
corner, then the shortest distance between the two spaceships is 300 𝑢𝑛𝑖𝑡𝑠. What is the side length of the
imaginary cube that the engineer has in mind?

𝑑 300 300√3
𝑠= = = = 100√3
√3 √3 3

B. Cuboid

Proof 8.27: Space Diagonal of a Cuboid


A face diagonal of a cube is a diagonal that lies entirely on a face. A space
diagonal of a cube is a diagonal that does not lie entirely on a face.
Show that a cuboid with length 𝑙, width 𝑤 and height ℎ has space diagonals
with side length √𝑙 2 + 𝑤 2 + ℎ2 .

In right ΔABC, by the Pythagorean Theorem:


𝐴𝐶 2 = 𝑙 2 + 𝑤 2 ⇒ 𝐴𝐶 = √𝑙 2 + 𝑤 2

In right ΔACG, by the Pythagorean Theorem:


2
𝐴𝐺 2 = (√𝑙 2 + 𝑤 2 ) + ℎ2 = 𝑙 2 + 𝑤 2 + ℎ2 ⇒ 𝐴𝐺 = √𝑙 2 + 𝑤 2 + ℎ2

Example 8.28

Property 8.29: Longest Diagonal of a Cuboid


The longest diagonal of a cuboid is its space diagonal. All four space diagonals have the same length.

Example 8.30

Example 8.31
Finding height given space diagonal, length, and width

P a g e 82 | 152
Get all the files at: https://bit.ly/azizhandouts
Aziz Manva ([email protected])

C. Triangular Prisms

Example 8.32
Height given slant edge and base

Example 8.33
Slant edge given height and base

Example 8.34
Base given slant edge and height

D. Pyramids

Example 8.35
Find Height

Example 8.36
Find Diagonal of Base

Example 8.37
Find Slant Edge

E. Cutting a Cylinder

Example 8.38
Find the length of the longest rod.

P a g e 83 | 152
Get all the files at: https://bit.ly/azizhandouts
Aziz Manva ([email protected])

9. SPECIAL RIGHT TRIANGLES


9.1 Isosceles Right Triangles
A. Context
We build some properties here, which are very useful in a number of ways:
➢ As an application of the Pythagorean Theorem, they help save time in calculations, letting us do
questions faster and easier.
➢ As a foundation for trigonometry, they create a base for later concepts. (In fact, remembering the
values of sin, cos, tan becomes much easier if you know the special right triangles thoroughly).

9.1: Sides in a 𝟒𝟓° − 𝟒𝟓° − 𝟗𝟎° Triangle


The hypotenuse of an isosceles right-angled triangle is √𝟐 times either of the congruent legs.
𝐻𝑦𝑝𝑜𝑡𝑒𝑛𝑢𝑠𝑒 = √2 × 𝐿𝑒𝑔
𝟏
The congruent legs of an isosceles right-angled triangle are times the hypotenuse.
√𝟐
𝐻𝑦𝑝
𝐿𝑒𝑔 =
√2

Consider an isosceles right-angled triangle with:


𝑙𝑒𝑛𝑔𝑡ℎ 𝑜𝑓 𝑙𝑒𝑔 = 𝑥
By Pythagoras Theorem:
𝐻𝑦𝑝2 = 𝑥 2 + 𝑥 2 = 2𝑥 2
Take square roots both sides:
𝐻𝑦𝑝 = √2𝑥 2 = √2𝑥
This also means that
𝐻𝑦𝑝
𝐿𝑒𝑔 = 𝑥 =
√2
B. Finding the Hypotenuse

Example 9.2
Find the hypotenuse in an isosceles right-angled triangle, if the leg is:
A. 5
B. 2√2
3 4
C. √2 − √2

𝑃𝑎𝑟𝑡 𝐵: 2√2 × √2 = 2 × 2 = 4
1 1 1 1 1 1 1 5 3
(√2)(√2 − √2) = (22 ) (23 − 24 ) = 22+3 − 22+4 = 26 − 24
3 4

Example 9.3
A garden is in the shape of an isosceles right triangle. If one of its smaller sides is 12 meters, find the length of
its longest side?

12√2
C. Finding the Leg

P a g e 84 | 152
Get all the files at: https://bit.ly/azizhandouts
Aziz Manva ([email protected])

Example 9.4
Find the leg in an isosceles right-angled triangle, if the hypotenuse is:
A. 3√2
B. 9√2
C. 4
D. 10
E. 7

3√2

1 4
4√2
4× = =
= 2√2
√2 √2 2
10 10 √2 10√2
= × = = 5√2
√2 √2 √2 2

Example 9.5
A garden is in the shape of an isosceles right triangle. If its longest side is 12 meters, find the length of its
shortest side?

12
= 6√2
√2
D. Finding the Perimeter

Example 9.6
A field that is in the shape of an isosceles right triangle is to be fenced. Find the cost of the fence at $5 per
square meter, if the length of one leg of the triangle is 70√2 meters.

2
√2 × 70√2 = 70(√2) = 70 × 2 = 140
$700
E. Finding the Area

Example 9.7
What is the area, in square units, of an isosceles right triangle with a hypotenuse of 20 units? (MathCounts
2009 State Countdown)

20 1 1 20 20 400
𝐿𝑒𝑔 = ⇒ 𝐴 = ℎ𝑏 = × × = = 100
√2 2 2 √2 √2 4

Example 9.8
ℎ𝑦𝑝2
Prove that the area of an isosceles right triangle with hypotenuse ℎ𝑦𝑝 is 4

ℎ𝑦𝑝 1 1 ℎ𝑦𝑝 ℎ𝑦𝑝 ℎ𝑦𝑝2


𝐿𝑒𝑔 = ⇒ 𝐴 = ℎ𝑏 = × × =
√2 2 2 √2 √2 4

P a g e 85 | 152
Get all the files at: https://bit.ly/azizhandouts
Aziz Manva ([email protected])

9.2 Applications
A. Diagrams

Example 9.9
Δ𝐴𝐵𝐶, drawn alongside, is an isosceles right-angled triangle, with a right angle at 𝐵.
Find the area and the perimeter for each part below (independently):
A. 𝐴𝐵 = 4
B. 𝐴𝐶 = 4
C. 𝐴𝐵 = √2
D. 𝐴𝐵 = √8
E. 𝐴𝐵 = 1 + √2
1
F. 𝐴𝐵 = 1+ 2

Part A
𝐵𝐶 = 𝐴𝐵 = 4, 𝐴𝐶 = 4√2
𝑃𝑒𝑟𝑖𝑚𝑒𝑡𝑒𝑟 = 4 + 4 + 4√2 = 8 + 4√2
1
Area = ℎ𝑏 = 2 × 4 × 4 = 8
2
Part B
4 4
4√2 √2
𝐴𝐵 = 𝐵𝐶 = = ×
= 2√2 =
√2 √2 √2 2
𝑃𝑒𝑟𝑖𝑚𝑒𝑡𝑒𝑟 = 4 + 2√2 + 2√2 = 4 + 4√2
1 1
𝐴𝑟𝑒𝑎 = ℎ𝑏 = × 2√2 × 2√2 = 4
2 2
Part C
𝐵𝐶 = √2, 𝐴𝐶 = 2
𝐴𝐶 = √2 × √2 = 2
Part D
𝐴𝐵 = √2 × √8 = √16 = 4
Part E
𝐴𝐶 = √2(1 + √2) = √2 + 2
Part F
1 √2 1 − √2 √2 − 2
𝐴𝐶 = √2 ( )= × = = 2 − √2
1 + √2 1 + √2 1 − √2 1−2

B. Squares

9.10: 𝟒𝟓° − 𝟒𝟓° − 𝟗𝟎° Triangle in Squares


The diagonal of a square divides it into two 45 − 45 − 90 triangles.

P a g e 86 | 152
Get all the files at: https://bit.ly/azizhandouts
Aziz Manva ([email protected])

Construct square ABCD, and draw diagonal AC.


In Δ𝐴𝐵𝐶:
∠𝐴𝐵𝐶 = 90° (𝐴𝑛𝑔𝑙𝑒 𝑜𝑓 𝑎 𝑆𝑞𝑢𝑎𝑟𝑒)

Since 𝐴𝐵𝐶𝐷 𝑖𝑠 𝑎 𝑠𝑞𝑢𝑎𝑟𝑒 ⇒ 𝐴𝐵 = 𝐴𝐶 ⇒ Δ𝐴𝐵𝐶 𝑖𝑠 𝐼𝑠𝑜𝑠𝑐𝑒𝑙𝑒𝑠:


180 − 90 90
∠𝐵𝐴𝐶 = ∠𝐵𝐶𝐴 = = = 45°
2 2

Δ𝐴𝐵𝐶 𝑖𝑠 𝑎 45 − 45 − 90 𝑡𝑟𝑖𝑎𝑛𝑔𝑙𝑒.
Similarly:
Δ𝐴𝐷𝐶 𝑖𝑠 𝑎 45 − 45 − 90 𝑡𝑟𝑖𝑎𝑛𝑔𝑙𝑒

Example 9.11
A square piece of paper is folded once so that one pair of opposite corners coincide. When the paper is
unfolded, two congruent triangles have been formed. Given that the area of the original square is 49 square
inches, what is the number of inches in the perimeter of one of these triangles? Express your answer in simplest
radical form. (MathCounts 1998 State Countdown)

𝐴𝑟𝑒𝑎 𝑜𝑓 𝑆𝑞𝑢𝑎𝑟𝑒 = 49 ⇒ 𝑆𝑖𝑑𝑒 𝑜𝑓 𝑆𝑞𝑢𝑎𝑟𝑒 = 7 ⇒ 𝐿𝑒𝑛𝑔𝑡ℎ 𝑜𝑓 𝐷𝑖𝑎𝑔𝑜𝑛𝑎𝑙 = 7√2


𝑃𝑒𝑟𝑖𝑚𝑒𝑡𝑒𝑟 = 7 + 7 + 7√2 = 14 + 7√2

C. Sequence of Triangles

Example 9.12
In the diagram alongside, 𝐴𝐵 = 𝐵𝐶 and ∠𝐴𝐵𝐶 = 90°. Δ𝐴𝐶𝐷 is an isosceles
right-angled triangle. If 𝐴𝐵 = 3, then find:
A. The perimeter of Δ𝐴𝐶𝐷
B. The area of Δ𝐴𝐶𝐷

Example 9.13
In the diagram alongside, Δ𝑋𝑌𝑍 is an isosceles right-angled triangle with 𝑋𝑍 = 4.
Also, 𝑋𝑃 = 𝑃𝑌 and ∠𝑋𝑃𝑌 = 90°. Find:
A. The perimeter of Δ𝑋𝑃𝑌
B. The area of Δ𝑋𝑃𝑌

D. Nesting

9.14: Perpendicular in a 𝟒𝟓° − 𝟒𝟓° − 𝟗𝟎° Triangle


The perpendicular in a 45 − 45 − 90 triangle is half the length of the
hypotenuse.

Consider 45 − 45 − 90 Δ𝐴𝐵𝐶, right-angled at ∠𝐵.


Draw perpendicular 𝐵𝑃1 .

Since Δ𝐴𝐵𝐶 is isosceles:


𝐵𝑃1 𝑖𝑠 𝑡ℎ𝑒 𝑝𝑒𝑟𝑝𝑒𝑛𝑑𝑖𝑐𝑢𝑙𝑎𝑟 𝑏𝑖𝑠𝑒𝑐𝑡𝑜𝑟 𝑜𝑓 𝐴𝐶
Also, Δ𝐴𝑃1 𝐵 is isosceles:

P a g e 87 | 152
Get all the files at: https://bit.ly/azizhandouts
Aziz Manva ([email protected])

1 1
𝐵𝑃1 = 𝐴𝑃1 = × 𝐴𝐶 = × 𝐻𝑦𝑝𝑜𝑡𝑒𝑛𝑢𝑠𝑒
2 2

Example 9.15
Δ𝐴𝐵𝐶 is an isosceles right triangle, right-angled at 𝐵 with 𝐴𝐶 = 1. Drop a perpendicular from point 𝐵, meeting
𝐴𝐶 at 𝑃1 . Find the length of 𝐵𝑃1 .

Δ𝐵𝑃1 𝐶 is 45 − 45 − 90
𝐻𝑒𝑛𝑐𝑒, 𝑖𝑡 𝑖𝑠 𝑖𝑠𝑜𝑠𝑐𝑒𝑙𝑒𝑠 ⇒ 𝐵𝑃1 = 𝐶𝑃1

Also, 𝐵𝑃1 is the perpendicular bisector of 𝐴𝐶:


1 1 1
𝐶𝑃1 = × 𝐴𝐶 = × 1 =
2 2 2

(Continuation) Example 9.16


Draw the perpendicular from 𝑃1 , meeting 𝐴𝐵 at 𝑃2 . Find the length of 𝑃1 𝑃2

Method I: Consider 𝚫𝑨𝑩𝑷𝟏


𝐻𝑦𝑝 1
𝐴𝐵 = 𝐿𝑒𝑔 𝑜𝑓 Δ𝐴𝐵𝐶 = =
√2 √2
1 1 1 1
𝑃1 𝑃2 = 𝐴𝑃2 = 𝑜𝑓 𝐴𝐵 = × =
2 2 √2 2√2

Nesting Method: Consider 𝚫𝑩𝑷𝟏 𝑷𝟐


1
𝐻𝑦𝑝 1 1 1
𝑃1 𝑃2 = = 2 = × =
√2 √2 2 √2 2√2

(Continuation) Example 9.17


Draw the perpendicular from 𝑃2 , meeting 𝐴𝐶 at 𝑃3 . Find the length of 𝑃2 𝑃3

1 1 1 1 1 1
𝑃2 𝑃3 = 𝑜𝑓 𝐴𝑃1 = 𝑜𝑓 𝑜𝑓 𝐴𝐶 = × × 1 =
2 2 2 2 2 4
E. Radicals

Example 9.18
The perimeter of an isosceles right-angled triangle is 2012. Its area is: (NMTC Sub-Junior/Screening 2012/Part
A/14)
Note: Write your answer in simplified form. Recall that in simplified form there will be no radicals in the
denominator.

P a g e 88 | 152
Get all the files at: https://bit.ly/azizhandouts
Aziz Manva ([email protected])

Let:
𝐿𝑒𝑔 = 𝑥 ⇒ 𝐻𝑦𝑝𝑜𝑡𝑒𝑛𝑢𝑠𝑒 = √2𝑥
Hence:
𝑥
𝑃𝑒𝑟𝑖𝑚𝑒𝑡𝑒𝑟 = 𝑥 + 𝑥 + √2𝑥 = 2𝑥 + √2𝑥 = 𝑥(2 + √2)
We know that
2012
𝑥(2 + √2) = 2012 ⇒ 𝑥 = 𝑥
2 + √2

Hence:
1 1 2012 2 1 20122 1 20122
𝐴𝑟𝑒𝑎 = ℎ𝑏 = × ( ) = × = ×
2 2 2 + √2 2 4 + 4√2 + 2 2 6 + 4√2
To rationalize the denominator, multiply numerator and denominator by 6 − 4√2:
1 20122 𝟔 − 𝟒√𝟐 20122 (3 − 2√2) 20122 (3 − 2√2)
× × = = = 10062 (3 − 2√2)
𝟐 6 + 4√2 6 − 4√2 36 − 32 4

9.3 Equilateral Triangles


A. Background
Equilateral triangles can be split into two 30 − 60 − 90 triangles by dropping a perpendicular from any vertex
to the opposite base. This is very useful, as we see now.

9.19: Height of an Equilateral Triangle


√𝟑
The height of an equilateral triangle is 𝟐
times the side length.
√3
ℎ= 𝑠
2

Construct equilateral Δ𝐴𝐵𝐶 with side length 𝑠.


Draw height AX ⊥ 𝐵𝐶:
∠𝐴𝑋𝐵 = ∠𝐴𝑋𝐶 = 90°
An equilateral triangle is also an isosceles triangle.
In an isosceles triangle, the altitude to the base bisects the base.
Hence,
𝑠
𝐵𝑋 = 𝑋𝐶 =
2
In right Δ𝐴𝐵𝑋, by the Pythagorean Theorem:
𝐴𝐵2 = 𝐵𝑋 2 + 𝐴𝑋 2
𝑠
Substitute 𝐴𝐵 = 𝑠, 𝐵𝑋 = 2 , 𝐴𝑋 = ℎ:
1 2 1 2 1 3
𝑠 2 = ( 𝑠) + ℎ2 ⇒ ℎ2 = 𝑠 2 − ( 𝑠) = 𝑠 2 − 𝑠 2 = 𝑠 2
2 2 4 4
Take square roots:
3 √3
ℎ = √ 𝑠2 = 𝑠
4 2

B. Finding the Height

Example 9.20

P a g e 89 | 152
Get all the files at: https://bit.ly/azizhandouts
Aziz Manva ([email protected])

A. If an equilateral triangle has perimeter 12, then find the height of the triangle.
B. If an equilateral triangle has perimeter 9, then find the height of the triangle.

12 √3 √3
𝑃 = 12 ⇒ 𝑠 =
=4⇒ℎ= ×𝑠 = × 4 = 2√3
3 2 2
9 √3 √3 3√3
𝑃=9⇒𝑠= =3⇒ℎ= ×3= ×3 =
3 2 2 2
C. Finding the Side

Example 9.21
A. If the height of an equilateral triangle is 4, then find the perimeter of the triangle.
B. If the height of an equilateral triangle is 3, then find the perimeter of the triangle.

√3 √3 2 8 √3 8√3
ℎ= 𝑠⇒4= 𝑠 ⇒𝑠 =4× = × = ⇒ 𝑃 = 8√3
2 2 √3 √3 √3 3

9.22: Area of an Equilateral Triangle


√𝟑
The area of an equilateral triangle is 𝟒
times the square of the side length.
√3 2
𝐴= 𝑠
4

We start with the height calculated above.

1 1 √3 √3 2
𝐴= × 𝐻𝑒𝑖𝑔ℎ𝑡 × 𝐵𝑎𝑠𝑒 = × ( 𝑠) × 𝑠 = 𝑠
2 2 2 4

D. Calculating Area

Example 9.23
An equilateral triangle has perimeter 4 units. What is its area?

4 √3 2 √3 4 2 √3 16 4√3
𝑃=4⇒𝑠= ⇒𝐴= 𝑠 = ( ) = × =
3 4 4 3 4 9 9

Example 9.24
What is the ratio of the numerical value of the area, in square units, of an equilateral triangle of side length 4
units to the numerical value of its perimeter, in units? Express your answer as a common fraction in simplest
radical form. (MathCounts 2007 Warm-Up 9)

√3 2 √3 √3
𝐴𝑟𝑒𝑎: 𝑃𝑒𝑟𝑖𝑚𝑒𝑡𝑒𝑟 = 𝑠 : 3𝑠 = 42 × : 3 × 4 = √3: 3 =
4 4 3
E. Back Calculations

Example 9.25
A square and an equilateral triangle have equal perimeters. The area of the triangle is 2√3 square inches. What
is the number of inches in the length of the diagonal of the square? (MathCounts 2004 State Team)

P a g e 90 | 152
Get all the files at: https://bit.ly/azizhandouts
Aziz Manva ([email protected])

Find the side length of the triangle:


√3 √3
𝐴 = 𝑠2 × ⇒ 2√3 = 𝑠 2 × ⇒ 𝑠 = √8 = 2√2
4 4
Find the perimeter of the triangle:
𝑃 = 3𝑠 = 6√2
Find the side length of the square:
𝑃 6√2 3√2
𝑆= = =
4 4 2
Find the length of the diagonal:
3√2 3×2
𝐷= × √2 = =3
2 2

Example 9.26
The altitude of an equilateral triangle is √6 units. What is the area of the triangle, in square units? Express your
answer in simplest radical form. (MathCounts 2008 School Countdown)

Find the side length:


√3 √3
ℎ= 𝑠 ⇒ √6 = 𝑠 ⇒ 𝑠 = 2√2
2 2
Find the area:
1 1
𝐴 = ℎ𝑏 = × √6 × 2√2 = √12 = 2√3
2 2

Example 9.27
The altitude of an equilateral triangle is ℎ units. Express, in simplest radical form, in terms of ℎ:
A. The perimeter of the triangle in units.
B. the area of the triangle, in square units.

Find the side length:


√3 2ℎ
ℎ= 𝑠⇒𝑠=
2 √3
Find the perimeter:
6ℎ 6ℎ√3
𝑃 = 3𝑠 = = = 2√3ℎ
√3 3
Find the Area:
√3 2ℎ 2 √3 4ℎ2 √3 ℎ2 √3
𝐴 = 𝑠2 × =( ) × = × =
4 √3 4 3 4 3

9.4 More with Area


A. Back Calculations

9.28: Triangle with minimum perimeter


The triangle with minimum perimeter for a given area is an equilateral triangle.

Example 9.29
What is the minimum perimeter of a triangle with area 1 unit?

P a g e 91 | 152
Get all the files at: https://bit.ly/azizhandouts
Aziz Manva ([email protected])

Triangle with minimum perimeter must be equilateral.


√3 2 4 2 6 3
𝐴=1⇒ 𝑠 = 1 ⇒ 𝑠2 = ⇒ 𝑠 = 4 ⇒ 𝑃 = 3𝑠 = 4 = 2 × 34
4 √3 √3 √3

9.30: Triangle with maximum area


The triangle with maximum area for a given perimeter is an equilateral triangle.

Example 9.31
What is the maximum area of a triangle with perimeter 1 unit?

Triangle with maximum area must be equilateral.

1 √3 2 √3 1 2 √3
𝑃=1⇒𝑠= ⇒𝐴= 𝑠 = ×( ) =
3 4 4 3 36
B. Equating Area and Perimeter

Example 9.32
The numerical value of the perimeter of a triangle is equal to the numerical value of its area. Find this numerical
value.

We know that:
√3 2 3×4
𝐴=𝑃⇒ 𝑠 = 3𝑠 ⇒ 𝑠 = = √3 × 4 ⇒ 𝑃 = 3𝑠 = 12√3
4 √3

9.5 𝟑𝟎° − 𝟔𝟎° − 𝟗𝟎° Triangles


9.33: 𝟑𝟎° − 𝟔𝟎° − 𝟗𝟎°
The sides of a 30° − 60° − 90° triangle are in the ratio:
1 √3

1 : :

2 ⏟
2
𝑯𝒚𝒑𝒐𝒕𝒆𝒏𝒖𝒔𝒆
𝑺𝒊𝒅𝒆 𝒐𝒑𝒑 𝑺𝒊𝒅𝒆 𝒐𝒑𝒑
𝟑𝟎° 𝟔𝟎°

Draw an equilateral triangle with side length 𝑠, and drop a perpendicular


from the vertex to get a 30-60-90 triangle.
Then, as seen above the sides of the triangle are:
1 √3

𝑠 : 𝑠 : 𝑠
𝑯𝒚𝒑𝒐𝒕𝒆𝒏𝒖𝒔𝒆
2⏟ ⏟2
𝑺𝒊𝒅𝒆 𝒐𝒑𝒑 𝑺𝒊𝒅𝒆 𝒐𝒑𝒑
𝟑𝟎° 𝟔𝟎°

Divide the ratio above by 𝑠, to get the ratio of the sides of a 30 − 60 − 90 𝑡𝑟𝑖𝑎𝑛𝑔𝑙𝑒:

A. Basics

Example 9.34
The hypotenuse of a 30 − 60 − 90 triangle is √3. Find the length of the other two sides.

P a g e 92 | 152
Get all the files at: https://bit.ly/azizhandouts
Aziz Manva ([email protected])

1 √3
𝑆𝑖𝑑𝑒 𝑜𝑝𝑝𝑜𝑠𝑖𝑡𝑒 30° = √3 × =
2 2
√3 3
𝑆𝑖𝑑𝑒 𝑜𝑝𝑝𝑜𝑠𝑖𝑡𝑒 60° = √3 × =
2 2

Example 9.35
The shortest side of a 30 − 60 − 90 is 3. Find the length of the other two sides.

Shortest side is side opposite 30°.

𝐻𝑦𝑝𝑜𝑡𝑒𝑛𝑢𝑠𝑒 = 3 × 2 = 6
𝑆𝑖𝑑𝑒 𝑜𝑝𝑝𝑜𝑠𝑖𝑡𝑒 60° = 3 × √3 = 3√3
B. Ratios

Example 9.36
The ratio of the measures of the angles of a triangle is 3:2:1. Given that the shortest side of the triangle is 12
meters long, what is the number of meters in the longest side of the triangle? (MathCounts 1997 Chapter
Countdown)
Since the angles are in the ratio
3: 2: 1 = 90: 60: 30 ⇒ 𝑇𝑟𝑖𝑎𝑛𝑔𝑙𝑒 𝑖𝑠 𝑎 30 − 60 − 90 𝑡𝑟𝑖𝑎𝑛𝑔𝑙𝑒.

In a 30 − 60 − 90 𝑡𝑟𝑖𝑎𝑛𝑔𝑙𝑒, the shortest side is half of the longest side.


Hence, the longest side is double of the shortest side
= 12 × 2 = 24
C. Calculating Area

Example 9.37
1 22
The hypotenuse of a 30 − 60 − 90 triangle has length units. Considering 𝜋 = , the area of the triangle can
√𝜋 7
𝑎√𝑏
be written in the form 𝑐
, 𝑎, 𝑏, 𝑐 ∈ ℕ, 𝑏 does not have any perfect square factors, and 𝐻𝐶𝐹(𝑎, 𝑐) = 1. Find 𝑎 +
𝑏 + 𝑐.

1 1 1 1 √3 √3 √3 √3 7 7√3
ℎ𝑏 = × × × = = = × =
2 2 2 ⏟√𝜋 ⏟ √𝜋 2 8𝜋 8 × 22 8 22 176
ℎ𝑒𝑖𝑔ℎ𝑡 𝑏𝑎𝑠𝑒 7

𝑎 + 𝑏 + 𝑐 = 7 + 3 + 176 = 186

Example 9.38
The hypotenuse of a 30 − 60 − 90 triangle has length ℎ units. Find the area of the triangle, in terms of ℎ.

D. Back Calculations

Example 9.39
A 30 − 60 − 90 triangle has area 1 unit. Find the perimeter of the triangle.

P a g e 93 | 152
Get all the files at: https://bit.ly/azizhandouts
Aziz Manva ([email protected])

Example 9.40
A 30 − 60 − 90 triangle has area A units. Find the perimeter of the triangle.

E. Nested Triangles

9.41: Similar Right Triangles


The two triangles formed by dropping a perpendicular from the vertex of a right-angled triangle are similar to
each other, and to the original triangle.

This is the same property that we used for 45 − 45 − 90 triangles earlier.

This is a very important property. Consider Δ𝐴𝐵𝐶 which is 30 − 60 − 90. Drop a


perpendicular from C to intersect AB at D.
∠𝐴𝐶𝐷 = 180 − 90 − 30 = 60 ⇒ Δ𝐴𝐶𝐷 𝑖𝑠 30 − 60 − 90
∠𝐷𝐶𝐵 = 180 − 90 − 60 = 30 ⇒ ΔDCB 𝑖𝑠 30 − 60 − 90

Example 9.42
If altitude CD is √3 centimeters, what is the number of square centimeters in the area of
Δ𝐴𝐵𝐶? (MathCounts 1999 School Team)

𝐴𝐶 = 2 × 𝐶𝐷 = 2√3
2 1
𝐵𝐶 = (2√3) ( ) ( ) = 2
√3 2
1 1
𝐴 = ℎ𝑏 = (2√3)(2) = 2√3
2 2

Example 9.43
The altitude to the hypotenuse of a triangle with angles of 30 and 60 degrees
is 3 units. What is the area of the triangle, in square units? Express your
answer in simplest radical form. (MathCounts 2009 State Countdown)

F. Multiple Triangles

Example 9.44
In the diagram, Δ𝐴𝐵𝐸, Δ𝐵𝐶𝐸 and Δ𝐶𝐷𝐸 are right-angled, with ∠𝐴𝐸𝐵 = ∠𝐵𝐸𝐶 =
∠𝐶𝐸𝐷 = 60, and 𝐴𝐸 = 24. (CEMC 2006 Hypatia)
A. Find the length of 𝐶𝐸.
B. Determine the perimeter of quadrilateral ABCD.
C. Determine the area of quadrilateral ABCD.

Example 9.45
Square ABCD has side length 1 unit. Points E and F are on sides AB and CB, respectively, with AE=CF. When the
square is folded along the lines DE and DF, sides AD and CD coincide and lie on diagonal BD. The length of

P a g e 94 | 152
Get all the files at: https://bit.ly/azizhandouts
Aziz Manva ([email protected])

segment AE can be expressed in the form \sqrt{k}-m units. What is the integer value of k+m? (MathCounts
2011 Chapter Target)

9.6 Review and Challenge


Concept Check 9.46
Consider the Question:
4 3
In an isosceles right-angled triangle, the leg is (√2 − √2). Find the hypotenuse.
Consider its Solution:
1 1 1 1 1 1 1 3 5
(√2)(√2 − √2) = (22 ) (24 − 23 ) = 22+4 − 22+3 = 24 − 26
4 3

Is there a mistake anywhere, or is everything fine?

4 3 4 3
√2 < √2 ⇒ √2 − √2 < 0
Length cannot be negative.
Hence, the question is not meaningful.

Example 9.47
Quadrilateral 𝐴𝐵𝐶𝐷 is a square with area 16 square inches. The figure represents
the pieces of a Chinese tangram in which all the triangles are isosceles and piece 𝑒
is a square. What is the area of the gray piece, in square inches? (MathCounts
2008 Chapter Countdown)

𝐴 = 16 ⇒ 𝑠 = 4
1 2 1
𝐴 = 𝑠 = ×2×2=2
2 2

P a g e 95 | 152
Get all the files at: https://bit.ly/azizhandouts
Aziz Manva ([email protected])

PART IV: LINES AND CIRCLES


10. ANGLE BISECTORS
10.1 Angle Chasing (Bisectors)
A. Definition

10.1: Angle Bisector


The line segment from the vertex of an angle that bisects the angle is
called an angle bisector.

In the diagram (not drawn to scale), AD is the angle bisector if


∠𝐵𝐴𝐷 = ∠𝐶𝐴𝐷

➢ Every triangle has three angle bisectors.


➢ An angle bisector never goes outside the triangle.

Example 10.2
In right triangle 𝐽𝐾𝐿, angle 𝐽 measures 60 degrees and angle 𝐾 measures 30 degrees. When
drawn, the angle bisectors of angles 𝐽 and 𝐾 intersect at a point 𝑀. What is the measure of
obtuse angle 𝐽𝑀𝐾? (MathCounts 2010 School Sprint)

60 30
180 − ∠𝐾𝐽𝑀 − ∠𝐽𝐾𝑀 = 180 − − = 180 − 30 − 15 = 135
2 2

B. Vertex Angle formed by Angle Bisectors

Example 10.3
The measure of angle 𝐴𝐵𝐶 is 50∘ , 𝐴𝐷 bisects angle 𝐵𝐴𝐶, and 𝐷𝐶 bisects angle 𝐵𝐶𝐴.
The measure of angle 𝐴𝐷𝐶 is (AMC 8 1996/24)

Let
∠𝐵𝐴𝐶 = 2𝑥 ⇒ ∠𝐷𝐴𝐶 = 𝑥
∠𝐵𝐶𝐴 = 2𝑦 ⇒ ∠𝐷𝐶𝐴 = 𝑦
By sum of angles of a triangle:
50 + 2𝑥 + 2𝑦 = 180 ⇒ 2𝑥 + 2𝑦 = 130 ⇒ 𝑥 + 𝑦 = 65
In Δ𝐴𝐷𝐶:
∠𝐴𝐷𝐶 = 180 − 𝑥 − 𝑦 = 180 − (𝑥 + 𝑦) = 180 − 65 = 115

C. Right Triangle

Example 10.4
In isosceles right-triangle 𝐴𝐵𝐶, right-angled at 𝐵, the angle bisector from 𝐵 intersects side 𝐴𝐶 at 𝐷. If the length
of one leg of the triangle is 1 unit:
A. Find the perimeter of ΔABD.
B. Find the area of ΔBDC.

P a g e 96 | 152
Get all the files at: https://bit.ly/azizhandouts
Aziz Manva ([email protected])

1
𝐴𝐷 = 𝐵𝐷 =
√2
1 1 2
𝑃 =1+ + =2+
√2 √2 √2
1 1 1 1
𝐴= × × =
2 √2 √2 4

D. Equilateral Triangles

Example 10.5
Show that the angle bisector of an equilateral triangle divides the triangle into two 30 − 60 − 90 triangles.

𝐵𝐴𝐶 60
∠𝐵𝐴𝑋 = ∠ = = 30°
2 2
∠𝐵𝑋𝐴 = 180 − 30 − 60 = 90°

Pending
Example 10.6
Equilateral Δ𝐴𝐵𝐶 has side length 1. The angle bisector from B intersects AC at X and the line perpendicular to
BC at Y. Find the area of Δ𝑋𝑌𝐶.

Δ𝐵𝑌𝐶 is a 30 − 60 − 90 triangle.
1 1
𝑌𝐶 = 𝐵𝐶 =
√3 √3
Δ𝑋𝑌𝐶 is a 30 − 60 − 90 triangle.
1 1 1 1
𝑋𝑌 = × 𝑌𝐶 = × =
2 2 √3 2√3
√3 √3 1 1
𝑋𝐶 = × 𝑌𝐶 = × =
2 2 √3 2
E. Vertex Angle

10.7: Vertex Angle formed by Angle Bisectors


The vertex angle in the triangle formed by the base of a triangle and the angle bisectors of its base is equal to a
right angle plus half the vertex angle of the original triangle.

In △𝐴𝐵𝐶 let ∠’s 𝐴, 𝐵 and 𝐶 have measures 2𝑎, 2𝑏 and 2𝑐 respectively.

By Sum of Angles of a Triangle:


2𝑎 + 2𝑏 + 2𝑐 = 180
Solve the above for 𝑎:
2𝑎 = 180 − 2𝑏 − 2𝑐 ⇒ 𝑎 = 90 − 𝑏 − 𝑐

The angle bisectors of ∠𝐵 and ∠𝐶 meet at 𝑋.

P a g e 97 | 152
Get all the files at: https://bit.ly/azizhandouts
Aziz Manva ([email protected])

By Sum of Angles of a Triangle:


∠𝑋 = 180 − 𝑏 − 𝑐 = 90 + 90 − 𝑏 − 𝑐 = 90 + 𝑎

Example 10.8
The measure of angle 𝐴𝐵𝐶 is 50∘ , 𝐴𝐷 bisects angle 𝐵𝐴𝐶, and 𝐷𝐶 bisects angle 𝐵𝐶𝐴.
The measure of angle 𝐴𝐷𝐶 is (AMC 8 1996/24)
(Redo the question using the property we just proved)

1 1
∠𝐴𝐷𝐶 = 90 + ∙ ∠𝐴𝐵𝐶 = 90 + ∙ 50 = 90 + 25 = 115°
2 2
F. Challenging Problems

Example 10.9
̅̅̅̅ so that 𝐵𝐷
In triangle 𝐴𝐵𝐶, 𝐴𝐵 = 𝐴𝐶 and 𝐷 is a point on 𝐴𝐶 ̅̅̅̅ bisects angle ABC. If 𝐵𝐷 = 𝐵𝐶, what is the measure,
in degrees, of angle 𝐴? (MathCounts 2010 Chapter Sprint)

In Isosceles Δ𝐴𝐵𝐶, let


∠𝐴𝐵𝐶 = ∠𝐴𝐶𝐵 = 2𝑥
In Isosceles ΔBDC,
∠𝐵𝐷𝐶 = ∠𝐵𝐶𝐷 = 2𝑥
By the definition of angle bisector
2𝑥
∠𝐷𝐵𝐶 = =𝑥
2
In Δ𝐵𝐷𝐶, by sum of angles of a triangle:

𝑥 + 2𝑥 ⏟ + 2𝑥 ⏟ = 180 ⇒ 𝑥 = 36
∠𝑫𝑩𝑪 ∠𝑩𝑪𝑫 ∠𝑩𝑫𝑪
In Δ𝐴𝐵𝐶, by sum of angles of a triangle:
∠𝐴 = 180 − 2𝑥 ⏟ − 2𝑥 ⏟ = 180 − 144 = 36°
∠𝑨𝑩𝑪 ∠𝑨𝑪𝑩

Challenge 10.10
In Δ𝐴𝐵𝐶, a line segment from B, drawn perpendicular to the angle bisector from 𝐴 intersects it at 𝐷 to form
1
isosceles Δ𝐴𝐵𝐷. Also, 𝐵𝐷 divides angle 𝐵 in the ratio 1:3. If the longest side of triangle Δ𝐴𝐵𝐶 has length , find
√3
the second longest side.

If the perpendicular from 𝐵 intersects 𝐴𝐷, then


∠𝐴𝐷𝐵 = 90°

Δ𝐴𝐵𝐷 is isosceles right-angled triangle:


∠𝐵𝐴𝐷 = ∠𝐴𝐵𝐷 = 45

P a g e 98 | 152
Get all the files at: https://bit.ly/azizhandouts
Aziz Manva ([email protected])

Since AD is the angle bisector:


∠𝐷𝐴𝐶 = ∠𝐵𝐴𝐷 = 45
Now, we are in a position to ∠𝐷𝐵𝐶 using the ratio 1: 3:
∠𝐷𝐵𝐶 = 3∠𝐴𝐵𝐷 = 135 ⇒ 𝑁𝑜𝑡 𝑃𝑜𝑠𝑠𝑖𝑏𝑙𝑒
1
∠𝐷𝐵𝐶 = ∠𝐴𝐵𝐷 = 15
3

∠𝐴𝐶𝐵 = 180 − 90 − 60 = 30 ⇒ Δ𝐵𝐴𝐶 𝑖𝑠 30 − 60 − 90 𝑡𝑟𝑖𝑎𝑛𝑔𝑙𝑒.

1 √3 1
𝐴𝐶 = × =
√3 2 2

10.2 Angle Bisector Theorem


A. Basics

10.11: Angle Bisector Theorem


An angle bisector of a triangle divides the side opposite it in the ratio of the other
two sides of the triangle.
𝐵𝐷 𝐷𝐶 𝐵𝐷 𝑐
= ⇔ = ⇔ 𝐵𝐷: 𝐷𝐶 = 𝑐: 𝑏
𝑐 𝑏 𝐷𝐶 𝑏
𝐴
Example 10.12
In the diagram, 𝐴𝐷 is an angle bisector of the vertex angle 𝐴. Also,
𝐴𝐷 = 𝑥 = 24, 𝐴𝐶 = 𝑦 = 36 𝑥 𝑦
A. Find the ratio 𝐵𝐷: 𝐷𝐶 in terms of 𝑥 and 𝑦 (as variables, not as numbers).
B. If 𝐵𝐶 = 5, find the length of 𝐵𝐷 and 𝐷𝐶.
C. If 𝐵𝐷 = 5, find the length of 𝐷𝐶.
𝐵 𝐷 𝐶
Part A
𝐵𝐷: 𝐷𝐶 = 𝑥: 𝑦

Part B
𝑥 24 2
= = ⇒ 2 + 3 = 5 ⇒ 𝐵𝐶 = ⏟
2 + ⏟
3 =5
𝑦 36 3 𝑩𝑫 𝑫𝑪

Part C
𝐵𝐷 2 5 2 3
= ⇒ = ⇒ 𝐷𝐶 = 5 × = 7.5
𝐷𝐶 3 𝐷𝐶 3 2

Example 10.13
In Δ𝑋𝑌𝑍, the angle bisector from X intersects side YZ at A. If the lengths of 𝑌𝐴 and
𝐴𝑍 are 𝑥 and 𝑦, respectively, and 𝑋𝑌 = 𝑌𝐴 + 𝑌𝑍, then find the length of 𝑋𝑍, in terms
of 𝑥 and 𝑦.

𝑋𝑌 𝑋𝑍 𝑥 + 𝑦 𝑋𝑍 𝑦(𝑥 + 𝑦)
= ⇒ = ⇒ 𝑋𝑍 =
𝑌𝐴 𝐴𝑍 𝑥 𝑦 𝑥

Example 10.14
The sides of triangle 𝐶𝐴𝐵 are in the ratio of 2: 3: 4. Segment 𝐵𝐷 is the angle bisector drawn to the shortest side,

P a g e 99 | 152
Get all the files at: https://bit.ly/azizhandouts
Aziz Manva ([email protected])

dividing it into segments 𝐴𝐷 and 𝐷𝐶. What is the length, in inches, of the longer subsegment of side 𝐴𝐶 if the
length of side 𝐴𝐶 is 10 inches? Express your answer as a common fraction. (MathCounts 2010 National Sprint)

Without loss of generality, let


𝐶𝐵 𝑏𝑒 𝑡ℎ𝑒 𝑙𝑜𝑛𝑔𝑒𝑟 𝑠𝑖𝑑𝑒 ⇒ 𝑊𝑒 𝑛𝑒𝑒𝑑 𝑡𝑜 𝑓𝑖𝑛𝑑 𝐷𝐶
AB and CD are in the ratio:
3𝑥: 4𝑥 = 3: 4
Hence, by Angle Bisector Theorem:
𝐴𝐷: 𝐶𝐷 = 3: 4
We know that length of AC is 10. Hence, we need to divide AC in the ratio
3: 4, and find the longer segment:
10 40
3 + 4 = 7 ⇒ 𝐷𝐶 = ×4=
7 7
B. Checking for Angle Bisectors

Pending
Example 10.15

C. Expressions
Some questions ask for a combination of two or more elements. It is sometimes to possible to calculate the two
elements together (even if the information given is not enough to calculate the two elements separately).

Example 10.16
In Δ𝐴𝐵𝐶 with perimeter 1 𝑚, 𝐴𝐷 is the angle bisector from 𝐴, intersecting side 𝐵𝐶 at 𝐷. 𝐷𝐶 is 25% longer than
𝐵𝐷. Find the sum of the length of 𝐴𝐵 and 𝐵𝐷, in cm.

Let
5
𝐵𝐷 = 𝑥 ⇒ 𝐷𝐶 = 𝑥
4
5
𝐴𝐵 = 𝑦 ⇒ 𝐴𝐶 = 𝑦
4
5 5 9
𝑥 + 𝑥 + 𝑦 + 𝑦 = 100 𝑐𝑚 ⇒ (𝑥 + 𝑦) = 100
4 4 4
100 × 4 400
𝐴𝐵 + 𝐵𝐷 = 𝑥 + 𝑦 = = 𝑐𝑚
9 9

D. Algebra

Example 10.17
In ΔPQR, the angle bisector from S intersects QR at S. What are the possible values of 𝑥, if:
𝑃𝑄 = 𝑥 + 5, 𝑄𝑆 = 𝑥 − 2, 𝑆𝑅 = 𝑥 − 3, 𝑃𝑅 = 𝑥 + 4

P a g e 100 | 152
Get all the files at: https://bit.ly/azizhandouts
Aziz Manva ([email protected])

𝑄𝑆 𝑆𝑅
= ⇒ (𝑄𝑆)(𝑃𝑅) = (𝑆𝑅)(𝑄𝑃)
𝑄𝑃 𝑃𝑅
(𝑥 − 2)(𝑥 + 4) = (𝑥 − 3)(𝑥 + 5)
𝑥 + 4𝑥 − 2𝑥 − 8 = 𝑥 2 − 3𝑥 + 5𝑥 − 15
2

2𝑥 − 8 = 2𝑥 − 15
−8 = −15
𝑁𝑜𝑡 𝑃𝑜𝑠𝑠𝑖𝑏𝑙𝑒
𝑥 ∈ 𝜙 ⇒ 𝑁𝑜 𝑆𝑜𝑙𝑢𝑡𝑖𝑜𝑛𝑠

Example 10.18
In ΔLMN, the angle bisector from L intersects MN at A. What are the possible values of 𝑥, if:
𝑀𝐴 = 𝑥 2 − 2𝑥 − 15
𝐴𝑁 = 𝑥 2 + 6𝑥 − 7
𝐿𝑀 = 𝑥 2 − 3𝑥 − 10
𝐿𝑁 = 𝑥 2 + 3𝑥 − 4

𝑀𝐴 𝐴𝑁
=
𝐿𝑀 𝐿𝑁
(𝑥 + 3)(𝑥 − 5) (𝑥 + 7)(𝑥 − 1)
=
(𝑥 − 5)(𝑥 + 2) (𝑥 − 1)(𝑥 + 4)
𝑥+3 𝑥+7
=
𝑥+2 𝑥+4
𝑥 2 + 7𝑥 + 12 = 𝑥 2 + 9𝑥 + 14
−2 = 2𝑥
𝑥 = −1

𝑀𝐴 = (𝑥 + 3)(𝑥 − 5) = (−1 + 3)(−1 − 5) = (2)(−6) = −12 ⇒ 𝑁𝑒𝑔𝑎𝑡𝑖𝑣𝑒 ⇒ 𝑁𝑜𝑡 𝑃𝑜𝑠𝑠𝑖𝑏𝑙𝑒


Had the first value been positive, we would have needed to check remaining three quadratic expressions.
If all values are positive, only then is 𝑥 a valid solution.
E. Right Triangles
In a right triangle, the angle bisector can be combined with the Pythagorean Theorem.

Example 10.19
𝑋𝑌 has length 12 in right ΔXYZ. 𝑋𝐴 is the angle bisector from 𝑋, and intersects 𝑌𝑍 at 𝐴. If 𝑌𝐴 = 4, find
hypotenuse 𝑋𝑍.

Let 𝐴𝑍 = 𝑥. By the angle bisector theorem:


𝑋𝑍 𝑋𝑌 12
= = = 3 ⇒ 𝑋𝑍 = 3𝑥
𝑥 𝑌𝐴 4

By the Pythagorean Theorem in Δ𝑋𝑌𝑍, we know that


𝑋𝑌 2 + 𝑌𝑍 2 = 𝑋𝑍 2
144 + (4 + 𝑥)2 = (3𝑥)2
144 + 16 + 8𝑥 + 𝑥 2 = 9𝑥 2
Collate all terms on one side to get a quadratic. Factor and solve:
8(𝑥 2 − 𝑥 − 20) = 0 ⇒ (𝑥 − 5)(𝑥 + 4) = 0
Take the positive solution:

P a g e 101 | 152
Get all the files at: https://bit.ly/azizhandouts
Aziz Manva ([email protected])

𝑥 = 5 ⇒ 𝑋𝑍 = 3𝑥 = 15

Example 10.20
Triangle 𝐴𝐵𝐶 has a right angle at 𝐵, 𝐴𝐵 = 1, and 𝐵𝐶 = 2. The angle bisector of
∠𝐴 intersects side 𝐵𝐶 at 𝐷. What is 𝐵𝐷? (AMC 10B 2009/20)

In Δ𝐴𝐵𝐶, by Pythagorean Theorem:


𝐴𝐶 = √𝐴𝐵2 + 𝐵𝐶 2 = √12 + 22 = √5
Now,
𝐴𝐵 = 1, 𝐴𝐶 = √5
Hence, by the angle bisector theorem,
𝐵𝐷 𝐴𝐵
=
𝐵𝐶 𝐴𝐵 + 𝐴𝐶
𝐵𝐶 2 2 √5 − 1 2(√5 − 1) √5 − 1
𝐵𝐷 = × 𝐴𝐵 = ×1= × = =
𝐴𝐵 + 𝐴𝐶 √5 + 1 √5 + 1 √5 − 1 4 2

In the above question, what is the area of Δ𝐴𝐷𝐶?

1 1 √5 − 1 √5 − 1 4 − √5 + 1 5 − √5
[𝐴𝐷𝐶] = [𝐴𝐵𝐶] − [𝐴𝐵𝐷] = ( × 1 × 2) − ( × 1 × )=1− = =
2 2 2 4 4 4

Pending
Example 10.21
In right Δ𝑋𝑌𝑍 the side lengths are 𝑋𝑌 = 6, 𝑌𝑍 = 8 and 𝑋𝑍 = 10. XP is the angle bisector of ∠𝑌𝑋𝑍 intersecting
YZ at P. YQ is the angle bisector of ∠𝑋𝑌𝑍, intersecting XP at Q. PR is the angle bisector of ∠𝑋𝑃𝑍 intersecting XZ
at Q. Find 𝑋𝑃 + 𝑋𝑄 + 𝑋𝑅.

Step I: Find YP using the Angle Bisector Theorem


𝑌𝑃 𝑋𝑌 6 3 3 3
= = = ⇒ 𝑌𝑃 = 𝑌𝑍 × =8× =3
𝑃𝑍 𝑋𝑍 10 5 3+5 8

Step II: Find XP using the Pythagorean Theorem


𝑋𝑃 = √𝑋𝑌 2 + 𝑌𝑃2 = √62 + 32 = √36 + 9 = √45 = 3√5

Step III: Find XQ using the Angle Bisector Theorem


𝑋𝑄 𝑋𝑌 6 2
= = = 2 ⇒ 𝑋𝑄 = 3√5 × = 2√5
𝑄𝑃 𝑌𝑃 3 3

Step IV: Find XR using the Angle Bisector Theorem


𝑋𝑅 𝑋𝑃 3√5 3√5 3√5(3√5 − 5) 45 − 15√5
= = ⇒ 𝑋𝑅 = 𝑋𝑍 × = 10 × = 10 × = 22.5 − 7.5√5
𝑅𝑍 𝑃𝑍 5 3√5 + 5 45 − 25 20

Step V: Find the Total


𝑋𝑃 + 𝑋𝑄 + 𝑋𝑄 = 3√5 + 2√5 + 22.5 − 7.5√5 = 22.5 − 2.5√5

F. Isosceles Triangles

P a g e 102 | 152
Get all the files at: https://bit.ly/azizhandouts
Aziz Manva ([email protected])

Example 10.22
In Δ𝐴𝐵𝐶, angle bisector 𝐴𝐷 divides side 𝐵𝐶 in the ratio 1: 1. If 𝐴𝐵 = 5, then find 𝐴𝐶.

If the angle bisector divides the opposite side in the ratio 1: 1, then it is also the median.
And if the angle bisector is the same as the median, the triangle is isosceles.

Hence,
𝐴𝐶 = 𝐴𝐵 = 5

Pending
10.3 Ratios
A. Ratio of Areas

Example 10.23
In Δ𝐴𝐶𝐵, the angle bisector from C intersects 𝐴𝐵 at X. If 𝐴𝐶 = 4, 𝐶𝐵 = 6 and Δ𝐴𝐶𝐵 has area 𝜋 units, then find
the area of Δ𝐶𝐴𝑋.

The angle bisector divides the triangle into two smaller triangles.
Find the ratio of the areas of these two smaller triangles.
1
𝐴(Δ𝐶𝐴𝑋) 2 × ℎ × 𝐴𝑋 𝐴𝑋 𝐶𝐴 4 2
= = = = = = 2: 3
𝐴(ΔXCB) 1 × ℎ × 𝑋𝐵 𝑋𝐵 𝐶𝐵 6 3
2

Hence, we need to divide Δ𝐴𝐶𝐵 in the ratio 2: 3:


2 2 2𝜋
𝐴(Δ𝐶𝐴𝑋) = Δ𝐴𝐶𝐵 × = 𝜋 × =
5 5 5

10.24: Ratio of Areas


Consider a triangle with base 𝑏 and height ℎ. The angle bisector to the
base divides it into two smaller triangles. The ratio of the areas of the
smaller triangles is the ratio of other two sides:
[𝐶𝐴𝑋] 𝑥
=
[𝑋𝐶𝐵] 𝑦

In Δ𝐴𝐶𝐵, the angle bisector from C intersects 𝐴𝐵 at X. If 𝐴𝐶 = 𝑥, 𝐶𝐵 = 𝑦,


then:
1
[𝐶𝐴𝑋] 2 × ℎ × 𝐴𝑋 𝐴𝑋 𝐶𝐴 𝑥
= = = =
[𝑋𝐶𝐵] 1 𝑋𝐵 𝐶𝐵 𝑦
2 × ℎ × 𝑋𝐵

In the same triangle, we can apply the properties of ratios to get:


[𝐶𝐴𝑋] 𝑥
=
[𝐶𝐴𝐵] 𝑥 + 𝑦

[𝐶𝐴𝑋] 𝑥
Take the reciprocal of both sides of [𝑋𝐶𝐵] = 𝑦 to get:
[𝑋𝐶𝐵] 𝑦
=
[𝐶𝐴𝑋] 𝑥

P a g e 103 | 152
Get all the files at: https://bit.ly/azizhandouts
Aziz Manva ([email protected])

Add 1 to both sides:


[𝑋𝐶𝐵] 𝑦 [𝑋𝐶𝐵] + [𝐶𝐴𝑋] 𝑦 + 𝑥 [𝐶𝐴𝐵] 𝑦 + 𝑥
+1= +1⇒ = ⇒ =
[𝐶𝐴𝑋] 𝑥 [𝐶𝐴𝑋] 𝑥 [𝐶𝐴𝑋] 𝑥
Again, take the reciprocal of both sides:
[𝐶𝐴𝑋] 𝑥
=
[𝐶𝐴𝐵] 𝑥 + 𝑦

B. Special Right Triangles

Example 10.25
𝐴𝐷 is the angle bisector from vertex 𝐴 in right Δ𝐴𝐵𝐶, which has ∠𝐵 = 30, and hypotenuse 𝐵𝐶 = 1. Find the
area of Δ𝐴𝐵𝐷 and Δ𝐴𝐷𝐶.

√3 1
[𝐴𝐵𝐷]: [𝐴𝐷𝐶] = : = √3: 1
2 2

1 1 √3 √3
[𝐴𝐵𝐶] =× × =
2 2 2 8
√3 1 √3(√3 − 1) 3 − √3
[𝐴𝐷𝐶] = × = =
8 √3 + 1 8(2) 16
√3 √3 3(√3 − 1) 3√3 − 3
[𝐴𝐵𝐷] = × = =
8 √3 + 1 8(2) 16

C. Heronian Triangles

Example 10.26
Δ𝐴𝐵𝐶 has side lengths 𝐴𝐵 = 15, 𝐵𝐶 = 13, 𝐶𝐴 = 4. AD is the bisector of ∠𝐴, intersecting BC at D. Find the area of
Δ𝐴𝐵𝐷.

Example 10.27
Δ𝐴𝐵𝐶 has side lengths 𝐴𝐵 = 17, 𝐵𝐶 = 10, 𝐶𝐴 = 9. AD is the bisector of ∠𝐴, intersecting BC at D. Find the area of
Δ𝐴𝐷𝐶.

Example 10.28
Δ𝐴𝐵𝐶 has side lengths 𝐴𝐵 = 20, 𝐵𝐶 = 25, 𝐶𝐴 = 3. AD is the bisector of ∠𝐴, intersecting BC at D. Find the area of
Δ𝐴𝐷𝐶.

D. Multiple Triangles

Example 10.29
Equilateral Δ𝐴𝐵𝐶 has side length 1. BC is extended to D such that 𝐵𝐷 = 6. The angle bisector from B intersects
AC at Y and the perpendicular bisector of BD at X. Find the area of quadrilateral CDXY.

P a g e 104 | 152
Get all the files at: https://bit.ly/azizhandouts
Aziz Manva ([email protected])

Example 10.30
∠𝐴𝐵𝐶 is 120°, with 𝐴𝐵 = 𝜋, 𝐵𝐶 = 1. Δ𝐷𝐵𝐶 is equilateral with side length 1. Point 𝑋 is on the intersection of 𝐵𝐷
and 𝐴𝐶. Find the area of Δ𝐷𝑋𝐶 in terms of the area of Δ𝐴𝐵𝐶.

See diagram (not drawn to scale).


1 1
∠𝐷𝐵𝐶 = 60° = × 120° = ∠𝐴𝐵𝐶
2 2
Hence, BD is the angle bisector of ∠𝐴𝐵𝐶.

From the property above:


[𝐴𝑋𝐵] 𝐴𝐵 𝜋
= =
[𝑋𝐵𝐶] 𝐵𝐶 1
Applying the ratio above:
[𝐴𝐵𝐶]
[𝑋𝐵𝐶] =
1+𝜋
Since Δ𝐷𝐵𝐶 is equilateral, its area is:
√3 2 √3 √3
[𝐷𝐵𝐶] = 𝑠 = × 12 =
4 4 4
And, finally,
√3 [𝐴𝐵𝐶]
[𝐷𝑋𝐶] = [𝐷𝐵𝐶] − [𝑋𝐵𝐶] = −
4 1+𝜋

E. Playing with Ratios

10.31:

Example 10.32
In right Δ𝐴𝐵𝐶, right-angled at B, 𝐴𝐵 = 8, and the hypotenuse is 2 units longer than 𝐵𝐶. If AD is the angle
bisector from A, then find the area of Δ𝐴𝐷𝐶.

82 + 𝑥 2 = (𝑥 + 2)2 ⇒ 64 + 𝑥 2 = 𝑥 2 + 4𝑥 + 4 ⇒ 𝑥 = 15

𝐴𝐷 𝐷𝐶 𝐴𝐷 8
= ⇒ = ⇒ 𝐴𝐷: 𝐷𝐶 = 8: 17
8 17 𝐷𝐶 17

P a g e 105 | 152
Get all the files at: https://bit.ly/azizhandouts
Aziz Manva ([email protected])

17 51
𝐷𝐶 = 15 × =
25 5
1 51 204
[𝐴𝐷𝐶] = × 8 × =
2 5 5

Example 10.33
Triangle 𝐴𝐵𝐶 has a right angle at 𝐵, 𝐴𝐵 = 1, and 𝐵𝐶 = 2. The angle bisector of
∠𝐴 intersects side 𝐵𝐶 at 𝐷. What is 𝐵𝐷? (AMC 10B 2009/20)

In Δ𝐴𝐵𝐶, by Pythagorean Theorem:


𝐴𝐶 = √𝐴𝐵2 + 𝐵𝐶 2 = √12 + 22 = √5
Now,
𝐴𝐵 = 1, 𝐴𝐶 = √5
Hence, by the angle bisector theorem,
𝐵𝐷 𝐴𝐵
=
𝐵𝐶 𝐴𝐵 + 𝐴𝐶
𝐵𝐶 2 2 √5 − 1 2(√5 − 1) √5 − 1
𝐵𝐷 = × 𝐴𝐵 = ×1= × = =
𝐴𝐵 + 𝐴𝐶 √5 + 1 √5 + 1 √5 − 1 4 2

10.4 Equilateral Triangles


A. Basics

Pending
Proof 10.34
Show, using the properties of the angle bisector (and without using congruence), that the angle bisector in an
equilateral triangle is also the median and the altitude.

Draw equilateral Δ𝐴𝐵𝐶, and construct the angle bisector from A.


𝐵𝐴𝐶 60
∠𝐵𝐴𝑋 = ∠ = = 30°
2 2
∠𝐶𝑋𝐴 = ∠𝐵𝑋𝐴 = 180 − 30 − 60 = 90° ⇒ 𝐴𝑋 𝑖𝑠 𝑎𝑙𝑡𝑖𝑡𝑢𝑑𝑒

By Angle Bisector Theorem:


𝐵𝑋 𝐶𝑋 𝐵𝑋 𝐶𝑋
= ⇒ = ⇒ 𝐵𝑋 = 𝐶𝑋 ⇒ 𝐴𝑋 𝑖𝑠 𝑚𝑒𝑑𝑖𝑎𝑛
𝐴𝐵 𝐴𝐶 1 1

Pending
Example 10.35
Equilateral Δ𝐴𝐵𝐶 has side length 1. AX is an angle bisector for ∠𝐵𝐴C, intersecting BC at X. BY is the angle
bisector for ∠𝐴𝐵𝐶 intersecting AX at Y. Find XY and 𝑌𝐴

B. 𝟏𝟓 − 𝟕𝟓 − 𝟗𝟎 Triangles (Optional)
We have seen how equilateral triangles can be used to derive the value of ratios in special triangles such as the
45 − 45 − 90 triangle and the 30 − 60 − 90 triangle. These two triangles are encountered frequently.
A less common triangle is the 15 − 75 − 90 triangle. We now derive the ratios of the sides in such a triangle.
(Warning: Heavy use of radicals).

P a g e 106 | 152
Get all the files at: https://bit.ly/azizhandouts
Aziz Manva ([email protected])

Challenge 10.36
Equilateral Δ𝐴𝐵𝐶 has side length 1. 𝐴𝑋 is the angle bisector for ∠𝐵𝐴𝐶 , intersecting 𝐵𝐶 at 𝑋. 𝐴𝑍 is the angle
bisector for ∠𝑋𝐴𝐶 intersecting XC at Z. Find 𝑋𝑍.

Since Δ𝐴𝐵𝐶 is equilateral:


√3 √3
𝐴𝑋 = 𝐻𝑒𝑖𝑔ℎ𝑡 = 𝑠=
2 2
By Angle Bisector Theorem:
√3
𝑋𝑍: 𝑍𝐶 = 𝐴𝑋: 𝐴𝐶 = :1
2
Since AX is median:
1 1
𝑋𝐶 = 𝑠 =
2 2

We need to divide XC in the ratio 𝑋𝑍: 𝑍𝐶:


√3 √3
1 2 4 √3 2 √3(2 − √3) 2√3 − 3
𝑋𝑍 = × = = × = =
2 √3 2 + √3 4 2 + √3 2(4 − 3) 2
1+ 2 2

(Continuation) Challenge 10.37


Show that
3 − √3
𝐴𝑍 =
√2

By the Pythagorean Theorem, in Δ𝐴𝑋𝑍:


2 2
2 2
2√3 − 3
2 √3 12 − 12√3 + 9 3 24 − 12√3
𝐴𝑍 = 𝑋𝑍 + 𝐴𝑋 = ( ) +( ) = + = = 6 − 3√3
2 2 4 4 4
2
3 − √3 9 − 6√3 + 3 12 − 6√3 3 − √3
( ) = = = 6 − 3√3 = 𝐴𝑍 2 ⇒ 𝐴𝑍 =
√2 2 2 √2

(Continuation) Challenge 10.38


Find the ratios
𝑋𝑍 𝐴𝑋 𝑋𝑍
, ,
𝐴𝑍 𝐴𝑍 𝐴𝑋

𝑋𝑍
=
𝐴𝑍
𝐴𝑋
=
𝐴𝑍
𝑋𝑍
=
𝐴𝑋

(Continuation) Challenge 10.39


Find the value of AZ by finding the square root of 𝐴𝑍 2 using an algebraic method (Not by squaring the given
value, and not by trial and error).

2
√6 − 3√3 = (𝑎 − 𝑏√3)

P a g e 107 | 152
Get all the files at: https://bit.ly/azizhandouts
Aziz Manva ([email protected])

Square both sides:


6 − 3√3 = 𝑎2 − 2𝑎𝑏√3 + 3𝑏 2

By the method of undetermined coefficients:


3
−2𝑎𝑏 = −3 ⇒ 𝑏 =
2𝑎
3 2 27
𝑎2 + 3𝑏 2 = 6 ⇒ 𝑎2 + 3 ( ) = 6 ⇒ 𝑎2 + 2 = 6 ⇒ 4𝑎4 − 24𝑎2 + 27 = 0
2𝑎 4𝑎

This is a disguised quadratic. Use a change of variable. Let 𝑥 = 𝑎2 :


4𝑥 2 − 24𝑥 + 27 = 0
2
4𝑥 − 6𝑥 − 18𝑥 + 27 = 0
2𝑥(2𝑥 − 3) − 9(2𝑥 − 3) = 0
(2𝑥 − 3)(2𝑥 − 9) = 0
3 9 3 9 3 3
𝑥 ∈ { , } ⇒ 𝑎2 ∈ { , } ⇒ 𝑎 ∈ {±√ , ± }
2 2 2 2 2 √2

We now have four possible values to check. We could start with any of them, but note that one value has a √3 in
it, and we already have a √3 in the coefficient of b.
So, start with the positive value of 𝑎 in the other two options:
3 3 3 1 3 1 3 − √3
𝑎= ⇒ 𝑏= ⇒𝑏= ⇒ (𝑎 − 𝑏√3) = ( + √3) =
√2 √2 2 √2 √2 √2 √2
Which we already know from above works.

P a g e 108 | 152
Get all the files at: https://bit.ly/azizhandouts
Aziz Manva ([email protected])

11. INCENTRE AND INRADIUS


11.1 Incenter
A. Definition
We first look at the definition of incircle, and incentre and inradius. Then, we use the definitions we just learned
to do some angle chasing.

11.1: Incircle and incentre


➢ The circle internally tangent to the three sides of a triangle is called its
incircle.
➢ The center of the incircle is called the incenter.
➢ The angle bisectors of a triangle are concurrent (meet) at its incenter.

In the diagram alongside:


➢ The circle is the incircle for Δ𝐴𝐵𝐶.
➢ The center of the circle (𝐼), is the incenter for Δ𝐴𝐵𝐶

B. Angle Chasing

Example 11.2
Δ𝐴𝐵𝐶 has incenter 𝐼. ∠𝐴 = ∠𝐴𝐵𝐼 = 40°. Find ∠𝐵𝐼𝐶.

Draw =Δ𝐴𝐵𝐶 with incentre I and note that 𝐵𝐼 is the bisector of ∠𝐴𝐵𝐶.
∠𝐴𝐵𝐶 = 2 × ∠𝐴𝐵𝐼 = 80
By sum of angles of a triangle:
∠𝐴𝐶𝐵 = 180 − ∠𝐴 − ∠𝐴𝐵𝐶 = 180 − 40 − 80 = 60
IC is the bisector of ∠𝐴𝐶𝐵:
60
∠𝐼𝐶𝐵 = = 30
2
By sum of angles of a triangle:
∠𝐵𝐼𝐶 = 180 − ∠𝐼𝐵𝐶 − ∠𝐼𝐶𝐵 = 180 − 40 − 30 = 110

Example 11.3
Δ𝐴𝐵𝐶 is an isosceles triangle with vertex angle ∠𝐴 = 100° and incentre 𝐼. What is the measure of ∠𝐵𝐼𝐶?

By Base Angles of an Isosceles Triangle:


180 − 100 80
∠𝐴𝐵𝐶 = ∠𝐴𝐶𝐵 = = = 40
2 2
Since 𝐼 is the incentre, 𝐼𝐵 and 𝐼𝐶 are angle bisectors of ∠𝐵 and ∠𝐶 respectively:
40
∠𝐼𝐵𝐶 = ∠𝐼𝐶𝐵 = = 20
2
By sum of angles of a triangle:
∠𝐵𝐼𝐶 = 180 − 20 − 20 = 140

Example 11.4
In Δ𝐴𝐵𝐶 with incentre 𝐼 we are given ∠𝐵𝐼𝐴 = 140°, ∠𝐼𝐴𝐶 = 30°. Find ∠𝐵𝐼𝐶.

P a g e 109 | 152
Get all the files at: https://bit.ly/azizhandouts
Aziz Manva ([email protected])

IA is an angle bisector
∠𝐵𝐴𝐼 = ∠𝐼𝐴𝐶 = 30°
By sum of angles of a triangle:
∠𝐴𝐵𝐼 = 180 − 140 − 30 = 10
IB is a bisector:
∠𝐴𝐵𝐶 = 2 × 𝐴𝐵𝐼 = 2 × 10 = 20
By sum of angles of a triangle:
∠𝐴𝐶𝐵 = 180 − 60 − 20 = 100
By sum of angles of a triangle:
∠𝐵𝐼𝐶 = 180 − 10 − 50 = 120

C. More Angle Chasing

Pending
Example 11.5
O is the incenter of Δ𝐴𝐵𝐶 with ∠𝐴 = 40°, and ∠𝐵 = 30°. Find ∠𝐴𝐶𝑂.

By Angles in a Triangle:
∠𝐶 = 180 − 40 − 30 = 110°
Since the incentre is the intersection of the angle bisectors, OC is the angle bisector
of ∠𝐶:
110
∠𝑂𝐶𝑂 = = 55°
2

Pending
Example 11.6
O is the incenter of isosceles Δ𝐴𝐵𝐶 with ∠𝐴 = 30°. Find the supplement of the sum of the possible values of
∠𝐴𝐶𝑂.

120
∠𝐵 = 30° ⇒ ∠𝐶 = 120 ⇒ ∠𝐴𝐶𝑂 = = 60°
2
30
∠𝐶 = 30° ⇒ ∠𝐴𝐶𝑂 = = 15°
2
75
∠𝐵 = ∠𝐶 = 75° ⇒ ∠𝐴𝐶𝑂 = = 37.5°
2

𝑆𝑢𝑚 = 60 + 15 + 37.5 = 112.5


𝑆𝑢𝑝𝑝𝑙𝑒𝑚𝑒𝑛𝑡 = 180 − 112.5 = 67.5

Pending
Example 11.7
O is the incenter of isosceles Δ𝐴𝐵𝐶 with ∠𝐴 = 𝑥°. The supplement of the sum of the possible values of ∠𝐴𝐶𝑂 is
60°. Find 𝑥.

∠𝐶 is the vertex angle:


∠𝐶 = 180 − 2𝑥 ⇒ ∠𝐴𝐶𝑂 = 90 − 𝑥
∠𝐵 is the vertex angle:

P a g e 110 | 152
Get all the files at: https://bit.ly/azizhandouts
Aziz Manva ([email protected])

𝑥
∠𝐶 = 𝑥 ⇒ ∠𝐴𝐶𝑂 =
2
∠𝐴 is the vertex angle:
180 − 𝑥 𝑥 𝐶 𝑥
∠𝐶 = = 90 − ⇒ ∠𝐴𝐶𝑂 = ∠ = 45 −
2 2 2 4
𝑥 𝑥 3𝑥 3𝑥
90 − 2 + + 45 − = 120 ⇒ 135 − = 120 ⇒ 15 = ⇒ 𝑥 = 20
2 4 4 4

D. Incentre divides Angle Bisectors

11.8: Incentre divides Angle Bisectors


The incenter of a triangle divides the internal angle bisectors in the ratio of the
sum of adjacent sides and the opposite side.
∵ Δ𝐴𝐵𝐶 with incentre I
𝐴𝐼 𝑏 + 𝑐 𝐵𝐼 𝑐 + 𝑎 𝐶𝐼 𝑎 + 𝑏
= , = , =
𝐼𝐷 𝑎 𝐼𝐸 𝑏 𝐼𝐹 𝑐

Example 11.9
Consider Δ𝐴𝐵𝐶, with sides 𝐴𝐵 = 3, 𝐵𝐶 = 4 and 𝐶𝐴 = 5. Angle bisectors 𝐴𝐷, 𝐵𝐸, and CF are drawn, intersecting
sides 𝐵𝐶, 𝐶𝐴, and 𝐴𝐵 at 𝐷, 𝐸, and 𝐹 respectively. Find:
𝐴𝐼
A. 𝐼𝐷
𝐵𝐼
B. 𝐼𝐸
𝐶𝐼
C. 𝐼𝐹

𝐴𝐼 𝑏 + 𝑐 5 + 3 8
= = = =2
𝐼𝐷 𝑎 4 4
𝐵𝐼 𝑐 + 𝑎 3 + 4 7
= = =
𝐼𝐸 𝑏 5 5
𝐶𝐼 𝑎 + 𝑏 5 + 4 9
= = = =3
𝐼𝐹 𝑐 3 3

Example 11.10
Back-Calculations

11.2 Inradius: Definition


A. Inradius

11.11: Inradius
The radius of an incircle is called its inradius.

Example 11.12
In Δ𝐴𝐵𝐶 with incentre 𝐼, the distance from 𝐼 to side 𝐴𝐵 is 4 𝑐𝑚. What is the sum of the distances from 𝐼 to 𝐵𝐶
and 𝐶𝐴.

P a g e 111 | 152
Get all the files at: https://bit.ly/azizhandouts
Aziz Manva ([email protected])

The circle is tangent to the three sides of the triangle. Hence, the distance
from the center to the sides is equal to the radius.

Radius is always equal. Hence, required distance is:


4+4=8
B. Isosceles Triangles

Example 11.13
In Isosceles triangle 𝑋𝑌𝑍, the base 𝑌𝑍 has length 10, and the length of the distance from the incentre 𝐼 to the
vertex 𝑍 is 8. What is the inradius?

Draw a diagram and note that 𝐼𝑍 = 8.


In Isosceles ΔX𝑌𝑍
∠𝑋𝑌𝑍 = ∠𝑋𝑍𝑌

Since the incentre is the intersection of the angle bisectors:


1
∠𝐼𝑌𝑍 = ∠𝐼𝑍𝑌 = ∠𝑋𝑌𝑍 ⇒ Δ𝐼𝑌𝑍 𝑖𝑠 𝑖𝑠𝑜𝑠𝑐𝑒𝑙𝑒𝑠
2
In Isosceles Δ𝐼𝑌𝑍 drop a perpendicular (which is also the median) to side YZ:
1 1
𝑀𝑍 = × 𝑌𝑍 = × 10 = 5
2 2

In Right Triangle 𝐼𝑀𝑍, by the Pythagorean Theorem:


𝐼𝑀 = √𝐼𝑍 2 − 𝑀𝑍 2 = √82 − 52 = √64 − 25 = √39

And since IM is perpendicular to YZ, it is also the inradius


𝐼𝑛𝑟𝑎𝑑𝑖𝑢𝑠 𝑜𝑓 Δ𝑋𝑌𝑍 = 𝐼𝑀 = √39
C. Equilateral Triangles

11.14: Incentre of Equilateral Triangle


If the incentre of a triangle is equidistant from its vertices, then the triangle
is an equilateral triangle.

In the diagram, if:


𝐼𝐴 = 𝐼𝐵 = 𝐼𝐶 ⇒ Δ𝐴𝐵𝐶 𝑖𝑠 𝑒𝑞𝑢𝑖𝑙𝑎𝑡𝑒𝑟𝑎𝑙

Example 11.15
The distance of the incentre of a triangle from each of its vertices is 1 cm. What is the area of the triangle?

P a g e 112 | 152
Get all the files at: https://bit.ly/azizhandouts
Aziz Manva ([email protected])

Since the distance to each of its vertices is equal, the triangle is


equilateral. Draw the triangle (see diagram).
𝐼𝐴 = 𝐼𝐵 = 1

Since the above distances are equal, we get an isosceles triangle.


The thought process needed here is similar to the one used when we
find the area of an isosceles triangle.

In Isosceles Δ𝐼𝐴𝐵:
Draw the altitude from 𝐼 intersecting side 𝐴𝐵 at 𝑃

In Δ𝐼𝐴𝑃:
60
∠𝐼𝐴𝑃 = = 30, ∠𝐼𝑃𝐴 = 90, ∠𝐴𝐼𝑃 = 180 − 90 − 30 = 60
2

Hence,
Δ𝐼𝐴𝑃 𝑖𝑠 𝑎 30 − 60 − 90 𝑡𝑟𝑖𝑎𝑛𝑔𝑙𝑒

√3
In a 30 − 60 − 90 𝑡𝑟𝑖𝑎𝑛𝑔𝑙𝑒, the side opposite 60° is times the hypotenuse
2
√3 √3 √3
𝐴𝑃 = × 𝐴𝐼 = ×1 =
2 2 2

In an Isosceles triangle, the altitude is also the median:


√3
𝐴𝐵 = 2 × 𝐴𝑃 = 2 × = √3
2
And, finally, now that we have the side, we can find the area using the formula for the area of an equilateral
triangle:
√3 2 √3 2 3√3
𝐴= 𝑠 = (√3) =
4 4 4

11.16: Incentre of Equilateral Triangle


Converse of Previous: If a triangle is equilateral, the incentre is equidistant from its vertices.

Example 11.17
Dropping a perpendicular from the incentre of an equilateral triangle to each of the three sides creates six
smaller triangles. What is the area of each smaller triangle if the side of the equilateral triangle is 1 cm?

Draw Δ𝐴𝐵𝐶 with incenter I. Connect I with the vertices.


In Δ𝐼𝐴𝐵, draw altitude IP intersecting AB at P.
1 1 1
𝐴𝑃 = 𝐴𝐵 = × 1 =
2 2 2
In Δ𝐼𝐴𝑃:
√3 √3 1 1 1 √3 √3
ℎ𝑦𝑝 × = 𝐴𝑃 ⇒ ℎ𝑦𝑝 × = ⇒ ℎ𝑦𝑝 = = × =
2 2 2 √3 √3 √3 3
In Δ𝐼𝐴𝑃:
1 1 √3 √3
Side opposite 30° = 𝐼𝑃 = × ℎ𝑦𝑝 = × =
2 2 3 6
Area of Δ𝐼𝐴𝑃

P a g e 113 | 152
Get all the files at: https://bit.ly/azizhandouts
Aziz Manva ([email protected])

1 1 1 √3 √3
= ℎ𝑏 = × × =
2 2 2 6 24

11.18: Combining Statements and Converses


The incentre of a triangle is equidistant from its vertices 𝑖𝑓 𝑎𝑛𝑑 𝑜𝑛𝑙𝑦 𝑖𝑓 the triangle is equilateral.

If both a statement, and its converse are true, they can be combined into a single statement using
𝑖𝑓 𝑎𝑛𝑑 𝑜𝑛𝑙𝑦 𝑖𝑓.

11.3 Calculating Inradius


A. Calculating Inradius

11.19: Inradius
In a triangle with lengths of sides 𝒂, 𝒃, and 𝒄:
Δ
𝐼𝑛𝑟𝑎𝑑𝑖𝑢𝑠 = 𝑟 =
𝑠

Recall that when we learnt Heron’s Formula, we had the following:


Area of a Triangle = Delta = Δ = √𝑠(𝑠 − 𝑎)(𝑠 − 𝑏)(𝑠 − 𝑐)
𝑎+𝑏+𝑐
𝑠 = Semiperimeter =
2

Consider Δ𝐴𝐵𝐶 with incentre 𝐼. We must have:


𝐴𝑟𝑒𝑎(Δ𝐴𝐵𝐶) = 𝐴𝑟𝑒𝑎(Δ𝐼𝐵𝐶) + 𝐴𝑟𝑒𝑎(Δ𝐼𝐴𝐶) + 𝐴𝑟𝑒𝑎(Δ𝐼𝐴𝐵) → 𝑰

Segment BC is tangent to the incircle, and since a radius is


perpendicular to the tangent at the point of tangency, we know that
𝐵𝐷 ⊥ 𝐼𝐷, which tells us that:
1 1 1
𝐴𝑟𝑒𝑎(Δ𝐼𝐵𝐶) = ℎ𝑏 = ∙ 𝐼𝐷 ∙ 𝐵𝐶 = 𝑟𝑎
2 2 2
Similarly:
1 1
𝐴𝑟𝑒𝑎(Δ𝐼𝐴𝐶) = 𝑟𝑏, 𝐴𝑟𝑒𝑎(Δ𝐼𝐴𝐵) = 𝑟𝑐
2 2

Substitute the above three in I:


1 1 1 𝑎+𝑏+𝑐 Δ
𝐴𝑟𝑒𝑎(Δ𝐴𝐵𝐶) = Δ = 𝑟𝑎 + 𝑟𝑏 + 𝑟𝑐 = 𝑟 ( ) = 𝑟𝑠 ⇒ Δ = 𝑟𝑠 ⇒ 𝑟 =
2 2 2 2 s

Example 11.20
Calculation of area given inradius and lengths of sides

Example 11.21
The sides of a triangle are 13, 14 and 15. Find its inradius.

𝑎 + 𝑏 + 𝑐 13 + 14 + 15 42
s= = = = 21
2 2 2

Δ = √𝑠(𝑠 − 𝑎)(𝑠 − 𝑏)(𝑠 − 𝑐) = √21(8)(7)(6) = √3 × 7(23 )(7)(2 × 3) = √32 × 72 (24 ) = 84

P a g e 114 | 152
Get all the files at: https://bit.ly/azizhandouts
Aziz Manva ([email protected])

Δ 84
𝑟= = =4
𝑠 21

11.4 Tangents
A. Lengths of tangents to incircle
Given a triangle with an incircle, we can consider the length of the segments to the circle as tangents. Given the
lengths of the sides, it is possible to calculate the lengths of the segments using the properties of tangents.

11.22: Tangent from a point to a circle


Tangents from a point to a circle are congruent.

Example 11.23
Tangents are drawn to a circle from points A, B and C. The tangents from point A touch the circle at X and Z. The
tangents from point B touch the circle at X and Y. The tangents from point C touch the circle at Y and Z. Draw a
diagram and identify the congruent tangents.

𝐵𝑋 = 𝐵𝑌 = 𝑥
𝐶𝑌 = 𝐶𝑍 = 𝑦
𝐴𝑋 = 𝐴𝑍 = 𝑧

Example 11.24
The incircle of Δ𝐴𝐵𝐶 is drawn touching it at points 𝑋, 𝑌 and 𝑍 respectively.
In other words, the circle is tangent to all three sides of the triangle. Given that A𝐵 = 4, 𝐵𝐶 = 5, 𝐴𝐶 = 6, find the
lengths of the tangents drawn to the circle from the vertices of Δ𝐴𝐵𝐶.

Tangents drawn from a point to a circle are congruent.

𝐴𝐵 = 𝑥 + 𝑧 = 4
𝐵𝐶 = 𝑥 + 𝑦 = 5
𝐶𝐴 = 𝑦 + 𝑧 = 6
Add the three equations:
2(𝑥 + 𝑦 + 𝑧) = 15 ⇒ 𝑥 + 𝑦 + 𝑧 = 7.5

𝐶𝑍 = 𝐶𝑌 = 𝑦 = 3.5
𝐴𝑋 = 𝐴𝑍 = 𝑧 = 2.5
𝐵𝑋 = 𝐵𝑌 = 𝑥 = 1.5

11.25: Lengths of Tangents


In Δ𝐴𝐵𝐶 with sides in the usual notation:
𝐵𝐶 = 𝑎, 𝐴𝐶 = 𝑏, 𝐴𝐵 = 𝑐
The tangents drawn from points 𝐴, 𝐵 and 𝐶 have length:
𝑧 = 𝑠 − 𝑎, 𝑦 = 𝑠 − 𝑐, 𝑥 = 𝑠−𝑏

Tangents drawn from a point to a circle are congruent.


𝑥
⏟+ 𝑦 = 𝑎
𝐸𝑞𝑢𝑎𝑡𝑖𝑜𝑛 𝐼

P a g e 115 | 152
Get all the files at: https://bit.ly/azizhandouts
Aziz Manva ([email protected])

𝑧+𝑦 =𝑏

𝐸𝑞𝑢𝑎𝑡𝑖𝑜𝑛 𝐼𝐼
𝑥+𝑧 =𝑐

𝐸𝑞𝑢𝑎𝑡𝑖𝑜𝑛 𝐼𝐼𝐼
Add the three equations:
𝑎+𝑏+𝑐
2(𝑥 + 𝑦 + 𝑧) = 𝑎 + 𝑏 + 𝑐 ⇒ 𝑥 + 𝑦 + 𝑧 = ⇒⏟
𝑥+𝑦+𝑧 =𝑠
2
𝐸𝑞𝑢𝑎𝑡𝑖𝑜𝑛 𝐼𝑉

Subtract Equation I from Equation IV:


𝑧 =𝑠−𝑎
Subtract Equation II from Equation IV:
𝑥 =𝑠−𝑏
Subtract Equation III from Equation IV:
𝑦 = 𝑠−𝑐

Pending
B. Triangle formed by Incircle at Points of Tangency
The incircle touches the triangle at three points, where the sides are tangent to the circle. These three points
can be joined to form a triangle.

Example 11.26
The incircle to Δ𝐴𝐵𝐶 is tangent to 𝐴𝐵 at 𝑋, 𝐵𝐶 at 𝑌 and 𝐴𝐶 at 𝑍. Find the angles of Δ𝑋𝑌𝑍 if ∠𝐴 = 20, ∠𝐵 = 40.

11.27: Angles in “Inner” Triangle


Let the incircle of Δ𝐴𝐵𝐶 be tangent to the sides at 𝑋, 𝑌 and 𝑍
respectively. Then the angles of Δ𝑋𝑌𝑍 are the averages of the angles
of Δ𝐴𝐵𝐶, taken two at a time.

180 − ∠𝐴
∠𝐴𝑋𝑍 = ∠𝐴𝑍𝑋 =
2
180 − ∠𝐵
∠𝐵𝑋𝑌 = ∠𝐵𝑌𝑋 =
2
180 − ∠𝐶
∠𝐶𝑌𝑍 = ∠𝐶𝑍𝑌 =
2

180 − ∠𝐴 180 − ∠𝐵 ∠𝐴 + ∠𝐵
∠𝑍𝑋𝑌 = 180 − − =
2 2 2
180 − ∠𝐵 180 − ∠𝐶 ∠𝐵 + ∠𝐶
∠𝑍𝑌𝑋 = 180 − − =
2 2 2
180 − ∠𝐴 180 − ∠𝐶 ∠𝐴 + ∠𝐶
∠𝑋𝑍𝑌 = 180 − − =
2 2 2

Example 11.28
The incircle to Δ𝐴𝐵𝐶 is tangent to 𝐴𝐵 at 𝑋, 𝐵𝐶 at 𝑌 and 𝐴𝐶 at 𝑍. If ∠𝐴 = 30, ∠𝐵 = 50. Find the angles of Δ𝑋𝑌𝑍.

P a g e 116 | 152
Get all the files at: https://bit.ly/azizhandouts
Aziz Manva ([email protected])

∠𝐶 = 180 − 30 − 50 = 100

30 + 50 80
∠𝑍𝑋𝑌 = = = 40
2 2
50 + 100 150
∠𝑋𝑌𝑍 = = = 75
2 2
30 + 100 130
∠𝑋𝑍𝑌 = = = 65
2 2

Example 11.29
The incircle to Δ𝐴𝐵𝐶 is tangent to 𝐴𝐵 at 𝑋, 𝐵𝐶 at 𝑌 and 𝐴𝐶 at 𝑍. If ∠𝐴 = 20, find ∠𝑋𝑌𝑍.

∠𝐵 + ∠𝐶 180 − ∠𝐴 160
∠𝑋𝑌𝑍 = = = = 80
2 2 2

Pending
C. Back Calculations

11.30: Circumcircle
Circumcircle of a triangle is the circle that passes through the vertices of the triangle.

Example 11.31
The circumcircle of Δ𝑋𝑌𝑍 is inscribed in Δ𝐴𝐵𝐶. If ∠𝑋 = 30, ∠𝑌 = 70, find the angles of Δ𝐴𝐵𝐶.

By angles in a triangle:
∠𝑍 = 180 − 30 − 70 = 80
Let:
∠𝐴 = 𝑎, ∠𝐵 = 𝑏, ∠𝐶 = 𝑐 ⇒ ⏟𝑎 + 𝑏 + 𝑐 = 180
𝑬𝒒𝒖𝒂𝒕𝒊𝒐𝒏 𝑰
Then, by using Equation I:
𝑎+𝑏
= 30 ⇒ 𝑎 + 𝑏 = 60 ⇒ 𝑐 = 120
2
𝑏+𝑐
= 70 ⇒ 𝑏 + 𝑐 = 140 ⇒ 𝑎 = 40
2
𝑎+𝑐
= 80 ⇒ 𝑎 + 𝑐 = 160 ⇒ 𝑏 = 20
2

Example 11.32
𝐶ℎ𝑒𝑐𝑘𝑖𝑛𝑔 𝐹𝑒𝑎𝑠𝑏𝑖𝑙𝑖𝑡𝑦
Is it possible for the circumcircle of a right-angled triangle to be inscribed in a triangle?

P a g e 117 | 152
Get all the files at: https://bit.ly/azizhandouts
Aziz Manva ([email protected])

Suppose
∠𝐴 + ∠𝐵
∠𝑍𝑋𝑌 = 90 ⇒ = 90 ⇒ ∠𝐴 + ∠𝐵 = 180 ⇒ 𝑁𝑜𝑡 𝑃𝑜𝑠𝑠𝑖𝑏𝑙𝑒
2

Pending
D. Arithmetic Sequences (Optional)3

11.33: Arithmetic Sequence


If the difference between two consecutive elements of a sequence is constant, then it is an arithmetic sequence.
It is of the form:
… , 𝑎 − 2𝑑, 𝑎 − 𝑑, 𝑎, 𝑎 + 𝑑, 𝑎 + 2𝑑, …
𝑎 = 𝑚𝑖𝑑𝑑𝑙𝑒 𝑡𝑒𝑟𝑚
𝑑 = 𝑐𝑜𝑚𝑚𝑜𝑛 𝑑𝑖𝑓𝑓𝑒𝑟𝑒𝑛𝑐𝑒

Example 11.34
In Δ𝐴𝐵𝐶, the measures of the angles form an arithmetic sequence such that the measure of the smallest angle is
equal to the common difference of the arithmetic sequence. Also, ∠𝐴 < ∠𝐵 < ∠𝐶. The incircle to Δ𝐴𝐵𝐶 is
tangent to 𝐴𝐵 at 𝑋, 𝐵𝐶 at 𝑌 and 𝐴𝐶 at 𝑍. Find the angles of Δ𝑋𝑌𝑍.

For any triangle with angles in arithmetic sequence, we must have:


𝑎 − 𝑑 + 𝑎 + 𝑎 + 𝑑 = 180 ⇒ 3𝑎 = 180 ⇒ 𝑎 = 60

For the triangle in this question, we have:


𝑎 − 𝑑 = 𝑑 ⇒ 𝑎 = 2𝑑 ⇒ 2𝑑 = 60 ⇒ 𝑑 = 30

Hence, the angles are:


∠𝐴 = 30°, ∠𝐵 = 60°, ∠𝐶 = 90°

30 + 60 90
∠𝑍𝑋𝑌 = = = 45
2 2
60 + 90 150
∠𝑋𝑌𝑍 = = = 75
2 2
30 + 90 120
∠𝑋𝑍𝑌 = = = 60
2 2

11.35: Arithmetic Sequence


The incircle to Δ𝐴𝐵𝐶 is tangent to 𝐴𝐵 at 𝑋, 𝐵𝐶 at 𝑌 and 𝐴𝐶 at 𝑍.
If in Δ𝐴𝐵𝐶, the measures of the angles form an arithmetic sequence with common difference d, then in Δ𝑋𝑌𝑍,
𝑑
the angles form an arithmetic sequence with common difference 2 .

That is,
𝐼𝑓 ∠𝐴 = 𝑎 − 𝑑, ∠𝐵 = 𝑎, ∠𝐶 = 𝑎 + 𝑑, 𝑑>0
Then,
𝑑 𝑑
∠𝑍𝑋𝑌 = 𝑎 − , ∠𝑍𝑌𝑋 = 𝑎 + , ∠𝑋𝑍𝑌 = 𝑎
2 2

3
For the questions in this section, some familiarity with arithmetic sequences is desirable.

P a g e 118 | 152
Get all the files at: https://bit.ly/azizhandouts
Aziz Manva ([email protected])

∠𝐴 + ∠𝐵 𝑎 − 𝑑 + 𝑎 𝑑
∠𝑍𝑋𝑌 = = =𝑎−
2 2 2
∠𝐵 + ∠𝐶 𝑎 + 𝑎 + 𝑑 𝑑
∠𝑍𝑌𝑋 = = =𝑎+
2 2 2
∠𝐴 + ∠𝐶 𝑎 − 𝑑 + 𝑎 + 𝑑
∠𝑋𝑍𝑌 = = =𝑎
2 2

Example 11.36
Let triangle 𝑇𝑛+1 have its vertices at the points where the incircle of triangle 𝑇𝑛 is tangent to its sides.
Let the triplet 𝑡𝑛 represent the angles of 𝑇𝑛 . Given that 𝑡1 = (12,60,108), find
A. 𝑡2
B. 𝑡3
C. 𝑡𝑛

𝑡1 = (12,60,108) = (60 − 48,60,60 + 48)


𝑡2 = (60 − 24,60,60 + 24)
𝑡3 = (60 − 12,60,60 + 12)
.
.
.
48 48
𝑡𝑛 = (60 − 𝑛−1
, 60,60 + 𝑛−1 )
2 2
Example 11.37
59.9375, 60
Go outward forming triangles from incircle.
Angles of outermost triangle

1 1
𝑡1 = (60 − 4
, 60, 60 + 4 )
2 2
1 1
𝑡2 = (60 − 3
, 60, 60 + 3 )
2 2
1 1
𝑡3 = (60 − 2
, 60, 60 + 2 )
2 2
.
.
.
1 1
𝑡𝑛 = (60 − , 60, 60 + )
25−𝑛 25−𝑛
P a g e 119 | 152
Get all the files at: https://bit.ly/azizhandouts
Aziz Manva ([email protected])

1 1
60 − > 0 ⇒ 60 > ⇒ 60 > 2𝑛−5
25−𝑛 25−𝑛
32 < 60 < 64 ⇒ 25 < 60 < 26
𝑛 − 5 < 6 ⇒ 𝑛 < 11 ⇒ 𝑛 ≤ 10

11.5 Isosceles and Equilateral Triangles


A. Isosceles Triangles

11.38: Incentre of an Isosceles Triangle


The altitude to the base of an isosceles triangle passes through the incentre of the triangle.

Example 11.39
In Isosceles Δ𝑋𝑌𝑍, with 𝑋𝑌 = 𝑋𝑍, the incentre is at the point of concurrency of the
angle bisectors 𝑋𝐴, 𝑌𝐶 and 𝑍𝐵.
Explain why the incentre lies on the altitude and the median from vertex X.

In an isosceles triangle when drawn from congruent sides to the base:


𝑀𝑒𝑑𝑖𝑎𝑛 = 𝐴𝑙𝑡𝑖𝑡𝑢𝑑𝑒 = 𝐴𝑛𝑔𝑙𝑒 𝐵𝑖𝑠𝑒𝑐𝑡𝑜𝑟

Hence,
𝐼 𝑙𝑖𝑒𝑠 𝑜𝑛 𝑎𝑙𝑡𝑖𝑡𝑢𝑑𝑒 𝑎𝑠 𝑤𝑒𝑙𝑙 𝑎𝑠 𝑚𝑒𝑑𝑖𝑎𝑛.

Example 11.40
What is the area of a circle inscribed in a triangle with side lengths 5, 5 and 6?

In Isosceles Δ𝐼𝑌𝑍 drop a perpendicular (which is also the median) to side YZ:
1 1
𝑀𝑍 = × 𝑌𝑍 = × 6 = 3
2 2

1
𝑟= =
Δ 2×6×4 =
24 3
=
𝑠 1
× (5 + 5 + 6) 16 2
2
2
3 2 9
𝐴𝑟𝑒𝑎 𝑜𝑓 𝐼𝑛𝑐𝑖𝑟𝑐𝑙𝑒 = 𝜋𝑟 = 𝜋 ( ) = 𝜋 = 2.25𝜋
2 4
B. Equilateral Triangle

11.41: Inradius of equilateral triangle


An equilateral triangle with side length 𝑥 has
√3𝑥
𝐼𝑛𝑟𝑎𝑑𝑖𝑢𝑠 = 𝑟 =
6

P a g e 120 | 152
Get all the files at: https://bit.ly/azizhandouts
Aziz Manva ([email protected])

𝜋𝑥 2
𝐴𝑟𝑒𝑎 𝑜𝑓 𝐼𝑛𝑐𝑖𝑟𝑐𝑙𝑒 =
12

√3 2
Δ 𝑥 √3 2 2 √3𝑥
𝐼𝑛𝑟𝑎𝑑𝑖𝑢𝑠 = 𝑟 = = 4 = 𝑥 × =
𝑠 3𝑥 4 3𝑥 6
2
2
2 √3𝑥 3𝑥 2 𝜋𝑥 2
𝐴𝑟𝑒𝑎 𝑜𝑓 𝐼𝑛𝑐𝑖𝑟𝑐𝑙𝑒 = 𝜋𝑟 = 𝜋 ( ) = 𝜋( )=
6 36 12

Example 11.42
The number of inches in the perimeter of an equilateral triangle equals the number of square inches in the area
of its inscribed circle. What is the radius, in inches, of the circle?

The relation between the side of an equilateral triangle, and the inradius is:
√3𝑥
𝑟=
6
Find the perimeter of the triangle in terms of the radius:
6𝑟 6√3𝑟
𝑥= = = 2√3𝑟 ⇒ 𝑃 = 3𝑥 = 6√3𝑟
√3 3
6√3
𝐴 = 𝑃 ⇒ 𝜋𝑟 2 = 6√3𝑟 ⇒ 𝑟 =
𝜋
11.6 Right Triangles
A. Right Triangles
The formula for the inradius of a right triangle can be stated in terms of the hypotenuse. Knowing this result
can greatly simplify “advanced” problems, as we will see.

11.43: Inradius for a right triangle


The inradius of a right triangle is:
𝑎+𝑏−𝑐
𝑟 =𝑠−ℎ =
2
Where
ℎ = 𝑐 = ℎ𝑦𝑝𝑜𝑡𝑒𝑛𝑢𝑠𝑒

1 𝑎+𝑏+𝑐
Substitute Δ = 2 𝑎𝑏, 𝑠 = 2
to get:
1
Δ 𝑎𝑏 𝑎𝑏
𝑟= = 2 =
𝑠 𝑎+𝑏+𝑐 𝑎+𝑏+𝑐
2
We need 𝑎 + 𝑏 − 𝑐 in the numerator, so create it in the numerator:
𝑎𝑏 𝑎+𝑏−𝑐
×
𝑎+𝑏+𝑐 𝑎+𝑏−𝑐
Apply the formula for difference of squares in the denominator:
𝑎𝑏(𝑎 + 𝑏 − 𝑐) 𝑎𝑏(𝑎 + 𝑏 − 𝑐)
2 2
= 2
(𝑎 + 𝑏) − 𝑐 𝑎 + 2𝑎𝑏 + 𝑏 2 − 𝑐 2
But 𝑎2 + 𝑏 2 = 𝑐 2 ⇒ 𝑎2 + 𝑏 2 − 𝑐 2 = 0:
𝑎𝑏(𝑎 + 𝑏 − 𝑐) 𝑎 + 𝑏 − 𝑐
=
2𝑎𝑏 2

P a g e 121 | 152
Get all the files at: https://bit.ly/azizhandouts
Aziz Manva ([email protected])

Example 11.44
A right triangle has perimeter 32 and area 20. What is the length of its hypotenuse? (AMC 10A 2008/18)

Method I: Pythagoras Theorem


From the diagram, since the perimeter is 32:
𝑎 + 𝑏 + 𝑐 = 32 ⇒ 𝑐 = 32 − (𝑎 + 𝑏)
Since the area is 20:
1
𝑎𝑏 = 20 ⇒ 𝑎𝑏 = 40
2
By Pythagoras Theorem:
𝑎2 + 𝑏 2 = [32 − (𝑎 + 𝑏)]2
𝑎2 + 𝑏 2 = 1024 − 64(𝑎 + 𝑏) + (𝑎 + 𝑏)2
𝑎 + 𝑏 2 = 1024 − 64(𝑎 + 𝑏) + 𝑎2 + 2𝑎𝑏 + 𝑏 2
2

0 = 1024 − 64(𝑎 + 𝑏) + 2𝑎𝑏


Substitute 𝑎𝑏 = 40:
64(𝑎 + 𝑏) = 1024 + 80
𝑎 + 𝑏 = 17.25

𝑐 = 32 − (𝑎 + 𝑏) = 32 − 17.25 = 14.75

Method II: Inradius


Δ 20 20 20 5
𝑟= = = = =
𝑠 16 32⁄ 16 4
2
5 59
𝑟 = 𝑠 − ℎ ⇒ ℎ = 𝑠 − 𝑟 = 16 − = = 14.75
4 4

Example 11.45
Calculate the inradius of a triangle with sides 3,4, 5.

3+4−5 7−5 2
𝑟= = = =1
2 2 2

11.46: Inradius for a right triangle


The inradius of a right triangle is:
𝑎+𝑏−𝑐
𝑟 =𝑠−ℎ =
2
Where

ℎ = 𝑐 = ℎ𝑦𝑝𝑜𝑡𝑒𝑛𝑢𝑠𝑒

We can calculate the formula for inradius using the length of tangent
formula calculated above.

Δ𝐵𝐶𝐴 𝑖𝑠 𝑟𝑖𝑔ℎ𝑡 − 𝑎𝑛𝑔𝑙𝑒𝑑 ⇒ ∠𝐵𝐶𝐴 = 90°

Since the radius is perpendicular to tangent at point of tangency:


∠𝐼𝑋𝐶 = ∠𝐼𝑌𝐶 = 90°

P a g e 122 | 152
Get all the files at: https://bit.ly/azizhandouts
Aziz Manva ([email protected])

In Quadrilateral 𝐼𝑋𝐶𝑌, by sum of angles:


∠𝑋𝐼𝐶 = 360 − 90 − 90 − 90 = 90° ⇒ 𝐼𝑋𝐶𝑌 𝑖𝑠 𝑎 𝑟𝑒𝑐𝑡𝑎𝑛𝑔𝑙𝑒

In Quadrilateral 𝐼𝑋𝐶𝑌,
𝐼𝑋 = 𝐼𝑌 ⇒ 𝐴𝑑𝑗𝑜𝑖𝑛𝑖𝑛𝑔 𝑠𝑖𝑑𝑒𝑠 𝑎𝑟𝑒 𝑒𝑞𝑢𝑎𝑙 ⇒ 𝐼𝑋𝐶𝑌 𝑖𝑠 𝑎 𝑠𝑞𝑢𝑎𝑟𝑒.

As already shown when we calculated the formula for lengths of


tangents to incircle:
𝑟 = 𝐼𝑋 = 𝐶𝑌 = 𝑠 − ℎ

Example 11.47
Δ𝐵𝐶𝐴, drawn alongside, is a 3 − 4 − 5 right triangle. 𝐼 is the incenter.
Find the lengths of the tangents to the incircle.

Find the semi-perimeter


𝑎 + 𝑏 + 𝑐 3 + 4 + 5 12
𝑠= = = =6
2 2 2

Use the formula to find the lengths of the tangents:


𝐶𝑌 = 𝐶𝑋 = 𝑠 − 𝐴𝐵 = 6 − 5 = 1
𝐴𝑌 = 𝐴𝑍 = 𝑠 − 𝐵𝐶 = 6 − 3 = 3
𝐵𝑋 = 𝐵𝑍 = 𝑠 − 𝐶𝐴 = 6 − 4 = 2

Find the area of quadrilateral:


A. 𝐼𝑋𝐶𝑌
B. 𝐼𝑍𝐴𝑌
C. 𝐼𝑍𝐵𝑋

[𝐼𝑋𝐶𝑌] = 𝑠 2 = 1 × 1 = 1
1
[𝐼𝑍𝐴𝑌] = 2[𝐼𝑌𝐴] = 2 [ × 1 × 3] = 3
2
1
[𝐼𝑍𝐵𝑋] = 2[𝐼𝑋𝐵] = 2 [ × 1 × 2] = 2
2

Example 11.48
Find the lengths of the tangents to the incircle in a 30 − 60 − 90 right triangle with length of hypotenuse 2.

Let
𝐵𝐴 = 2 ⇒ 𝐵𝐶 = 1, 𝐶𝐴 = √3

Find the semi-perimeter


2 + 1 + √3 3 + √3
𝑠= =
2 2

3 + √3 √3 − 1
𝐶𝑋 = 𝐶𝑌 = −2=
2 2
3 + √3 √3 + 1
𝐴𝑌 = 𝐴𝑍 = −1=
2 2

P a g e 123 | 152
Get all the files at: https://bit.ly/azizhandouts
Aziz Manva ([email protected])

3 + √3 3 − √3
𝐵𝑋 = 𝐵𝑍 = − √3 =
2 2

Find the area of quadrilateral:


A. 𝐼𝑋𝐶𝑌
B. 𝐼𝑍𝐴𝑌
C. 𝐼𝑍𝐵𝑋

2
√3 − 1
2
3 − 2√3 + 1 4 − 2√3 2 − √3
[𝐼𝑋𝐶𝑌] = 𝑠 = ( ) = = =
2 4 4 2
1 √3 + 1 √3 − 1 3−1 2 1
[𝐼𝑍𝐴𝑌] = 2[𝐼𝑌𝐴] = 2 [ ( )( )] = = =
2 2 2 4 4 2
1 3 − √3 √3 − 1 3√3 − 3 − 3 + √3 4√3 − 6
[𝐼𝑍𝐵𝑋] = 2[𝐼𝑋𝐵] = 2 [ ( )( )] = =
2 2 2 4 4

Find the area of the incircle of the triangle.


2
2 √3 − 1 2 − √3
𝐴 = 𝜋𝑟 = 𝜋 ( ) =𝜋
2 2

The area inside the triangle, but outside the incircle is divided into three distinct parts. Find the area of each
part.

11.7 Review
A. Definition and Formulas

Terminology 11.49
Define the following terms, which were introduced in this chapter:
A. Angle Bisector
B. Incircle
C. Incentre
D. Inradius

Refer the definitions in the chapter

Formulas 11.50
A. 𝐵𝐷 is an angle bisector in Δ𝐴𝐵𝐶. If ∠𝐵 = 𝑏°, then what is the measure of ∠𝐴𝐵𝐷?
B. Δ𝐴𝐵𝐶 has sides 𝑎, 𝑏 and 𝑐. What is the inradius of the triangle?

𝑏
∠𝐴𝐵𝐷 =
2
Δ 𝑎+𝑏+𝑐
𝑟= , Δ = √𝑠(𝑠 − 𝑎)(𝑠 − 𝑏)(𝑠 − 𝑐), 𝑠=
𝑠 2

B. Concepts

Concept 11.51

P a g e 124 | 152
Get all the files at: https://bit.ly/azizhandouts
Aziz Manva ([email protected])

In how many places does the incircle of a triangle touch it?

Concept 11.52
At the point where an incircle touches the sides of a triangle, what is the angle between the radius and the side?
Why?

90° because the sides are tangent to the circle, and a radius is perpendicular to the tangent at the point of
tangency.

P a g e 125 | 152
Get all the files at: https://bit.ly/azizhandouts
Aziz Manva ([email protected])

12. PERPENDICULAR BISECTORS


12.1 Perpendicular Bisectors
A. Perpendicular Bisectors

12.1: Perpendicular Bisector Theorem


Any point on the perpendicular bisector of a line segment is equidistant from the endpoints of the segment.

To be precise, if 𝑃 lies on the perpendicular bisector of 𝐴𝐵, then


𝑃𝐴 = 𝑃𝐵

Consider line segment AB. Draw perpendicular bisector of AB, perpendicular


to AB at X.

Case I: The point is on Segment AB


X is the midpoint of AB, and hence
𝐴𝑋 = 𝑋𝐵 ⇒ 𝑃𝑟𝑜𝑣𝑒𝑑

Case II: The point is not on Segment AB


Draw PA and PB.

In Δ𝑃𝐴𝑋 and Δ𝑃𝐵𝑋:


𝑃𝑋 = 𝑃𝑋 (𝐶𝑜𝑚𝑚𝑜𝑛 𝑆𝑖𝑑𝑒)
𝐴𝑋 = 𝑋𝐵 (𝑋 𝑖𝑠 𝑚𝑖𝑑𝑝𝑜𝑖𝑛𝑡 𝑜𝑓 𝐴𝐵)
∠𝑃𝑋𝐴 = ∠𝑃𝐵𝑋 = 90° (𝑃𝑋 ⊥ 𝐴𝐵)

∴ By SAS Congruence:
Δ𝑃𝐴𝑋 ≅ Δ𝑃𝐵𝑋

By CPCTC:
𝑃𝐴 ≅ 𝑃𝐵 ⇒ 𝑃𝑟𝑜𝑣𝑒𝑑

Case II: Using Pythagorean Theorem


Since X is the midpoint of 𝐴𝐵:
𝐴𝑋 = 𝑋𝐵
By the Pythagorean Theorem
𝐼𝑛 Δ𝑃𝐴𝑋: 𝑃𝐴2 = 𝑃𝑋 2 + 𝐴𝑋 2
𝐼𝑛 𝑃𝐵𝑋: 𝑃𝐵2 = 𝑃𝑋 2 + 𝑋𝐵2 = 𝑃𝑋 2 + 𝐴𝑋 2

𝑃𝐴2 = 𝑃𝐵2 ⇒ 𝑃𝐴 = 𝑃𝐵

Example 12.2
Points 𝐴 and 𝐵 have a distance of 3 units between them. Find all points that are equidistant from 𝐴 and 𝐵.

All points that lie on the perpendicular bisector of A and B.

12.3: Converse of Perpendicular Bisector Theorem


If a point is equidistant from the endpoints of a line segment, then it lies on the perpendicular bisector of that

P a g e 126 | 152
Get all the files at: https://bit.ly/azizhandouts
Aziz Manva ([email protected])

segment.

To be precise, if 𝑃𝐴 = 𝑃𝐵, then 𝑃 lies on the perpendicular bisector of AB.

𝑃𝐴 = 𝑃𝐵 ⇒ Δ𝑃𝐴𝐵 𝑖𝑠 𝐼𝑠𝑜𝑠𝑐𝑒𝑙𝑒𝑠
Draw 𝑃𝑋 ⊥ 𝐴𝐵:

Since 𝑃𝑋 ⊥ 𝐴𝐵
∠𝑃𝑋𝐴 = ∠𝑃𝑋𝐵 = 90°

𝑺𝒕𝒂𝒕𝒆𝒎𝒆𝒏𝒕 𝑰
𝐴𝑋 = 𝑋𝐵 (𝐼𝑛 𝐼𝑠𝑜𝑠𝑐𝑙𝑒𝑙𝑒𝑠 Δ, 𝑎𝑙𝑡𝑖𝑡𝑢𝑑𝑒 𝑖𝑠 𝑎𝑙𝑠𝑜 𝑡ℎ𝑒 𝑚𝑒𝑑𝑖𝑎𝑛)

𝑺𝒕𝒂𝒕𝒆𝒎𝒆𝒏𝒕 𝑰𝑰

Hence, by combining Statement I and II:


𝑃 𝑙𝑖𝑒𝑠 𝑜𝑛 𝑡ℎ𝑒 𝑝𝑒𝑟𝑝𝑒𝑛𝑑𝑖𝑐𝑢𝑙𝑎𝑟 𝑏𝑖𝑠𝑒𝑐𝑡𝑜𝑟 𝑜𝑓 𝐴𝐵

Example 12.4
Point X lies on the perpendicular bisector of Points 𝐴 and 𝐵. The distance from Point 𝑋 to Point 𝐴 is 4 units.
What is the distance from Point X to Point B?

Also 4 units
B. Perpendicular Bisectors of a Triangle

12.5: Perpendicular Bisectors of a Triangle


The lines perpendicular to the sides of a triangle, and bisecting it, are the perpendicular bisectors of the
triangle.

Example 12.6
What is the number of perpendicular bisectors of a triangle?

12.7: Concurrent Lines


If a set of lines intersects at a single point, they are said to be concurrent.

12.8: Circumcenter
➢ Part A: The three perpendicular bisectors of a triangle are concurrent at the circumcenter.
➢ Part B: The circumcenter is the center of the circle that passes through the three vertices of the triangle.

Draw Δ𝐴𝐵𝐶.
Draw the perpendicular bisectors of 𝐴𝐵, and BC intersecting
𝐴𝐵 𝑎𝑡 𝑋, 𝐵𝐶 𝑎𝑡 𝑌

Since the sides of the triangle cannot be parallel, the lines perpendicular
also cannot be parallel, and hence the lines must intersect.

Let 𝑂 be the point of intersection of the two perpendicular bisectors


drawn.

P a g e 127 | 152
Get all the files at: https://bit.ly/azizhandouts
Aziz Manva ([email protected])

Since point 𝑂 lies on line 𝑋𝑂, by the Perpendicular Bisector Theorem:


∴ 𝑂 𝑖𝑠 𝑒𝑞𝑢𝑖𝑑𝑖𝑠𝑡𝑎𝑛𝑡 𝑓𝑟𝑜𝑚 𝐴 𝑎𝑛𝑑 𝐵 ⇒ ⏟𝑂𝐴 = 𝑂𝐵
𝑬𝒒𝒖𝒂𝒕𝒊𝒐𝒏 𝑰

Since point 𝑂 lies on line 𝑌𝑂, by the Perpendicular Bisector Theorem:


∴ 𝑂 𝑖𝑠 𝑒𝑞𝑢𝑖𝑑𝑖𝑠𝑡𝑎𝑛𝑡 𝑓𝑟𝑜𝑚 𝐵 𝑎𝑛𝑑 𝐶 ⇒ ⏟
𝑂𝐵 = 𝑂𝐶
𝑬𝒒𝒖𝒂𝒕𝒊𝒐𝒏 𝑰𝑰

Combine Equations I and II into a single equation, giving us:


𝑂𝐴 = 𝑂𝐵 = 𝑂𝐶 ⇒ 𝑂 𝑖𝑠 𝑒𝑞𝑢𝑖𝑑𝑖𝑠𝑡𝑎𝑛𝑡 𝑓𝑟𝑜𝑚 𝐴, 𝐵 𝑎𝑛𝑑 𝐶

𝑬𝒒𝒖𝒂𝒕𝒊𝒐𝒏 𝑰𝑰𝑰

From the above, we can say that:


∴ 𝑂 𝑖𝑠 𝑒𝑞𝑢𝑖𝑑𝑖𝑠𝑡𝑎𝑛𝑡 𝑓𝑟𝑜𝑚 𝐴 𝑎𝑛𝑑 𝐶 ⇒ ⏟
𝑂𝐴 = 𝑂𝐶
𝑬𝒒𝒖𝒂𝒕𝒊𝒐𝒏 𝑰𝑽
Part A
Hence,
𝑂 𝑙𝑖𝑒𝑠 𝑜𝑛 𝑡ℎ𝑒 𝑝𝑒𝑟𝑝𝑒𝑛𝑑𝑖𝑐𝑢𝑙𝑎𝑟 𝑏𝑖𝑠𝑒𝑐𝑡𝑜𝑟 𝑜𝑓 𝐴𝐶
We already knew that 𝑂 lies on 𝑋𝑂 and 𝑌𝑂. Now, since it lies on the perpendicular bisector of AC as well:
𝑂 𝑙𝑖𝑒𝑠 𝑜𝑛 𝑎𝑙𝑙 𝑡ℎ𝑟𝑒𝑒 ⊥ 𝑏𝑖𝑠𝑒𝑐𝑡𝑜𝑟𝑠 ⇒ 𝐴𝑙𝑙 𝑡ℎ𝑟𝑒𝑒 ⊥ 𝑏𝑖𝑠𝑒𝑐𝑡𝑜𝑟𝑠 𝑎𝑟𝑒 𝑐𝑜𝑛𝑐𝑢𝑟𝑟𝑒𝑛𝑡

Part B
From Equation III, we know that
𝑂𝐴 = 𝑂𝐵 = 𝑂𝐶 ⇒ 𝑂 𝑖𝑠 𝑒𝑞𝑢𝑖𝑑𝑖𝑠𝑡𝑎𝑛𝑡 𝑓𝑟𝑜𝑚 𝐴, 𝐵 𝑎𝑛𝑑 𝐶

𝑬𝒒𝒖𝒂𝒕𝒊𝒐𝒏 𝑰𝑰𝑰
This means that
𝑂 𝑖𝑠 𝑡ℎ𝑒 𝑐𝑒𝑛𝑡𝑒𝑟 𝑜𝑓 𝑡ℎ𝑒 𝑐𝑖𝑟𝑐𝑙𝑒 𝑡ℎ𝑎𝑡 𝑝𝑎𝑠𝑠𝑒𝑠 𝑡ℎ𝑟𝑜𝑢𝑔ℎ 𝐴, 𝐵 𝑎𝑛𝑑 𝐶

Example 12.9
Consider points 𝑋, 𝑌 and 𝑍. Find all points that are equidistant from 𝑋, 𝑌 and 𝑍.

Points equidistant from X and Y are those that lie on the perpendicular bisector of X and Y.
Points equidistant from X and Z are those that lie on the perpendicular bisector of X and Z.
Points equidistant from Y and Z are those that lie on the perpendicular bisector of Y and Z.

To meet all three conditions, they must lie on all three perpendicular bisectors above.
This is the circumcenter of Δ𝑋𝑌𝑍.

Example 12.10
Δ𝐴𝐵𝐶 is an obtuse triangle with ∠𝐴𝐵𝐶 = 120°, and circumcenter 𝑂. 𝑋 and 𝑌 are the midpoints of 𝐴𝐵 and 𝐵𝐶.
Find ∠𝑋𝑂𝑌.

P a g e 128 | 152
Get all the files at: https://bit.ly/azizhandouts
Aziz Manva ([email protected])

Draw a diagram.
In Quadrilateral 𝑋𝑂𝑌𝐵, by angle sum property of a quadrilateral:
∠𝑋𝑂𝑌 = 360 − 90 ⏟ − 90⏟ − 120 ⏟ = 60°
∠𝑂𝑋𝐵 ∠𝑂𝑌𝐵 ∠𝐴𝐵𝐶

12.2 Circumcenter
A. Definition

12.11: Circumcircle, Circumcenter, and Circumradius


➢ The unique circle that passes through the three vertices of a triangle is
called its circumcircle.
➢ The center of the circumcircle is called circumcenter.
➢ The radius of the circumcircle is called the circumradius.

Example 12.12
What is the maximum number of circumcircles that a triangle can have? The minimum?

1
1
B. Angle Chasing

Example 12.13
In right-triangle Δ𝑋𝑌𝑍, with 𝑌 as the right-angle, with O as circumcenter, ∠𝑋𝑂𝑌 = 36°. Find ∠𝑋𝑂𝑍 and ∠𝑌𝑂𝑍.

Draw a diagram.
O is the circumcenter and is the midpoint of hypotenuse XZ.
∠𝑋𝑂𝑍 = 180°
∠𝑌𝑂𝑍 = 180 − 36 = 144°

Example 12.14
In acute triangle Δ𝑋𝑌𝑍 with O as circumcenter, ∠𝑋𝑂𝑌 = 140°, and ∠𝑌𝑂𝑍 = 130°.
A. Find ∠𝑋𝑂𝑍
B. If XZ is 3 units, find XO.

Part A
By angles around a point:
∠𝑋𝑂𝑍 = 360 − 140
⏟ − 130
⏟ = 90°
∠𝑋𝑂𝑌 ∠𝑌𝑂𝑍
Part B
𝑂𝑋 = 𝑂𝑍 ⇒ Δ𝑋𝑂𝑍 𝑖𝑠 𝐼𝑠𝑜𝑠𝑐𝑒𝑙𝑒𝑠 ⇒ Δ𝑋𝑂𝑍 𝑖𝑠 45 − 45 − 90 𝑡𝑟𝑖𝑎𝑛𝑔𝑙𝑒
1 1 3
𝑋𝑂 = 𝑋𝑍 × =3× =
√2 √2 √2

Example 12.15
In acute-angled Δ𝑃𝑄𝑅, O is the point of intersection of the perpendicular bisectors of the sides. Find ∠𝑃𝑂𝑅 if:
∠𝑂𝑄𝑅 = 30°, ∠𝑂𝑅𝑄 = 50°, ∠𝑃𝑄𝑅 = 85°, ∠𝑄𝑃𝑂 = 40°

P a g e 129 | 152
Get all the files at: https://bit.ly/azizhandouts
Aziz Manva ([email protected])

∠𝑄𝑂𝑅 = 180 − 30 − 50 = 100°


∠𝑃𝑄𝑂 = 85 − 30 = 55°
∠𝑃𝑂𝑄 = 180 − 55 − 40 = 85
∠𝑃𝑂𝑅 = 360 − 100 − 85 = 175°
C. Location of Circumcenter

12.16: Location of Circumcenter


➢ For an acute-angled triangle, the circumcenter is inside the triangle.
➢ For a right-angled triangle, the circumcenter is the midpoint of the hypotenuse.
➢ For an obtuse-angled triangle, the circumcenter is outside the triangle.

MCQ 12.17
In Δ𝐴𝐵𝐶 with circumcenter O:
∠𝐴𝑂𝐵 + ∠𝐵𝑂𝐶 + ∠𝐴𝑂𝐶 = 360°
Then, Δ𝐴𝐵𝐶 cannot be:
A. Acute Angled
B. Obtuse Angled
C. Right Angled
D. Isosceles
E. Scalene

We evaluate this using cases.

Case I: 𝚫𝑨𝑩𝑪 is obtuse


For an obtuse-angled Δ𝐴𝐵𝐶 (see diagram):
∠𝐴𝑂𝐵 + ∠𝐵𝑂𝐶 + ∠𝐴𝑂𝐶 = 𝜃1 + 𝜃2 + (𝜃1 + 𝜃2 ) = 2𝜃1 + 2𝜃2
And the above does not add up 180° for an obtuse angled triangle.

Option B is correct.

Case II: 𝚫𝑨𝑩𝑪 is right-angled


For an right-angled Δ𝐴𝐵𝐶 (see diagram):
∠𝐴𝑂𝐵 + ∠𝐵𝑂𝐶 + ∠𝐴𝑂𝐶 = 𝜃1 + 𝜃2 + 𝜃3

And, by angles around a point:


𝜃1 + 𝜃2 + 𝜃3 = 360°

Option C is not correct

Example 12.18
In obtuse-angled triangle Δ𝑋𝑌𝑍 with 𝑂 as circumcenter, ∠𝑋𝑂𝑌 = 70°, and ∠𝑌𝑂𝑍 = 100°. Then ∠𝑋𝑂𝑍:
A. Is 90°
B. Is 170°
C. Is 30°
D. Either 𝐵 or 𝐶

P a g e 130 | 152
Get all the files at: https://bit.ly/azizhandouts
Aziz Manva ([email protected])

Case I: 𝚫𝑿𝒀𝒁 is obtuse-angled at ∠𝒀


∠𝑋𝑂𝑍 = ∠𝑋𝑂𝑌 + ∠𝑌𝑂𝑍 = 70 + 100 = 170°
This case gives us a solution, which is Option B.

Case II: 𝚫𝑿𝒀𝒁 is obtuse-angled at ∠𝒁


∠𝑌𝑂𝑋 = ∠𝑌𝑂𝑍 + ZOX ⇒ 70 = 80 + XOZ ⇒ XOZ 𝑖𝑠 − 𝑣𝑒 ⇒ 𝑁𝑜 𝑆𝑜𝑙𝑢𝑡𝑖𝑜𝑛
This case does not give us any solutions.

Case III: 𝚫𝑿𝒀𝒁 is obtuse-angled at ∠𝑿


∠𝑌𝑂𝑍 = ∠𝑋𝑂𝑌 + ∠𝑋𝑂𝑍 ⇒ 100 = 70 + ∠𝑋𝑂𝑍 ⇒ ∠𝑋𝑂𝑍 = 30°
This case gives us a solution, which is Option C.

Since both Option B or Option C can be correct, the final answer is Option D.

12.3 Circumcircle
A. Circle Properties
In this section, we focus on the circle properties of the circumcircle. If you have not seen these properties
before, you can refer the Note on Circles.
B. Inscribed Angles

12.19: Angles in the same Arc


Two angles inscribed in the same arc are congruent.

Example 12.20
O is the circumcenter of Δ𝐴𝐵𝐶 with ∠𝐴 = 40. O is also the circumcenter of Δ𝐷𝐵𝐶. Find the sum of the possible
values of ∠𝐵𝐷𝐶.

There are two cases to consider.

Case I: D is on the same side of the chord as A


See Diagram.
⏜.
∠𝐵𝐴𝐶 is inscribed in 𝐵𝑋𝐶
⏜.
∠𝐵𝐷𝐶 is inscribed in 𝐵𝑋𝐶

Hence,
∠𝐵𝐴𝐶 ≅ ∠𝐷𝐴𝐶 = 40°

Case I: D is on the opposite side of the chord as A


See Diagram.
⏜.
∠𝐵𝐴𝐶 is inscribed in 𝐵𝑋𝐶
⏜.
∠𝐷𝐴𝐶 is inscribed in 𝐵𝑋𝐶

Hence,
∠𝐵𝐴𝐶 ≅ ∠𝐷𝐴𝐶 = 40°

P a g e 131 | 152
Get all the files at: https://bit.ly/azizhandouts
Aziz Manva ([email protected])

12.21: Angles in congruent Arcs


Two angles inscribed in congruent arcs are congruent.

Example 12.22
O is the circumcenter of Δ𝐴𝐵𝐶 with ∠𝐴 = 40. P is the circumcenter of ΔXYZ. The circumradius of Δ𝐴𝐵𝐶 is equal
to the circumradius of ΔXYZ.

12.23: Congruent Inscribed Angles


If two inscribed angles have the same measure, the arcs they intercept are congruent.

C. Central Angle

12.24: Inscribed Angle


An inscribed angle in a circle is one-half of the corresponding central angle.

Example 12.25
O is the circumcenter of Δ𝐴𝐵𝐶 with ∠𝐴 = 40 and ∠𝐵 = 70. Find

12.4 Equilateral Triangles


A. Circumcenter
An equilateral triangle has all three sides equal. This gives it symmetry. And the symmetry results in many
elegant and powerful properties, which do not otherwise hold for general triangles.
One of these properties is that the incenter and the circumcenter of an equilateral triangle are the same. We
first prove this property, and then we put it to work for us.

12.26: Incenter and Circumcenter coincide in an equilateral triangle


The incentre of an equilateral triangle is also its circumcenter.

Draw equilateral Δ𝐴𝐵𝐶 and construct incentre I as the intersection of:


𝐴𝑛𝑔𝑙𝑒 𝐵𝑖𝑠𝑒𝑐𝑡𝑜𝑟𝑠 𝐼𝐴, 𝐼𝐵, 𝐼𝐶
Since Δ𝐴𝐵𝐶 is equilateral:
∠𝐴𝐵𝐶 = ∠𝐴𝐶𝐵 = 60°
Since IB and IC are angle bisectors:
60
∠𝐼𝐵𝐷 = ∠𝐼𝐶𝐵 = = 30°
2
In Isosceles Δ𝐼𝐵𝐶,
𝑀𝑒𝑑𝑖𝑎𝑛 𝐼𝐷 = 𝐴𝑙𝑡𝑖𝑡𝑢𝑑𝑒 𝐼𝐷 ⇒ 𝐼𝐷 𝑖𝑠 ⊥ 𝑏𝑖𝑠𝑒𝑐𝑡𝑜𝑟 𝑜𝑓 𝐵𝐶
Similarly,
𝐼𝐸 𝑖𝑠 ⊥ 𝑏𝑖𝑠𝑒𝑐𝑡𝑜𝑟 𝑜𝑓 𝐴𝐶
𝐼𝐹 𝑖𝑠 ⊥ 𝑏𝑖𝑠𝑒𝑐𝑡𝑜𝑟 𝑜𝑓 𝐴𝐵
Hence, point I is intersection of perpendicular bisectors of the sides of Δ𝐴𝐵𝐶.
Hence, it is also the circumcenter.

12.27: Angle Bisector and Perpendicular Bisectors


In an equilateral triangle, the angle bisectors and perpendicular bisectors are concurrent.

P a g e 132 | 152
Get all the files at: https://bit.ly/azizhandouts
Aziz Manva ([email protected])

𝐴𝑛𝑔𝑙𝑒 𝐵𝑖𝑠𝑒𝑐𝑡𝑜𝑟𝑠(𝐼𝐴, 𝐼𝐵, 𝐼𝐶) and Perpendicular Bisectors of the sides (𝐼𝐷, 𝐼𝐸 𝑎𝑛𝑑 𝐼𝐹) are concurrent.

This should be clear from the proof of the previous property, but is important enough to state as a property on
its own.
Since the circumcenter and the incenter in an equilateral triangle are the same, that common point is the point
of concurrency of both its angle bisectors and the perpendicular bisectors of its sides.

12.28: 𝟑𝟎 − 𝟔𝟎 − 𝟗𝟎 Triangles formed by Incenter/Circumcenter


The common incenter/circumcenter of an equilateral triangle divides it into 6
congruent 30 − 60 − 90 triangles.
In the diagram, these are:
Δ𝐼𝐵𝐷, Δ𝐼𝐶𝐷, Δ𝐼𝐶𝐸, Δ𝐼𝐴𝐸, Δ𝐼𝐴𝐹, Δ𝐼𝐵𝐹

Draw equilateral Δ𝐴𝐵𝐶 with incenter/circumcenter I.

Since Δ𝐴𝐵𝐶 is equilateral:


∠𝐴𝐵𝐶 = 60°
Since IB is an angle bisector:
60
∠𝐼𝐵𝐷 = = 30
2
Since ID is a perpendicular bisector:
∠𝐼𝐷𝐵 = 90°
By sum of angles in a triangle, in ΔIBD:
∠𝐵𝐼𝐶 = 180 − 30 − 90 = 60
Hence,
Δ𝐼𝐵𝐷 𝑖𝑠 𝑎 30 − 60 − 90 𝑡𝑟𝑖𝑎𝑛𝑔𝑙𝑒
Similarly, the following are all 30 − 60 − 90 𝑡𝑟𝑖𝑎𝑛𝑔𝑙𝑒𝑠:
Δ𝐼𝐶𝐷, Δ𝐼𝐶𝐸, Δ𝐼𝐴𝐸, Δ𝐼𝐴𝐹, Δ𝐼𝐵𝐹

12.29: Circumradius of an equilateral triangle


1
The circumradius of an equilateral triangle is times the side length
√3

Note: Pay attention to the properties of equilateral triangles, being shown here, not just the answer. In
particular, how can we make use of the fact that
𝐶𝑖𝑟𝑐𝑢𝑚𝑐𝑒𝑛𝑡𝑒𝑟 = 𝐼𝑛𝑐𝑒𝑛𝑡𝑒𝑟

Draw Δ𝐴𝐵𝐶 with side length 𝑠 with I as circumcenter.


The circumradius is the distance from the circumcenter to the vertices.
𝐶𝑖𝑟𝑐𝑢𝑚𝑟𝑎𝑑𝑖𝑢𝑠 = 𝐼𝐴 = 𝐼𝐵 = 𝐼𝐶

Since the triangle is equilateral, the circumcenter is also the incenter.


Hence, 𝐼 is the intersection of angle bisectors:
𝐼𝐴, 𝐼𝐵, 𝐼𝐶

P a g e 133 | 152
Get all the files at: https://bit.ly/azizhandouts
Aziz Manva ([email protected])

12.30: Circumradius and Area of Circumcircle of an equilateral triangle


1
The circumradius of an equilateral triangle is times the side length
√3
𝜋𝑥 2
𝐴𝑟𝑒𝑎 𝑜𝑓 𝐶𝑖𝑟𝑐𝑢𝑚𝑐𝑖𝑟𝑐𝑙𝑒 =
3

𝑎𝑏𝑐 𝑥3 𝑥
𝐶𝑖𝑟𝑐𝑢𝑚𝑟𝑎𝑑𝑖𝑢𝑠 = 𝑅 = = =
4Δ √3 √3
4 ( 4 × 𝑥2)

Example 12.31
What is the area of the circumcircle on an equilateral triangle with side length 𝑥?

𝑥 2 𝜋𝑥 2
𝐴𝑟𝑒𝑎 𝑜𝑓 𝐶𝑖𝑟𝑐𝑢𝑚𝑐𝑖𝑟𝑐𝑙𝑒 = 𝜋𝑅 2 = 𝜋 ( ) =
√3 3

Example 12.32
The number of inches in the perimeter of an equilateral triangle equals the number of square inches in the area
of its circumscribed circle. What is the radius, in inches, of the circle? (AMC 10A 2003/17)

𝑥
𝑅= ⇒ 𝑥 = 𝑅√3 ⇒ 𝑃 = 3𝑥 = 3√3𝑅
√3
3√3
𝐴 = 𝑃 ⇒ 𝜋𝑅 2 = 3√3𝑅 ⇒ 𝑅 =
𝜋

Example 12.33
What is the ratio of the inradius and the circumradius of an equilateral triangle?

Example 12.34
What is the ratio of the area of the circumcircle and the area of the incircle of an equilateral triangle?

Example 12.35
A circle is circumscribed about an equilateral triangle with side lengths of 9 units each. What is the area of the
circle, in square units? Express your answer in terms of 𝜋. (MathCounts 2008 School Countdown)
2

𝑎𝑏𝑐 2 93 9 2
81
𝐴 = 𝜋𝑅 2 = 𝜋 ( ) =𝜋 = 𝜋( ) = 𝜋( ) = 27𝜋
4Δ √3 √3 3
4 ( 4 × 92 )
( )

12.5 Circumradius
A. Definition

12.36: Circumradius
Let the circumradius of a triangle be 𝑹. And let the lengths of the three sides of the triangle be 𝒂, 𝒃, and 𝒄.
𝑎𝑏𝑐 𝑎𝑏𝑐
𝐶𝑖𝑟𝑐𝑢𝑚𝑟𝑎𝑑𝑖𝑢𝑠 = 𝑅 = =
4Δ 4𝑟𝑠

P a g e 134 | 152
Get all the files at: https://bit.ly/azizhandouts
Aziz Manva ([email protected])

Draw Diameter 𝐶𝐷 passing through center 𝑂 in the circumcircle of Δ𝐴𝐵𝐶. 4


∴ ∠𝐶𝐴𝐷 = 90°
Draw altitude 𝐶𝐸 in Δ𝐶𝐴𝐵 with length ℎ.
∠𝐶𝐴𝐷 = ∠𝐶𝐸𝐵 = 90° ⇒ 𝐈
Since both ∠𝐴𝐷𝐶 and ∠𝐴𝐵𝐶 subtend the same arc 𝒅:
∠𝐴𝐷𝐶 = ∠𝐴𝐵𝐶 ⇒ 𝐈𝐈
Combine 𝐈 and 𝐈𝐈 above to show that we have two similar triangles:
⇒ Δ𝐶𝐴𝐷~Δ𝐶𝐸𝐵 (𝐴𝐴 𝑆𝑖𝑚𝑖𝑙𝑎𝑟𝑖𝑡𝑦)
Hence:
ℎ 𝑏 𝑎𝑏
= ⇒𝑅=
𝑎 2𝑅 2ℎ
1 2Δ
And if we substitute Δ = 2 𝑐ℎ ⇒ ℎ = c :
𝑎𝑏 𝑎𝑏𝑐
𝑅= =
2Δ 4Δ
2( c )

B. General Triangles

Example 12.37
The sides of a triangle are 13, 14 and 15. Find its:
A. Semi-perimeter
B. Area
C. Circumradius.

𝑎 + 𝑏 + 𝑐 13 + 14 + 15 42
Semiperimeter = = = = 21
2 2 2

Δ = √𝑠(𝑠 − 𝑎)(𝑠 − 𝑏)(𝑠 − 𝑐) = √21(8)(7)(6) = √32 × 72 (24 ) = 84


𝑎𝑏𝑐 13 × 14 × 15 13 × 15 195 1
𝐶𝑖𝑟𝑐𝑢𝑚𝑟𝑎𝑑𝑖𝑢𝑠 = 𝑅 = = = = =8
4Δ 4 × 84 4×6 24 8

Challenge 12.38
The diagram alongside has two perpendicular chords in a circle. Given that
𝐴𝐸 = 2, 𝐸𝐵 = 9, 𝐸𝐶 = 3
find twice the square of the radius of the given circle. (NMTC Sub-Junior 2021,
Adapted)

Consider the circle passing through points


𝐴, 𝐵, 𝑎𝑛𝑑 𝐶
Recall that three points define a circle. Hence, if we find the circumradius of the triangle, we are done.

Sides of 𝚫𝑨𝑩𝑪
Using Pythagorean Theorem in right Δ𝐸𝐶𝐵:
𝑎 = √𝐸𝐵2 + 𝐸𝐶 2 = √92 + 32 = √81 + 9 = √90 = 3√10

Using Pythagorean Theorem in right ΔAEC:


𝑏 = √𝐴𝐸 2 + 𝐸𝐶 2 = √22 + 32 = √4 + 9 = √13

4
This link from Khan Academy has a video with the same proof.

P a g e 135 | 152
Get all the files at: https://bit.ly/azizhandouts
Aziz Manva ([email protected])

𝑐 = 𝐴𝐸 + 𝐸𝐵 = 2 + 9 = 11
Four times the area of 𝚫𝑨𝑩𝑪
1 1 1
4Δ = 4[𝐴𝐵𝐶] = 4 ( ℎ𝑏) = 4 ( × 𝐸𝐶 × 𝐴𝐵) = 4 ( × 3 × 11) = 6 × 11
2 2 2
Substitution
𝑎𝑏𝑐
Substitute above values in 𝑅 = 4Δ
to get:
𝑎𝑏𝑐 3√10 × √13 × 11 √10 × 13 √130 130
𝑅= = = = ⇒ 2𝑅 2 = 2 × = 65
4Δ 6 × 11 2 2 4

C. Right Triangles

Example 12.39
The sides of a triangle are 3, 4, and 5. Find its circumradius.

𝑎𝑏𝑐 3×4×5 5
𝑅= = = = 2.5
4Δ 1 2
2×4×3×4

12.40: Circumradius of a Right Triangle


The circumradius of a right triangle is half the length of its hypotenuse.

The sides of a right triangle are 𝑎, 𝑏, and 𝑐 (with 𝑐 as the hypotenuse). Find its circumradius.

𝑎𝑏𝑐 𝑎𝑏𝑐 𝑐
𝑅= = =
4Δ 1 2
2 × 4 × 𝑎𝑏

Example 12.41
The sides of a triangle are 3, 4, and 5. Find its circumradius.

(3,4,5) is a Pythagorean Triplet. Hence, the triangle is right-angled.

ℎ𝑦𝑝 5
∴ 𝑅= = = 2.5
2 2

Example 12.42
Two sides of a right triangle are 3 and 4. Find the sum of all possible values of the area of the circle that passes
through the vertices of the triangle. (Answer in terms of 𝜋).

Case I: 3 and 4 are the legs


ℎ𝑦𝑝 5
∴ 𝑅= = = 2.5 ⇒ 𝐴 = 𝜋𝑅 2 = 𝜋 × 2.52 = 6.25𝜋
2 2
Case II: 3 is the hypotenuse
𝐻𝑦𝑝 𝑖𝑠 𝑙𝑜𝑛𝑔𝑒𝑠𝑡 𝑠𝑖𝑑𝑒 ⇒ 3 𝑖𝑠 𝑙𝑜𝑛𝑔𝑒𝑠𝑡 𝑠𝑖𝑑𝑒 ⇒ 3 > 4 ⇒ 𝐶𝑜𝑛𝑡𝑟𝑎𝑑𝑖𝑐𝑡𝑖𝑜𝑛 ⇒ 𝑁𝑜𝑡 𝑃𝑜𝑠𝑠𝑖𝑏𝑙𝑒

Case III: 4 is the hypotenuse


ℎ𝑦𝑝 4
∴ 𝑅= = = 2 ⇒ 𝐴 = 𝜋𝑅 2 = 𝜋 × 22 = 4𝜋
2 2

P a g e 136 | 152
Get all the files at: https://bit.ly/azizhandouts
Aziz Manva ([email protected])

𝑇𝑜𝑡𝑎𝑙 = 6.25𝜋 + 4𝜋 = 10.25𝜋

Example 12.43
A triangle with side lengths in the ratio 3: 4: 5 is inscribed in a circle with radius 3. What is the area of the
triangle? (AMC 10A 2007/14)

3: 4: 5 𝑖𝑠 𝑎 𝑃𝑦𝑡ℎ𝑎𝑔𝑜𝑟𝑒𝑎𝑛 𝑇𝑟𝑖𝑝𝑙𝑒𝑡 ⇒ Δ 𝑖𝑠 𝑟𝑖𝑔ℎ𝑡 − 𝑎𝑛𝑔𝑙𝑒𝑑


∴ 𝐶𝑖𝑟𝑐𝑢𝑚𝑟𝑎𝑑𝑖𝑢𝑠 𝑖𝑠 ℎ𝑎𝑙𝑓 𝑜𝑓 𝑡ℎ𝑒 ℎ𝑦𝑝𝑜𝑡𝑒𝑛𝑢𝑠𝑒.

Hence, hypotenuse is double of the circumradius:


𝐻𝑦𝑝 = 2𝑅 = 6

The triangle has side lengths in the ratio ⏟


3 : ⏟
4 : ⏟
5 , and ℎ𝑦𝑝𝑜𝑡𝑒𝑛𝑢𝑠𝑒 = 6, so its other sides are
𝐿𝑒𝑔 1 𝐿𝑒𝑔 2 𝐻𝑦𝑝𝑜𝑡𝑒𝑛𝑢𝑠𝑒
6 18 6 24
3× = , 4× =
5 5 5 5
Finally, area of the triangle:
1 1 18 24 216
= ℎ𝑏 = × × =
2 2 5 5 25

12.44: Circumcenter of a Right Triangle


The circumcenter of a right triangle is the midpoint of its hypotenuse.

Draw right Δ𝐴𝐵𝐶.


Draw perpendicular bisector of 𝐵𝐶 and let it intersect the
hypotenuse at 𝑀3 , and BC at 𝑀2 .

From 𝑀3 , draw line ⊥ to 𝑀3 𝑀2 and let it intersect the leg 𝐴𝐵


at 𝑀1 .

Δ𝐴𝑀1 𝑀3 and Δ𝑀3 𝑀2 𝐶 are similar because:


∠𝐴𝑀1 𝑀3 = ∠𝑀3 𝑀2 𝐶 = 90°
∠𝐴𝑀3 𝑀1 = ∠𝑀3 𝐶𝑀2
Also,
𝑀1 𝑀3 = 𝐵𝑀2 = 𝑀2 𝐶 ⇒ Δ𝐴𝑀1 𝑀3 ≅ Δ𝑀3 𝑀2 𝐶
By CPCT:
𝐴𝑀1 = 𝑀3 𝑀2 = 𝑀1 𝐵 ⇒ 𝑀1 𝑖𝑠 𝑚𝑖𝑑𝑝𝑜𝑖𝑛𝑡 𝑜𝑓 𝐴𝐵
Also, by CPCT:
𝐴𝑀3 = 𝑀3 𝐶 ⇒ 𝑀3 𝑖𝑠 𝑚𝑖𝑑𝑝𝑜𝑖𝑛𝑡 𝑜𝑓 𝐴𝐶

Example 12.45
A right triangle with circumradius 1 has its legs in the ratio 3: 5. Determine the inradius of the triangle.

𝐶𝑖𝑟𝑐𝑢𝑚𝑟𝑎𝑑𝑖𝑢𝑠 = 𝑅 = 1 ⇒ 𝐻𝑦𝑝 = 2
The legs are in the ratio:
3: 5 = 3𝑥: 5𝑥
Hence, by the Pythagorean Theorem:

P a g e 137 | 152
Get all the files at: https://bit.ly/azizhandouts
Aziz Manva ([email protected])

2 2 2
(3𝑥)2 + (5𝑥)2 = 22 ⇒ 34𝑥 2 = 4 ⇒ 𝑥 = √ ⇒ 3𝑥 = 3√ ⇒ 5𝑥 = 5√
17 17 17
1 2 2 15 2
Δ 2 × 3√17 × 5√17 × 15 2 30
𝐼𝑛𝑟𝑎𝑑𝑖𝑢𝑠 = 𝑟 = = = 2 17 = × =
𝑠 2 2 2 17 2 2
3√ + 5√ + 2 8√ + 2 8√ + 2 136√ + 34
17 17 17 17 17
2 2
D. Isosceles Triangles

Example 12.46
An isosceles triangle with equal sides of 5 inches and a base of 6 inches is inscribed in a circle. What is the
radius, in inches, of the circle? Express your answer as a mixed number. (MathCounts 1992 State Target)

𝑎𝑏𝑐 5 × 5 × 6 50 25 1
𝑅= = = = =3
4Δ 4(4 × 3) 16 8 8

12.6 Review
A. Definition and Formulas

Terminology 12.47
Define the following terms:
A. Perpendicular bisector of a side of a triangle
B. Circumcircle
C. Circumcenter
D. Circumradius

B. Concepts

Concept Check 12.48


What can you say about a triangle where the
A. inradius is the same as the circumradius.
B. circumcircle is the same as the incircle.
C. incentre is the same as the circumcenter.

Parts A and B
Such a triangle does not exist.

Part C
Triangle is equilateral. The two circles are concentric

True or False 12.49


A. The incenter of a triangle is also its circumcenter if and only if the triangle is isosceles.
B. If a triangle is equilateral, then its incentre is the same as its circumcenter, and hence its inradius is the
same as its circumradius.

P a g e 138 | 152
Get all the files at: https://bit.ly/azizhandouts
Aziz Manva ([email protected])

𝐹𝑎𝑙𝑠𝑒. 𝑆ℎ𝑜𝑢𝑙𝑑 𝑏𝑒 𝑒𝑞𝑢𝑖𝑙𝑎𝑡𝑒𝑟𝑎𝑙


𝐹𝑎𝑙𝑠𝑒.

MCQ 12.50
Consider isosceles triangle 𝐴𝐵𝐶 with incentre I. Out of 𝐼𝐴, 𝐼𝐵 and 𝐼𝐶 (these represent lengths):
A. Exactly two are the same
B. At most two are the same
C. At least two are the same
D. None of the above

𝑂𝑝𝑡𝑖𝑜𝑛 𝐶

MCQ 12.51
In Δ𝐴𝐵𝐶 construct the perpendicular bisectors of the sides and find their point of intersection O. Then, the
distance from O to the:
A. sides is the inradius.
B. vertices is the inradius.
C. sides is the circumradius.
D. vertices is the circumradius.

𝑂𝑝𝑡𝑖𝑜𝑛 𝐷

P a g e 139 | 152
Get all the files at: https://bit.ly/azizhandouts
Aziz Manva ([email protected])

13.MEDIANS AND ALTITUDES


13.1 Medians
A. Median
The line segment from the vertex of a triangle to the midpoint of the
opposite side is called its median.
➢ By definition, a median bisects the side opposite.
➢ Every triangle has three medians.
➢ The medians never go outside the triangle.

In the diagram alongside (not drawn to scale), 𝐴𝐷 is the median


drawn from vertex A. This means that
𝐵𝐷 = 𝐷𝐶
Intersection of Medians
The three medians of a triangle intersect at the centroid.
The centroid divides each median in the ratio 2: 1.

Physical Interpretation of Centroid


Centroid has applications in physics as well. It represents the
point at which an object will balance.

B. Centroid
The centroid is the point at which an object will balance. For
example, consider the see-saw drawn alongside. The weight
at the left is larger than the weight on the right.
Hence, to balance the see-saw, the pole on which it is
balanced is to the left of the center.

In general, the point at which an object can be balanced on a pole is the centroid of the object.

13.1: Centroid divides medians in the ratio 𝟐: 𝟏


The centroid of a triangle divides medians in the ratio 2: 1, with the longer part towards the vertex, and the
smaller part towards the base.

C. Equilateral Triangles

13.2: Points in an Equilateral Triangle


In an equilateral triangle
𝐼𝑛𝑐𝑒𝑛𝑡𝑟𝑒 = 𝐶𝑖𝑟𝑐𝑢𝑚𝑐𝑒𝑛𝑡𝑒𝑟 = 𝐶𝑒𝑛𝑡𝑟𝑜𝑖𝑑

Example 13.3

P a g e 140 | 152
Get all the files at: https://bit.ly/azizhandouts
Aziz Manva ([email protected])

A circle is circumscribed about an equilateral triangle with side lengths of 9 units each. What is the area of the
circle, in square units? Express your answer in terms of 𝜋. (MathCounts 2008 School
Countdown)

√3 9√3
𝐻𝑒𝑖𝑔ℎ𝑡 = 𝐴𝑀 = ×9=
2 2
2 2 9√3
𝐶𝑖𝑟𝑐𝑢𝑚𝑟𝑎𝑑𝑖𝑢𝑠 = × 𝐴𝑀 = × = 3√3
3 3 2
2
𝐴𝑟𝑒𝑎 = 𝜋𝑟 2 = 𝜋(3√3) = 27𝜋

Example 13.4
In triangle 𝐴𝐵𝐶, medians 𝐴𝐷 and 𝐶𝐸 intersect at 𝑃, 𝑃𝐸 = 1.5, 𝑃𝐷 = 2, and 𝐷𝐸 = 2.5. What is the area of 𝐴𝐸𝐷𝐶?
(AMC 10B 2013/16)

13.2 Altitudes
A. Altitude
The line segment from the vertex of a triangle which is perpendicular to the opposite side is called its altitude.
➢ The length of altitude is called its height. Make sure you
understand the difference between the two.
➢ Every triangle has three altitudes.

Right-Angled Triangle:
In the diagram alongside, Δ𝐴𝐵𝐶 is right angled at ∠𝐵.
The altitude to side AB is BC.
The altitude to side BC is AB.
The altitude to side AC is AB.

In a right-angled triangle, the altitudes are the legs of the triangle.

Acute Triangle
In the diagram alongside, Δ𝐴𝐵𝐶 is an acute -angled triangle, with
altitudes AF, BD and CE.
Note that all the altitudes fall within the triangle.

Obtuse-Angled Triangle:
In the diagram alongside, obtuse triangle 𝐴𝐵𝐶 has altitudes BF, AD
and CH.
BF, which is the the altitude opposite the obtuse angle lies inside the
triangle.
AD and CH lie outside the triangle and sides BC and AB must be
“produced” (extended) for them to intersect the altitudes.

Orthocenter
The three altitudes of a triangle intersect at the orthocenter.
➢ In an acute triangle, the orthocenter lies inside the triangle.
➢ In a right triangle, the orthocenter lies at the intersection of the legs.
In an obtuse triangle, the orthocenter lies outside the triangle.

P a g e 141 | 152
Get all the files at: https://bit.ly/azizhandouts
Aziz Manva ([email protected])

Example 13.5
Right triangle 𝐴𝐵𝐶 has leg lengths 𝐴𝐵 = 20 and 𝐵𝐶 = 21. Including 𝐴𝐵 and 𝐵𝐶, how many line segments with
integer length can be drawn from vertex 𝐵 to a point on hypotenuse 𝐴𝐶? (AMC 10A 2018/16)

Primitive Pythagorean Triplet:


(20,21,29) ⇒ 𝐴𝐶 = 29
1 1 420
× 20 × 21 = × 29 × ℎ ⇒ ℎ = ∈ (14,15)
2 2 29

To the left, going towards AB, we get


{15,16, … ,20} ⇒ 6 𝑁𝑢𝑚𝑏𝑒𝑟𝑠
To the left, going towards BC, we get
{15,16, … ,21} ⇒ 7 𝑁𝑢𝑚𝑏𝑒𝑟𝑠

Total
= 6 + 7 = 13 𝑉𝑎𝑙𝑢𝑒𝑠

Example 13.6
Point 𝑃 is inside equilateral △ 𝐴𝐵𝐶. Points 𝑄, 𝑅, and 𝑆 are the feet of the perpendiculars from 𝑃 to 𝐴𝐵, 𝐵𝐶, and
𝐶𝐴, respectively. Given that 𝑃𝑄 = 1, 𝑃𝑅 = 2, and 𝑃𝑆 = 3, what is 𝐴𝐵? (AMC 10B 2007/17)

Example 13.7
A paper triangle with sides of lengths 3, 4, and 5 inches, as shown, is folded so that point 𝐴 falls on point 𝐵.
What is the length in inches of the crease? (AMC 10A 2018/13)

13.3 Equilateral Triangles


13.8: Altitudes, Medians, Angle Bisectors and Perpendicular Bisectors

13.9: Nine-Point Circle

P a g e 142 | 152
Get all the files at: https://bit.ly/azizhandouts
Aziz Manva ([email protected])

PART V: TRIANGLE INEQUALITY


14. TRIANGLE INEQUALITY
14.1 Basics
A. Statement
The triangle inequality combines geometry with one of the more difficult topics in Algebra – Inequalities. It
leads to some important restrictions on the length of values that triangle sides can take.

14.1: Maximum Length of Side


In a triangle, any side is less than the sum of the other two sides.

Specifically, in a triangle with sides 𝑎, 𝑏 and 𝑐:


𝑎 <𝑏+𝑐
𝑏 <𝑎+𝑐
𝑐 <𝑎+𝑏

We take it as an axiom in geometry that


𝐴 𝑠𝑡𝑟𝑎𝑖𝑔ℎ𝑡 𝑙𝑖𝑛𝑒 𝑖𝑠 𝑡ℎ𝑒 𝑠ℎ𝑜𝑟𝑡𝑒𝑠𝑡 𝑑𝑖𝑠𝑡𝑎𝑛𝑐𝑒 𝑏𝑒𝑡𝑤𝑒𝑒𝑛 𝑡𝑤𝑜 𝑝𝑜𝑖𝑛𝑡𝑠.
And the property above directly follows from the axiom.
B. Proving Validity
If an ordered triple of lengths does not satisfy the triangle inequality, then the lengths cannot form a triangle.
This is a powerful tool, since it shows there is 𝑛𝑜 triangle that satisfies the given conditions.

Example 14.2
Show that a triangle with side lengths 3, 4 𝑎𝑛𝑑 5 is a valid triangle.

3+4 = 7> 5
3+5 = 8> 4
4+5 = 9> 3

Example 14.3
Show that a triangle with side lengths 3, 4 𝑎𝑛𝑑 6 is a valid triangle.

3+4 = 7> 6
3+6 = 9> 4
4 + 6 = 10 > 3

Example 14.4
Show that a triangle with side lengths 3, 4 𝑎𝑛𝑑 7 is not a valid triangle.

3+4=7
Sum of two sides is equal to the third side.
Hence, the triangle inequality is not satisfied.

This is not a valid triangle.

P a g e 143 | 152
Get all the files at: https://bit.ly/azizhandouts
Aziz Manva ([email protected])

C. Checking for Validity

Example 14.5
Is a triangle with side lengths 5, 8 and 12 a valid triangle?

5 + 8 = 13 > 12
8 + 12 = 20 > 5
5 + 12 = 17 > 8

Example 14.6
Is a triangle with side lengths 4, 7 and 13 a valid triangle?

4 + 7 = 11 < 13
This is not a valid triangle.

Example 14.7
Given that 𝑥 is an integer, how many triangles exist with side lengths 3𝑥, 4𝑥 𝑎𝑛𝑑 8𝑥?

3𝑥 + 4𝑥 > 8𝑥 ⇒ 7𝑥 > 8𝑥 ⇒ 𝑁𝑜𝑡 𝑃𝑜𝑠𝑠𝑖𝑏𝑙𝑒 ⇒ 𝑵𝒐 𝑻𝒓𝒊𝒂𝒏𝒈𝒍𝒆𝒔


The number of such triangles is zero.

(Continuation) Example 14.8


Harry solved the above example, and he wrote
𝑇ℎ𝑒𝑟𝑒 𝑎𝑟𝑒 𝑛𝑜 𝑣𝑎𝑙𝑢𝑒𝑠 𝑜𝑓 𝑥 𝑡ℎ𝑎𝑡 𝑠𝑎𝑡𝑖𝑠𝑓𝑦 𝑡ℎ𝑒 𝑖𝑛𝑒𝑞𝑢𝑎𝑙𝑖𝑡𝑦 7𝑥 > 8𝑥
A. State the exceptions to Harry’s statement?
B. Can you make a valid triangle with those exceptions?

𝐸𝑥𝑐𝑒𝑝𝑡𝑖𝑜𝑛𝑠 𝑎𝑟𝑒 𝑤ℎ𝑒𝑛 𝑥 ≤ 0


𝑁𝑜, 𝑏𝑒𝑐𝑎𝑢𝑠𝑒 𝑙𝑒𝑛𝑔𝑡ℎ > 0

14.9: Distance is never negative


The distance between two points is never negative.
The smallest value that that distance can have is zero.

D. Difference of Sides

14.10: Difference of Sides


In a triangle, the difference of any two sides is less than the third side.

In a triangle with sides 𝑎, 𝑏 and 𝑐:


𝑎−𝑏 <𝑐
𝑏−𝑎 <𝑐
𝑎−𝑐 <𝑏
𝑐−𝑎 <𝑏
𝑏−𝑐 <𝑎
𝑐−𝑏 <𝑎

P a g e 144 | 152
Get all the files at: https://bit.ly/azizhandouts
Aziz Manva ([email protected])

The properties above can be directly derived from the earlier properties on the sum of sides.
For example:
𝑎 <𝑏+𝑐 ⇒𝑎−𝑏 <𝑐
𝑏 <𝑎+𝑐 ⇒𝑏−𝑎 <𝑐

Example 14.11
A triangle has two sides 5 𝑓𝑒𝑒𝑡 𝑎𝑛𝑑 6 𝑓𝑒𝑒𝑡. What are the values that the third side can be? (Answer in
inequality form).

Maximum Length
Third side must be less than 5 + 6 = 11. That is:
𝑇ℎ𝑖𝑟𝑑 𝑆𝑖𝑑𝑒 < 11 𝑓𝑒𝑒𝑡

Minimum Length
And the difference of the two given sides must be less than the third side
6 − 5 = 1 < 𝑇ℎ𝑖𝑟𝑑 𝑆𝑖𝑑𝑒

1 < 𝑇ℎ𝑖𝑟𝑑 𝑆𝑖𝑑𝑒 < 11

Example 14.12
A triangle has two sides which 7 𝑖𝑛𝑐ℎ𝑒𝑠 𝑎𝑛𝑑 1 𝑓𝑜𝑜𝑡, respectively. What are the values that the third side can be?
(Answer in inequality form).

𝑇ℎ𝑖𝑟𝑑 𝑆𝑖𝑑𝑒 < 7 + 12 ⇒ 𝑇ℎ𝑖𝑟𝑑 𝑆𝑖𝑑𝑒 < 19


12 − 7 < 𝑇ℎ𝑖𝑟𝑑 𝑆𝑖𝑑𝑒 ⇒ 5 < 𝑇ℎ𝑖𝑟𝑑 𝑆𝑖𝑑𝑒

5 < 𝑇ℎ𝑖𝑟𝑑 𝑆𝑖𝑑𝑒 < 19

Example 14.13
1 1
A triangle has two sides which 2 𝑖𝑛𝑐ℎ𝑒𝑠 𝑎𝑛𝑑 3 𝑖𝑛𝑐ℎ𝑒𝑠, respectively. What are the values that the third side can
be? (Answer in inequality form).

Third side has to be less than the sum of the other two sides:
1 1 5
𝑇ℎ𝑖𝑟𝑑 𝑆𝑖𝑑𝑒 < + =
2 3 6

Third side has to be more than the difference the other two sides:
1 1 1
− = < 𝑇ℎ𝑖𝑟𝑑 𝑆𝑖𝑑𝑒
2 3 6

1 5
< 𝑇ℎ𝑖𝑟𝑑 𝑆𝑖𝑑𝑒 <
6 6
E. Integer Lengths

Example 14.14
Robin has made a triangle out of matchsticks, placed end to end. One side of the triangle is made up of 12
matchsticks. The second side needs 15 matchsticks. Let the total number of matchsticks used by Robin be 𝑡.
What is the difference between the maximum possible value of 𝑡, and the minimum possible value of 𝑡?

P a g e 145 | 152
Get all the files at: https://bit.ly/azizhandouts
Aziz Manva ([email protected])

Because Robin is using matchsticks, he cannot use a fractional number of matchsticks.

𝑡 < 12 + 15 = 27 ⇒ 𝑡 ≤ 26
15 − 12 < 𝑡 ⇒ 3 < 𝑡 ⇒ 4 ≤ 𝑡

𝐷𝑖𝑓𝑓𝑒𝑟𝑒𝑛𝑐𝑒 = 26 − 4 = 22

Example 14.15
Two sides of a triangular plot to grow flowers are 30 inches and 39 inches, respectively.
A. If the third side of the plot is an integral number of feet long, what is its maximum possible length?
B. If the perimeter of the plot is an integral number of feet, what is its maximum possible value?

69 9
𝑆𝑢𝑚 𝑜𝑓 𝑇𝑤𝑜 𝑆𝑖𝑑𝑒𝑠 = ⏟
30 + 42 = 69
⏟ = =5

12 ⏟ 12
𝐼𝑛𝑐ℎ𝑒𝑠 𝐼𝑛𝑐ℎ𝑒𝑠
𝐹𝑒𝑒𝑡 𝐹𝑒𝑒𝑡
9
𝑇ℎ𝑖𝑟𝑑 𝑆𝑖𝑑𝑒 < 5 ⇒ 𝑀𝑎𝑥 = 5
12
F. Degenerate Triangles
A special case of invalid triangles important enough to have their own name are degenerate triangles. These
“triangles” are actually lines, but occur, for example, in the formula for the area of a triangle (using coordinate
geometry. If the area of the triangle is zero, then it is a degenerate triangle.

14.16: Degenerate “Triangles”-I


If point B lies on line segment AC, the three points are collinear. However, it is also a “triangle”.
Such triangles are called degenerate triangles.

Degenerate triangles do not satisfy the triangle inequality. For most questions, you will not consider degenerate
triangles as triangles. (If you need to, then the question will specify).

Example 14.17
If a triangle has side lengths 𝐴𝐵 = 8 𝑓𝑒𝑒𝑡, 𝐵𝐶 = 9 𝑓𝑒𝑒𝑡 𝑎𝑛𝑑 𝐴𝐶 = 17 𝑓𝑒𝑒𝑡, then
A. What kind of triangle is it?
B. What can be said about the points 𝐴, 𝐵 and 𝐶?

Part A
8 + 9 = 17 ⇒ 𝑎 + 𝑏 = 𝑐
Sum of two sides is equal to the third side.
This is not possible in any triangle other than a degenerate triangle.

Part B
Hence, the three points are collinear.

14.18: Degenerate “Triangles”-II


Degenerate triangles have zero area.

Example 14.19
A triangle has side lengths 𝐴𝐵 = 4 𝑓𝑒𝑒𝑡, 𝐵𝐶 = 5 𝑓𝑒𝑒𝑡 𝑎𝑛𝑑 𝐴𝐶 = 9 𝑓𝑒𝑒𝑡. What is the area of the triangle?

P a g e 146 | 152
Get all the files at: https://bit.ly/azizhandouts
Aziz Manva ([email protected])

4 + 5 = 9 ⇒ 𝑇𝑟𝑖𝑎𝑛𝑔𝑙𝑒 𝑖𝑠 𝑑𝑒𝑔𝑒𝑛𝑒𝑟𝑎𝑡𝑒 ⇒ 𝐴𝑟𝑒𝑎 𝑖𝑠 𝑍𝑒𝑟𝑜


G. Semiperimeter

14.20: Semiperimeter
Semiperimeter of a geometrical figure is half the perimeter.

Example 14.21
A triangle has side lengths of 3, 4 and 6. Find the semiperimeter.

𝑃 3 + 4 + 6 13
𝑠= = = = 6.5
2 2 2

14.22: Semiperimeter is greater than any side


𝑠>𝑎
𝑠>𝑏
𝑠>𝑐

𝑎+𝑏 >𝑐
𝑎 + 𝑏 + 𝑐 > 2𝑐
𝑎+𝑏+𝑐
>𝑐
2
𝑠>𝑐

Example 14.23
A triangle having integer sides has a perimeter of 23. What is the largest possible length of any one of its sides?

𝑃 23
𝑃𝑒𝑟𝑖𝑚𝑒𝑡𝑒𝑟 = 23 ⇒ 𝑠 = = = 11.5
2 2

𝐿𝑎𝑟𝑔𝑒𝑠𝑡 𝑝𝑜𝑠𝑠𝑖𝑏𝑙𝑒 𝑠𝑖𝑑𝑒 < 11.5

Try 11:
6,6,11 ⇒ 𝑉𝑎𝑙𝑖𝑑 𝑇𝑟𝑖𝑎𝑛𝑔𝑙𝑒
6 + 6 = 12 > 11

Example 14.24
In any triangle, the length of the longest side is less than half of the perimeter. All triangles with perimeter 57
and integer side lengths 𝑥, 𝑦, 𝑧, such that 𝑥 < 𝑦 < 𝑧 are constructed. How many such triangles are there? (Gauss
2013/24)

Triangles cannot be Isosceles


𝑃 = 𝑥 + 𝑦 + 𝑧 = 57

Maximum value of z is 57/2 =28.5


Since z is integer, maximum value is 28

𝑧 𝑥+𝑦 𝑃𝑎𝑖𝑟𝑠 Number of Pairs


28 29 (27,2)(26,3), … , (15,14) 27 − 15 + 1 = 13
27 30 (26,4)(25,5), … , (16,14) 26 − 14 + 1 = 11

P a g e 147 | 152
Get all the files at: https://bit.ly/azizhandouts
Aziz Manva ([email protected])

26 31 (25,6)(25,5), … , (16,15) 25 − 16 + 1 = 10
25 32 (24,8)(23,9), … , (17,15) 24 − 17 + 1 = 8
24 33 (23,10)(22,11), … , (17,16) 23 − 17 + 1 = 7
23 34 (22,12)(21,13), … , (18,16) 22 − 18 + 1 = 5
H. Average

14.2 Maximum, Minimum and Range


A. Max and Min

Example 14.25
Consider a triangle with sides lengths 3 and 4. Find the:
A. Maximum value of the length of the third side
B. Minimum value of the length of the third side
C. Range of values that the third side can take

Let the length of the third side be 𝑥.

We can find the maximum value of the length of the third side:
3+4>𝑥 ⇒𝑥 <7

Similarly, we can find the minimum value of the length of the third side:
4−3<𝑥 ⇒𝑥 >1

We can combine the above two inequalities to get the valid range of values that the third side can take:
1 < 𝑥 < 7 ⇒ 𝑥 ∈ (1,7)

Example 14.26
In a triangle with side lengths 5, 6 and 𝑥, what is the sum of all possible integral values of 𝑥? (MathCounts 1997
Warm-Up 12)

6 − 5 < 𝑥 < 11 ⇒ 1 < 𝑥 < 11 ⇒ 𝑥 ∈ {2,3,4, … ,10}

The sum of these values will be:


10 × 11
2 + 3 + ⋯ + 10 = (1 + 2 + 3 + ⋯ + 10) − 1 = − 1 = 55 − 1 = 54
2

Example 14.27
The sides of a triangle have lengths 6.5, 10, and 𝑠, where 𝑠
is a whole number. What is the smallest possible value of 𝑠?
(AMC 8 1992/17)

Example 14.28
What is the smallest whole number larger than the perimeter of any triangle with a side of length 5 and a side
of length 19? (AMC 8 2015/8)

P a g e 148 | 152
Get all the files at: https://bit.ly/azizhandouts
Aziz Manva ([email protected])

Example 14.29
Nondegenerate △ 𝐴𝐵𝐶 has integer side lengths, 𝐵𝐷 is an angle bisector, 𝐴𝐷 = 3, and 𝐷𝐶 = 8. What is the
smallest possible value of the perimeter? (AMC 10A 2010/16)

Example 14.30
In a triangle with integer side lengths, one side is three times as long as a second side, and the length of the
third side is 15. What is the greatest possible perimeter of the triangle? (AMC 10B 2006/10)

B. Range

Example 14.31
The sides of a triangle with positive area have lengths 4, 6, and 𝑥 . The sides of a second triangle with positive
area have lengths 4, 6, and 𝑦 . What is the smallest positive number that is not a possible value of |𝑥 − 𝑦|? (AMC
10 2000/10)

14.3 Geometrical Inequalities


A. Introduction
We can use the triangle inequality to establish other inequalities that triangles must fulfill. Some of these
inequalities can require a high degree of creativity to establish.

14.32: Medians and Semiperimeter


The sum of the lengths of the three medians of a triangle is not greater than the triangle's perimeter. (NMTC
Primary/Final 2004/15)

Let M, P and Q be the midpoints of sides BC, AC and AB in Triangle ABC. (Construction)
Extend AM such that AM = MD. (Construction)
TPT: AM + BP + CQ < AB + AC + BC

In Δ𝐴𝐵𝐷:
𝐴𝐷 < 𝐴𝐵 + 𝐵𝐷
𝐴𝐷 < 𝐴𝐵 + 𝐴𝐶
2𝐴𝑀 < 𝐴𝐵 + 𝐴𝐶 (𝐼𝑛𝑒𝑞𝑢𝑎𝑙𝑖𝑡𝑦 𝐼)

By similar argument as above, we can show that:


2BP < AB + BC (Inequality II)
2CQ < AC + BC (Inequality III)

Add Inequality I, II and III:


2(AM + BP + CQ) < 2(AB + AC + BC)
AM + BP + CQ < AB + AC + BC

14.4 Review
MCQ 14.33
𝑀𝑎𝑟𝑘 𝑎𝑙𝑙 𝑐𝑜𝑟𝑟𝑒𝑐𝑡 𝑜𝑝𝑡𝑖𝑜𝑛𝑠.
A triangle with zero area:
A. Does not exist

P a g e 149 | 152
Get all the files at: https://bit.ly/azizhandouts
Aziz Manva ([email protected])

B. Can be a line segment


C. Can be a single point
D. Is a degenerate triangle

Options A, B, and C

MCQ 14.34
𝑀𝑎𝑟𝑘 𝑡ℎ𝑒 𝑐𝑜𝑟𝑟𝑒𝑐𝑡 𝑜𝑝𝑡𝑖𝑜𝑛
The longest side of a triangle:
A. Is its hypotenuse
B. Must be its hypotenuse
C. Must be greater than the sum of the other two sides
D. Must be less than the sum of the other two sides.

We do not know whether the triangle is right-angled. So, we put aside Options A and B, and look for other
options.

Option C is incorrect, and violates the triangle inequality


𝑎+𝑏 <𝑐
Option D states the triangle inequality correctly.
Option D is correct.

14.5 AMC Questions


Example 14.35
𝐴𝑇 𝐵𝑇
In △ 𝐴𝐵𝐶, points 𝐷 and 𝐸 lie on 𝐵𝐶 and 𝐴𝐶, respectively. If 𝐴𝐷 and 𝐵𝐸 intersect at 𝑇 so that = 3 and = 4,
𝐷𝑇 𝐸𝑇
𝐶𝐷
what is 𝐵𝐷
? (AMC 10B 2004/20)

(Official Solution, Adapted)

Example 14.36
In △ 𝐴𝐵𝐶, we have 𝐴𝐶 = 𝐵𝐶 = 7 and 𝐴𝐵 = 2. Suppose that 𝐷 is a point on line 𝐴𝐵 such that 𝐵 lies between
𝐴 and 𝐷 and 𝐶𝐷 = 8. What is 𝐵𝐷? (AMC 10B 2005/10)

Example 14.37
Points 𝐴, 𝐵, 𝐶 and 𝐷 lie on a line, in that order, with 𝐴𝐵 = 𝐶𝐷 𝑎𝑛𝑑 𝐵𝐶 = 12. Point 𝐸 is not on the line, and
𝐵𝐸 = 𝐶𝐸 = 10. The perimeter of △ 𝐴𝐸𝐷 is twice the perimeter of △ 𝐵𝐸𝐶. 𝐹𝑖𝑛𝑑 𝐴𝐵. (AMC 10A 2002/23)

P a g e 150 | 152
Get all the files at: https://bit.ly/azizhandouts
Aziz Manva ([email protected])

Example 14.38
The area of △ 𝐸𝐵𝐷 is one third of the area of 3 − 4 − 5 △ 𝐴𝐵𝐶. Segment 𝐷𝐸 is perpendicular to segment 𝐴𝐵.
What is 𝐵𝐷? (AMC 10B 2011/9)

Example 14.39
Let 𝑇1 be a triangle with sides 2011, 2012, and 2013. For 𝑛 ≥ 1, if 𝑇𝑛 =△ 𝐴𝐵𝐶 and 𝐷, 𝐸, and 𝐹 are the points of
tangency of the incircle of △ 𝐴𝐵𝐶 to the sides 𝐴𝐵, 𝐵𝐶 and 𝐴𝐶, respectively, then 𝑇𝑛+1 is a triangle with side
lengths 𝐴𝐷, 𝐵𝐸, and 𝐶𝐹, if it exists. What is the perimeter of the last triangle in the sequence (𝑇𝑛 )? (AMC 10B
2011/25)

Example 14.40
In triangle 𝐴𝐵𝐶, 𝐴𝐵 = 13, 𝐵𝐶 = 14, and 𝐶𝐴 = 15. Distinct points 𝐷, 𝐸, and 𝐹 lie on segments 𝐵𝐶, 𝐶𝐴, and 𝐷𝐸,
𝑚
respectively, such that 𝐴𝐷 ⊥ 𝐵𝐶, 𝐷𝐸 ⊥ 𝐴𝐶, and 𝐴𝐹 ⊥ 𝐵𝐹. The length of segment 𝐷𝐹 can be written as , where
𝑛
𝑚 and 𝑛 are relatively prime positive integers. What is 𝑚 + 𝑛? (AMC 10B 2013/23)

Example 14.41
In a given plane, points 𝐴 and 𝐵 are 10 units apart. How many points 𝐶 are there in the plane such that the
perimeter of △ 𝐴𝐵𝐶 is 50 units and the area of △ 𝐴𝐵𝐶 is 100 square units? (AMC 10B 2019/10)

Example 14.42
Right triangles 𝑇1 and 𝑇2 have areas 1 and 2, respectively. A side of 𝑇_1 is congruent to a side of 𝑇2 , and a
different side of 𝑇1 is congruent to a different side of 𝑇2 . What is the square of the product of the other (third)
sides of 𝑇1 and 𝑇2 ?(AMC 10B 2019/15)

Example 14.43
In △ 𝐴𝐵𝐶 with a right angle at 𝐶, point 𝐷 lies in the interior of 𝐴𝐵 and point 𝐸 lies in the interior of 𝐵𝐶 so that
𝐴𝐶 = 𝐶𝐷, 𝐷𝐸 = 𝐸𝐵, and the ratio 𝐴𝐶: 𝐷𝐸 = 4: 3. What is the ratio 𝐴𝐷: 𝐷𝐵? (AMC 10B 2019/16)

Example 14.44
Points 𝑃 and 𝑄 lie in a plane with 𝑃𝑄 = 8. How many locations for point 𝑅 in this plane are there such that the
triangle with vertices 𝑃, 𝑄, and R is a right triangle with area 12 square units? (AMC 10B 2020/8)

45 Examples

P a g e 151 | 152
Get all the files at: https://bit.ly/azizhandouts
Aziz Manva ([email protected])

P a g e 152 | 152

You might also like